You are on page 1of 316

StudyMRCOG

MRCOGPart 1
Anatomy
Compiled by StudyMRCOG Mentor’s team

These questions are complied from various sources/materials for MRCOG/MRCPI


study purpose, STUDYMRCOG doesn't own any credit or doesn't hold any copy rights
on this & used only for educational purpose.
1.Whilst examining the abdomen of a 21-year-old female with abdominal pain you notice a
well-defined “six-pack”. Which muscle is this?
A. Rectus abdominis
B. Transversus abdominis
C. Cremaster
D. External oblique
E. Internal oblique

2. Which of the following is a branch of the femoral artery?


A. Peroneal artery
B. The ascending genicular artery
C. The deep epigastric artery
D. Medial circumflex iliac artery
E. Superficial circumflex iliac artery

3. From which of the following is the nerve supply to


the vulva derived?
A. Genitofemoral nerve
B. Ilioinguinal nerve
C. Pudendal nerve
D. All the above
E. None of the above
4. During an operation, a midline incision was made at an anatomic location 2 cm below the
umbilicus. Which of the following lists (in order) the layers of the anterior abdominal wall as
they would be incised or separated?
(A) skin, subcutaneous fat, superficial fascia (Camper’s), deep fascia (Scarpa’s), fascial muscle
cover (anterior rectus sheath), rectus muscle, a deep fascial muscle cover (posterior rectus
sheath), preperitoneal fat, and peritoneum
(B) skin, subcutaneous fat, superficial fascia (Scarpa’s), deep fascia (Camper’s), fascial muscle
covering (anterior abdominal sheath), transverse abdominal muscle, a deep fascial muscle
cover (posterior rectus sheath), preperitoneal fat, and peritoneum
(C) skin, subcutaneous fat, superficial fascia (Camper’s), deep fascia (Scarpa’s), fascial muscle
cover (anterior rectus sheath), rectus muscle, a deep fascial muscle cover (posterior rectus
sheath), peritoneum, and preperitoneal fat
(D) skin, subcutaneous fat, superficial fascia (Scarpa’s), deep fascia (Camper’s), fascial muscle
cover (anterior rectus sheath), rectus muscle, a deep fascial muscle cover (posterior rectus
sheath), preperitoneal fat, and peritoneum
(E) skin, subcutaneous fat, superficial fascia (Camper’s), deep fascia (Scarpa’s), fascial muscle
cover (anterior rectus sheath), transverse abdominal muscle, a deep fascial muscle covering
(posterior rectus sheath), preperitoneal fat, and peritoneum

5. When performing a hysterectomy, the surgeon should be aware that at its closest position to
the cervix, the ureter is normally separated from the cervix by which of the following distances?
(A) 0.5 mm
(B) 1.2 mm
(C) 12 mm
(D) 3 cm
(E) 5 cm

6. Which artery supplies the structures derived from the foregut of the embryo?
A Coeliac trunk
B Inferior mesenteric
C Middle rectal
D Renal
E Superior mesenteric

7. Which image shows the engaging diameter of Brow

A B C D
8. A 27-year-old woman has a cervical smear result which shows ‘borderline’
changes. Which cells line the ectocervix?
A Ciliated cells
B Columnar epithelium
C Cuboidal epithelium
D Smooth muscle cells
E Stratified squamous epithelium
9. What is the uterine corpus mainly composed of?
(A) fibrous tissue
(B) estrogen receptors
(C) smooth muscle
(D) elastic tissue
(E) endometrium
10. A 32-year-old woman undergoes an emergency caesarean section for failure to progress at
9 cm cervical dilatation. Which of the following correctly describes the pelvic shape which has
an anteroposterior diameter of the inlet, greater than the transverse diameter?
A Android
B Anthropoid
C Gynaecoid
D Male
E Platypelloid
1.Whilst examining the abdomen of a 21-year-old female with abdominal pain you notice a
well-defined “six-pack”. Which muscle is this?
A. Rectus abdominis
B. Transversus abdominis
C. Cremaster
D. External oblique
E. Internal oblique

Answer A Rectus abdominis

2. Which of the following is a branch of the femoral artery?


A. Peroneal artery
B. The ascending genicular artery
C. The deep epigastric artery
D. Medial circumflex iliac artery
E. Superficial circumflex iliac artery

Answer E Superficial circumflex iliac artery

3. From which of the following is the nerve supply to


the vulva derived?
A. Genitofemoral nerve
B. Ilioinguinal nerve
C. Pudendal nerve
D. All the above
E. None of the above

Answer D. All the above

4. During an operation, a midline incision was made at an anatomic location 2 cm below the
umbilicus.
Which of the following lists (in order) the layers of the anterior abdominal wall as they would be
incised or separated?
(A) skin, subcutaneous fat, superficial fascia (Camper’s), deep fascia (Scarpa’s), fascial muscle
cover (anterior rectus sheath), rectus muscle, a deep fascial muscle cover (posterior rectus
sheath), preperitoneal fat, and peritoneum
(B) skin, subcutaneous fat, superficial fascia (Scarpa’s), deep fascia (Camper’s), fascial muscle
covering (anterior abdominal sheath), transverse abdominal muscle, a deep fascial muscle
cover (posterior rectus sheath), preperitoneal fat, and peritoneum
(C) skin, subcutaneous fat, superficial fascia (Camper’s), deep fascia (Scarpa’s), fascial muscle
cover (anterior rectus sheath), rectus muscle, a deep fascial muscle cover (posterior rectus
sheath), peritoneum, and preperitoneal fat
(D) skin, subcutaneous fat, superficial fascia (Scarpa’s), deep fascia (Camper’s), fascial muscle
cover (anterior rectus sheath), rectus muscle, a deep fascial muscle cover (posterior rectus
sheath), preperitoneal fat, and peritoneum
(E) skin, subcutaneous fat, superficial fascia (Camper’s), deep fascia (Scarpa’s), fascial muscle
cover (anterior rectus sheath), transverse abdominal muscle, a deep fascial muscle covering
(posterior rectus sheath), preperitoneal fat, and peritoneum

Answer (A) Layers at the midline of the abdominal wall, 2 cm below the umbilicus that would be
incised orseparated are skin, subcutaneous fat, superficial fascia (Camper’s), deep fascia
(Scarpa’s), and thefascial muscle coverings (anterior rectus sheath). The rectus muscles would
be separated and the deep fascial layer (posterior rectus sheath), preperitoneal fat, and
peritoneum would be incised. The posterior rectus sheath is only present cephalad to the
arcuate line. Camper’s is the most superficialfascia and transversus abdominal muscle would
not be found in the midline
5. When performing a hysterectomy, the surgeon should be aware that at its closest position to
the cervix, the ureter is normally separated from the cervix by which of the following distances?
(A) 0.5 mm
(B) 1.2 mm
(C) 12 mm
(D) 3 cm
(E) 5 cm

Answer (C) 12mm


A surgeon has a little more than a 1-cm space between the cervix and the ureter when
performing a hysterectomy. Just lateral to the cervix is a high-risk area for injury to the ureter
during gynecologic surgery. The importance of dissecting away the bladder, staying close to the
cervix, and not placing clamps too far laterally or inserting wide sutures is apparent. At times, it
is necessary to dissect enough to allow visualization of both ureters prior to ligation of the
uterine arteries.

6. Which artery supplies the structures derived from the foregut of the embryo?
A Coeliac trunk
B Inferior mesenteric
C Middle rectal
D Renal
E Superior mesenteric
Answer A Coeliac trunk
The embryonic foregut is the part which forms the mouth to the duodenum.
The coeliac trunk is the first branch of the aorta once it has passed through the diaphragm. The
coeliac trunk then branches into three: to the left gastric artery, the splenic artery and the
common hepatic arteries. The superior mesenteric artery provides blood supply to the
embryonic midgut and he inferior mesenteric to the embryonic hindgut.

7. Which image shows the engaging diameter of Brow

A B C D

Answer C
8. A 27-year-old woman has a cervical smear result which shows ‘borderline’
changes.
Which cells line the ectocervix?
A Ciliated cells
B Columnar epithelium
C Cuboidal epithelium
D Smooth muscle cells
E Stratified squamous epithelium

Answer E Stratified squamous epithelium


The cervix has a conical shape with a varied epithelium. The ectocervix is the lower intravaginal
portion of the cervix and is lined by non-keratinised stratified squamous epithelium. The
endocervix is the cavity of the cervix, linking the external and the internal os. It is lined by
mucin-secreting simple columnar epithelium. The border between these two types of epithelium
is the squamocolumnar junction, or transformation zone. The transformation zone is the area
where metaplasia frequently takes place and it is from here that the cervical smear test is taken
.There are certain times when metaplasia is physiological, such as during puberty when the
endocervix everts and postmenopause when the transformation moves upwards.

9. What is the uterine corpus mainly composed of?


(A) fibrous tissue
(B) estrogen receptors
(C) smooth muscle
(D) elastic tissue
(E) endometrium

Answer (C) The uterus has a body (corpus) composed mainly of smooth muscle, and a cervix
composed mainly of connective and elastic tissues that are joined by a transitional portion
(isthmus). It is an estrogen-dependent organ measuring about 7.5 cm long x 5 cm wide, with a
4-cm anterior-to-posterior diameter. After puberty, the uterus weighs about 50 g in the nullipara
and 70 g in the multipara. It lies between the bladder anteriorly and the pouch of Douglas in
front of the rectum posteriorly, with the cervical portion extending from the intraperitoneal area
into the vagina. The opening at the distal tip of the cervix is called the external os. It is
connected by the cervical canal to the internal os, which is located just below the endometrial
cavity. This cavity is lined by an epithelium, the endometrium.

10. A 32-year-old woman undergoes an emergency caesarean section for failure to progress at
9 cm cervical dilatation. Which of the following correctly describes the pelvic shape which has
an anteroposterior diameter of the inlet, greater than the transverse diameter?
A Android
B Anthropoid
C Gynaecoid
D Male
E Platypelloid

Answer B Anthropoid
The basic shapes of the pelvis are as follows:
• Gynaecoid pelvis (50%): normal female type, inlet is slightly transverse oval; sacrum is wide
with average concavity and inclination; subpubic angle is 90–100°.
• Anthropoid pelvis (25%): ape-like; anteroposterior (AP) diameters are long; transverse
diameter short; sacrum long and narrow, subpubic angle is narrow.
• Android pelvis (20%): male type, inlet is triangular or heart-shaped with anterior narrow
apex, subpubic angle is narrow < 90°.
• Platypelloid pelvis (5%): flat female type, AP diameter is short, transverse diameter is long,
subpubic angle is wide.
1. Which of the following is true regarding the human rectum?
A. Drains lymph to the pre-aortic nodes.
B. Has mesentery in the posterior third.
C. Is covered anteriorly by peritoneum along its whole length.
D. Has a blood supply from the terminal branches of the superior mesenteric artery.
E. Has appendices epiploicae.

2. From the figures given below, which one shows Anthropoid

A B

C D
3.A 63-year-old man in the surgical ward is complaining of numbness over the anterior thigh
and medial aspect of his right leg. He is unable to extend his right knee and the knee jerk is
reduced. He had undergone a femoral aneurysm repair 3 days ago.
Which is the most probable nerve damaged in this case?
A. Femoral nerve
B. Saphenous nerve
C. Femoral branch of genitofemoral nerve
D. Genital branch of genitofemoral nerve
E. Sciatic nerve

4. Presence of vaginal septa is not associated with which of the following?


A. Dysmenorrhoea
B. Dyspareunia
C. Obstructed labour
D. Uterine abnormalities
E. Easy removal in most of the cases

5. A healthy 5 ft 6 in. tall, adult female is most likely to have a pelvic inlet that would be
classified as which of the following Caldwell-Moloy types?
(A) android
(B) platypelloid
(C) anthropoid
(D) gynecoid
(E) triangular

6. The part of the pelvis lying above the linea ter-minalis has little effect on a woman’s ability to
deliver a baby vaginally. What is the name of this portion of the pelvis?
(A) true pelvis
(B) midplane
(C) outlet
(D) false pelvis
(E) sacrum

7. A patient presents approximately 10 years post-menopausal with complaints of pressure


vaginally and the sensation that something is falling out. When told she has a fallen uterus, she
wonders if it is due to the damage from her round ligaments since she had a great deal of round
ligament pain during her pregnancies. Which of the following ligaments provide the most
support to the uterus in terms of
preventing prolapse?
(A) broad ligaments
(B) round ligaments
(C) utero-ovarian ligaments
(D) cardinal ligaments
(E) arcuate ligament
8. The anatomy of the spinal cord and dural space is important when giving regional spinal
anesthesia.
At what approximate spinal level do the dural space and the spinal cord, respectively, end?
(A) T10, T8
(B) L2, T10
(C) L5, T12
(D) S2, L2
(E) S5, S2

9.What is the nerve root of the ilioinguinal nerve?


A T12 and L1
B L1
C L1 and L2
D L2
E L2 and L3

10.Which of the following organs is derived from ectodermal neural crest cells?
A Adrenal gland inner medulla
B Adrenal gland outer cortex
C Liver
D Pancreas
E Spleen
1. Which of the following is true regarding the human rectum?
A. Drains lymph to the pre-aortic nodes.
B. Has mesentery in the posterior third.
C. Is covered anteriorly by peritoneum along its whole length.
D. Has a blood supply from the terminal branches of the superior mesenteric artery.
E. Has appendices epiploicae.
Answer A
2. From the figures given below, which one shows Anthropoid
Answer D
3.A 63-year-old man in the surgical ward is complaining of numbness over the anterior thigh
and medial aspect of his right leg. He is unable to extend his right knee and the knee jerk is
reduced. He had undergone a femoral aneurysm repair 3 days ago.
Which is the most probable nerve damaged in this case?
A. Femoral nerve
B. Saphenous nerve
C. Femoral branch of genitofemoral nerve
D. Genital branch of genitofemoral nerve
E. Sciatic nerve

Answer A

4. Presence of vaginal septa is not associated with which of the following?


A. Dysmenorrhoea
B. Dyspareunia
C. Obstructed labour
D. Uterine abnormalities
E. Easy removal in most of the cases

Answer A

5. A healthy 5 ft 6 in. tall, adult female is most likely to have a pelvic inlet that would be
classified as
which of the following Caldwell-Moloy types?
(A) android
(B) platypelloid
(C) anthropoid
(D) gynecoid
(E) triangular

Answer (D) Pelvises in most U.S. women are gynecoid, but they may be of a mixed type (for
instance, having a gynecoid forepelvis and an anthropoid posterior pelvis). The obstetrician has
to judge the capacity of the pelvis on the basis of its total configuration, including midplane and
outlet capacities, and always in relation to the size and position of the fetus.

6. The part of the pelvis lying above the linea ter-minalis has little effect on a woman’s ability to
deliver a baby vaginally. What is the name of this portion of the pelvis?
(A) true pelvis
(B) midplane
(C) outlet
(D) false pelvis
(E) sacrum

Asnwer (D) The false pelvis or pelvis major lies above the linea terminalis. It seldom affects
obstetric management, and measurements of the iliac crest flare do not usually aid in
determining the size of the true pelvis. An important measurable indicator of the size of the true
pelvis is the inter-spinous diameter.

7. A patient presents approximately 10 years post-menopausal with complaints of pressure


vaginally and the sensation that something is falling out. When told she has a fallen uterus, she
wonders if it is due to the damage from her round ligaments since she had a great deal of round
ligament pain during her pregnancies. Which of the following ligaments provide the most
support to the uterus in terms of preventing prolapse?
(A) broad ligaments
(B) round ligaments
(C) utero-ovarian ligaments
(D) cardinal ligaments
(E) arcuate ligament

Answer (D) The cardinal ligaments are also called the transverse cervical ligaments, or
Mackenrodt’s ligaments, and are considered part of the uterosacral ligament complex. These
ligaments serve as the major support for the apex of the vagina and are severed at the time of
hysterectomy. Once divided at hysterectomy, vaginal vault prolapse becomes more likely. The
broad ligaments are mainly peritoneum and the round ligaments mainly muscle. Neither
provides much support. The arcuate ligament is not attached to the uterus.
8. The anatomy of the spinal cord and dural space is important when giving regional spinal
anesthesia.
At what approximate spinal level do the dural space and the spinal cord, respectively, end?
(A) T10, T8
(B) L2, T10
(C) L5, T12
(D) S2, L2
(E) S5, S2

Answer (D) The spinal cord ends within the dura at about L2. The dural space ends at about
S2. The filum terminal and cauda equina extend within the dura for some distance after the
spinal cord ends. Caudal anesthesia intercepts the spinal nerves after they emerge from the
dural space. When giving spinal anesthesia, one should recognize that one usually enters the
subarachnoid space at or below the termination of the spinal cord. The cauda equina extends
for some distance within the dura. This relationship allows for effective anesthesia and
analgesia with minimal risk of injury to the spinal cord.

9. What is the nerve root of the ilioinguinal nerve?


A T12 and L1
B L1
C L1 and L2
D L2
E L2 and L3

Answer B L1
The ilioinguinal nerve arises from the L1 nerve root along with the larger iliohypogastric nerve. It
travels obliquely across the quadratus lumborum and perforates the transversus abdominis
near the anterior part of the iliac crest. It travels through part of the inguinal canal, passing
through the superficial inguinal ring. It supplies the mons pubis and labium majus.

10. Which of the following organs is derived from ectodermal neural crest cells?
A Adrenal gland inner medulla
B Adrenal gland outer cortex
C Liver
D Pancreas
E Spleen

Answer A Adrenal gland inner medulla


The adrenal glands are retroperitoneal endocrine organs and are situated near the kidneys.
They are surrounded by adipose tissue and renal fascia and are usually found at the level of the
12th thoracic vertebra. The outer cortex is mainly responsible for the synthesis of corticosteroid
hormones and aldosterone and is derived from coelomic mesothelium. The inner medulla
chromaffin cells are the source of catecholamines and these cells are derived from ectodermal
neural crest cells.
1.A 26-year-old female presents with right iliac fossa pain and is taken to theatre for an
appendectomy. An incision is made through the skin and onto muscle with fibres passing
inferiorly in an oblique direction. Which muscle is cut in this scenario?
A. Internal oblique
B. External oblique
C. Rectus abdominis
D. Transversus abdominis
E. Pyramidalis

2. Regarding the blood supply of anterior abdominal wall, which is not true?
A.Cutaneous branches from superior and inferior epigastric arteries supply the flanks.
B.Cutaneous branches from the superior and inferior epigastric arteries supply the area near
the midline.
C. Branches from the intercostal and lumbar arteries supply the flanks.
D. All the above.
E. None of the above

3. Which of the following structures pass under the inguinal ligament?


A. The long saphenous vein
B. The superficial femoral artery
C. The superficial epigastric vein
D. The genital branch of the genitofemoral nerve
E. The femoral branch of the genitofemoral nerve

4.A 33-year-old lady, who is 38 weeks pregnant, presents to her general practitioner with a 4-
week history of pain and paraesthesia over the upper outer aspect of her right thigh. There is
no restriction of movements in her hips or knees and her gait is normal. Which is the most likely
nerve, which is affected in this case?
A. Lateral cutaneous nerve of thigh
B. Femoral nerve
C. Common peroneal nerve
D. Sciatic nerve
E. Tibial nerve

5. Which of the following organs does not respond to oestrogens?


A. Breasts
B. Cervical glands
C. Fallopian tube
D. Vaginal epithelium
E. None of the above

6. Which of the following organs is derived from the ectodermal neural crest cells?
A. Inner medulla of adrenal glands
B. Outer cortex of adrenal glands
C. Liver
D. Spleen
E. Pancreas

7. Which of the following layers shows external oblique muscle


8. Which of the following is the best description of the pelvic diaphragm?
(A) made up mainly by the coccygeus
(B) covered on one side by fascia and on the other by peritoneum
(C) a muscle innervated by L2, L3, and L4
(D) an extension of the sacrococcygeal ligament
(E) synonymous with the pelvic floor
9. A patient develops a neurologic disease that destroys components of S2, S3, S4 bilaterally.
What clinical manifestation would you expect the patient to have as a result?
(A) inability to abduct her thigh
(B) rectal incontinence
(C) painless menses
(D) labor without pain
(E) inability to extend her knees
10. During delivery of a first twin, a very tight nuchal cord is reduced from the baby’s neck by
clamping and dividing it. After this, the second twin (as yet unborn) develops severe fetal
distress. Of the following, what is the most likely mechanism for the distress in the second twin?
(A) a twin-to-twin transfusion before birth
(B) the second twin may no longer be connected to its placenta
(C) placenta previa in the second twin
(D) amniotic fluid embolism
(E) uterine rupture
1.A 26-year-old female presents with right iliac fossa pain and is taken to theatre for an
appendectomy. An incision is made through the skin and onto muscle with fibres passing
inferiorly in an oblique direction. Which muscle is cut in this scenario?
A. Internal oblique
B. External oblique
C. Rectus abdominis
D. Transversus abdominis
E. Pyramidalis

Answer B

2. Regarding the blood supply of anterior abdominal wall, which is not true?
A.Cutaneous branches from superior and inferior epigastric arteries supply the flanks.
B.Cutaneous branches from the superior and inferior epigastric arteries supply the area near
the midline.
C. Branches from the intercostal and lumbar arteries supply the flanks.
D. All the above.
E. None of the above

Answer A

3. Which of the following structures pass under the inguinal ligament?


A. The long saphenous vein
B. The superficial femoral artery
C. The superficial epigastric vein
D. The genital branch of the genitofemoral nerve
E. The femoral branch of the genitofemoral nerve

Answer E

4.A 33-year-old lady, who is 38 weeks pregnant, presents to her general practitioner with a 4-
week history of pain and paraesthesia over the upper outer aspect of her right thigh. There is
no restriction of movements in her hips or knees and her gait is normal. Which is the most likely
nerve, which is affected in this case?
A. Lateral cutaneous nerve of thigh
B. Femoral nerve
C. Common peroneal nerve
D. Sciatic nerve
E. Tibial nerve

Answer A

5. Which of the following organs does not respond to oestrogens?


A. Breasts
B. Cervical glands
C. Fallopian tube
D. Vaginal epithelium
E. None of the above

Answer E

6.Which of the following organs is derived from the


ectodermal neural crest cells?
A. Inner medulla of adrenal glands
B. Outer cortex of adrenal glands
C. Liver
D. Spleen
E. Pancreas

Answer A
7. Which of the following layers shows external oblique muscle

Answer A
8. Which of the following is the best description of the pelvic diaphragm?
(A) made up mainly by the coccygeus
(B) covered on one side by fascia and on the other by peritoneum
(C) a muscle innervated by L2, L3, and L4
(D) an extension of the sacrococcygeal ligament
(E) synonymous with the pelvic floor

E) The pelvic diaphragm (also called the pelvic floor) is made up of the levator ani muscle and
the coccygeus. It is connected to the pelvic
side-wall by its attachment to the obturator
internus muscle at the arcus tendineus. The
pelvic diaphragm provides support and
closure for the intraperitoneal cavity
caudally just as the thoracic diaphragm
provides closure in the cephalad direction.
It is covered by fascia on both sides and
innervated from S2, S3, S4. The potential
spaces through which the vagina, urethra,
and rectum pass are the possible sites of
pelvic prolapse
9. A patient develops a neurologic disease that destroys components of S2, S3, S4 bilaterally.
What
clinical manifestation would you expect the patient to have as a result?
(A) inability to abduct her thigh
(B) rectal incontinence
(C) painless menses
(D) labor without pain
(E) inability to extend her knees

Answer (B) The S2, S3, S4 innervation, if damaged at the level of the spinal cord, is most likely
to produce incontinence of bladder or bowel. The patient may also have decreased vulvar
sensation. Uterine pain with labor or menses is mediated by the sympathetic and
parasympathetic system. Movement of the leg is mediated by L2-L4.

10. During delivery of a first twin, a very tight nuchal cord is reduced from the baby’s neck by
clamping and dividing it. After this, the second twin (as yet unborn) develops severe fetal
distress. Of the following, what is the most likely mechanism for the distress in the second twin?
(A) a twin-to-twin transfusion before birth
(B) the second twin may no longer be connected to its placenta
(C) placenta previa in the second twin
(D) amniotic fluid embolism
(E) uterine rupture
Answer (B) In this case placenta previa can be ruled out because the first twin has already been
delivered through the cervix. If there had been a severe twin-twin transfusion, it would be
unlikely to manifest itself at this time in the pregnancy. An amniotic fluid embolism does not
affect the fetus but rather the mother. Uterine rupture with no other signs and occurring at that
precise time would be unlikely. That leaves us with a cord accident. Using our knowledge of the
placenta, we know that there may be one placenta or two, but we know that both babies have
their own umbilical cord. The cord wrapped around the neck of the first twin might belong to the
second twin!
StudyMRCOG
Physiology
Dr Chitra
1. Which of the following is not a physiological change in pregnancy?
a. Increase in plasma volume by 50%.
b. Increase in red cell mass by 30%.
c. Increase in coaugulation factors.
d. decrease in platelets by 10%
e. Decrease in MCV
ans e-decrease in MCV
physio changes in pregnancy-
Increase in plasma vol by 50%
Increase in red cell mass by 30%
Increased level of coagulation factors
Decfrease in platelet count
Increased MCV

2. Regarding haemoglobin which of the following is true?


a.It is a pentamer
b.1 gm of haemoglobin combines with 1.34 ml of O2
c.Shift of HbA to HbFb starts at 4 mths of life.
d.Normal store of Vit B12 is 2-4 mg & lasts for 2-4 days
e.Absorption of iron is mainly ion ferrous form.
Ans b—1 gm of Hb combines with 1-34 ml of O2.
Hb is a tetramer-4 haem grps & 4 globin chains
HbA is made up of 2 α & 2 β globin chains.switch from HbF to HbA starts at birth & is
complete by 6 mths.
Iron is mainly absorbed in ferric form from the duodenum & jejunum.
Vit B12 is found mainly in animal products.daily requirement is 1-2 ug .body stores of 2-3 mg
last for 2-3 years

3.One of the following is a pathological change in pregnancy.


a. Increase in response to Angiotensin
b. Heart rate is increased by 15 beats /min
c. Cardiac output is increased by 30-50%
d. Reduction in systolic & diastolic pressure
e. 50% increase in plasma volume.

Ans a—increased response to Angiotensin


In a normal pregnancy ,cardiac output increases by 30-50%,due to increase in stoke volume
& increase in heart rate.
Plasma volume expands by around 50%
There is a fall in BP –both systolic & diastolic

4.Respiratory system changes in pregnancy are


a. Functional residual capacity is reduced
b. Oxygen demand is increased
c. Respiratory rate increases
d. Minute ventilation increases
e. All of the above.

Ans e—all of the above


In pregnancy functional residual capacity is the only one to be reduced.all aother parameters
are increased
O2 demand is increased
Minute ventilation is increased
Respiratory rate is increased
There is mild respiratory alkalosis.

5.Renal changes in pregnancy are all except


a. Right ureter is more dilated than left ureter
b. 30% of women develop asymptomatic bacteruria
c. Kidney size increases by 1.5 cm
d. Sigmoid colon is responsible for dextrorotation of uterus
e. There is decreased risk of urinary tract infection in pregnancy.

Ans e- there is decreased risk of UTI in pregnancy


There is increase in kidney size by 1.5cm-collecting system & ureters dilate
Right ureter dilates more than left –due to dextrorotation of uterus & pressure of IVC on the
right ureter
The dilated collecting system holds upto 200-300 ml of urine which acts as a reservoir for
bacteria-leading to increased risk of UTI in pregnancy.

6. How is MAP calculated ?


a. 1/3 diastolic BP + 2/3 systolic BP
b. 2/3 diastolic BP + Systolic BP
c. Diastolic BP + systolic BP
d. Diastolic BP + 1/3 pulse pressure
e. Systolic +1/3 pulse pressure
Ans d– diastolic BP + 1/3 pulse pressure
Pulse pressure = systolic pressure – diastolic pressure

7.In an ECG which lead reflects the inferior wall of heart ?


a. V1,V2
b. V3,V4
c. aVR
d. 1,aVL
e. II,III,aVF
Ans e—II,III,aVF
V1,V2,V3 & V4 view the anterior portion of heart & are called anterior leads
V5 & V6 look at the lateral wall

8.What happens in Horners syndrome?


a. Loss of motor supply.
b. Excessive sympathetic stimulation
c. Excessive parasympathetic stimulation
d. Loss of sympathetic supply
e. Loss of parasympathetic supply

Ans d—loss of sympathetic supply


There is damage to sympathetic nerves of the face resulting in miosis-constriction of
pupil,ptosis-drooping of upper eyelid & anhidrosis-absence of sweating of the face.

9.The enzyme in red blood cells which buffers blood CO2 is?
a. Cytochrome –b5 reductase
b. 5-nucleotidase
c. Glucose -6-phosphate dehydrogenase
d. Carbonic anhydrase
e. Pyruvate kinase

Ans –carbonic anhydrase - aids in conversion of Co2 to carbonic acid & bicarbonate
ions.found in RBCs & Renal tubules.
When RBCs reach the lungs,the same enzyme helps to convert bicarbonate ions to CO2
which is then breathed out.

10.In which part of the cell does glycolysis take place?


a. Cytoplasm
b. Mitochondrial matrix
c. Mitochondrial cristae
d. Rough endoplasmic reticulum
e. Smooth endoplasmic reticulum

Ans a cytoplasm –glycolysis is the first step in breakdown of glucose to extract energy for
cellular metabolism.
Converts glucose into pyruvate & a hydrogen ion.
1.A 32-year-old primiparous woman suffers a massive antepartum haemorrhage and
undergoes an emergency caesarean section. During the caesarean section she suffers a
massive obstetric haemorrhage requiring transfusion of blood and clotting factors. As a result
of these events, she develops disseminated intravascular coagulopathy.
Which of the following laboratory results would be expected in this condition?
A Decreased activated partial thromboplastin time
B Increased factor VII levels
C Increased fibrinogen
D Increased soluble fibrin
E Thrombocytopenia

2.A 72-year-old woman undergoes a total abdominal hysterectomy. She has chronic obstructive
pulmonary disease. Postoperatively, she is difficult to extubate and has a prolonged stay on the
intensive care unit.
Which of the following is the most important direct stimulus to respiration?
A Decreased arterial pH
B Decreased arterial pO2
C Decreased arterial pCO2
D Increased H+ concentration of the cerebrospinal fluid (CSF)
E Increased pCO2 of the CSF

3. Normal clotting of blood requires


A. Inactivation of heparin
B. Inactivation of plasmin (fibrinolysin)
C. Calcium ions
D. An adequate intake of vitamin D
E. An adequate intake of vitamin C

4. Which of the following substances occur in lower concentration in foetal blood in comparison
to the maternal blood?
A. Amino acids
B. Ca2+
C. CO2
D. Glucose
E. Urea

5. Identify the blood cell


6. Which of the following is true about the smooth muscle cells?
A. Controlled by the autonomic nervous system
B. Do not contain actin and myosin
C. Mitochondria are absent
D. Present a striated appearance
E. Do not cause spontaneous muscle contraction

7. Which of the following regions of nephron does the water leave via diffusion?
A. Descending thin limb
B. Thick ascending limb
C. Proximal convoluted tubule
D. Distal convoluted tubule
E. Bowman’s capsule

8. Sympathetic drive to the heart is increased in which of the following conditions?


A. In exercise
B. In excitement
C. In hypotension
D. None of the above
E. All of the above
9. Which of the following occurs during the isometric ventricular contraction?
A. The entry and exit valves of the ventricle are closed
B. Pressure in the aorta rises
C. Pressure in the atria falls
D. Left coronary blood flow rises
E. The rate of rise in pressure is greater in the right than in the left ventricle

10. What is the main site of iron absorption?


A. Stomach
B. Rectum
C. Upper small intestine
D. Lower small intestine
E. Colon
1. A 32-year-old primiparous woman suffers a massive antepartum haemorrhage and
undergoes an emergency caesarean section. During the caesarean section she suffers a
massive obstetric haemorrhage requiring transfusion of blood and clotting factors. As a result
of these events, she develops disseminated intravascular coagulopathy.
Which of the following laboratory results would be expected in this condition?
A Decreased activated partial thromboplastin time
B Increased factor VII levels
C Increased fibrinogen
D Increased soluble fibrin
E Thrombocytopenia

Answer E Thrombocytopaenia
Disseminated intravascular coagulopathy (DIC) is a condition, where there is generalised and
widespread pathological activation of the clotting system. There is clotting within the
microvasculature which causes consumption of coagulation products. Pathological changes
include inflammatory activation, suppression of anticoagulation and inhibition of fibrinolysis.
The obstruction of the microvascular vessels can cause disruption of blood flow to major
organs, potentially leading to multiorgan failure.
DIC usually occurs as a result of activation of the intrinsic coagulation pathway. There are
usually increased levels of soluble fibrin. Diagnosis is by blood tests which usually reveal:
• Thrombocytopaenia
• Prolonged activated partial thromboplastin time
• Low fibrinogen
• Increased fibrinogen degradation products
Treatment is via reversal of the underlying cause. Fresh frozen plasma may be given as well as
platelets depending on the degree of thrombocytopaenia.

2. A 72-year-old woman undergoes a total abdominal hysterectomy. She has chronic


obstructive pulmonary disease. Postoperatively, she is difficult to extubate and has a prolonged
stay on the intensive care unit.
Which of the following is the most important direct stimulus to respiration?
A Decreased arterial pH
B Decreased arterial pO2
C Decreased arterial pCO2
D Increased H+ concentration of the cerebrospinal fluid (CSF)
E Increased pCO2 of the CSF

Answer D Increased H+ concentration of the cerebrospinal fluid


The main stimulus to the respiratory centre comes from the chemoreceptors. These are central
and peripheral. The central chemoreceptors are found on the surface of the upper medulla.
Peripheral chemoreceptors are around the aortic arch, innervated by the vagus nerve, and in
the carotid body, innervated by the glossopharyngeal nerve. The central chemoreceptors are
only sensitive to changes in the pH. The carotid body receptors are sensitive to changes in
pO2. Both the carod body and aortic arch receptors are sensitive to changes in pCO2 and pH.
Variations in CO2 are altered via a change in ventilation. A rise in CO2 leads to an increase in
ventilation and hypoxia increases the respiratory centre sensitivity to CO2. The response to
hypoxia is less marked than the response to CO2. The response to respiration as a result of
acidosis is reduced because of the production of deoxygenated haemoglobin which acts as a
buffer.

3. Normal clotting of blood requires


A. Inactivation of heparin
B. Inactivation of plasmin (fibrinolysin)
C. Calcium ions
D. An adequate intake of vitamin D
E. An adequate intake of vitamin C

Answer C

4.Which of the following substances occur in lower


concentration in foetal blood in comparison to the
maternal blood?
A. Amino acids
B. Ca2+
C. CO2
D. Glucose
E. Urea

Answer D

5. Identify the blood cell

Basophil

6. Which of the following is true about the smooth muscle cells?


A. Controlled by the autonomic nervous system
B. Do not contain actin and myosin
C. Mitochondria are absent
D. Present a striated appearance
E. Do not cause spontaneous muscle contraction

Answer A

7. Which of the following regions of nephron does the water leave via diffusion?
A. Descending thin limb
B. Thick ascending limb
C. Proximal convoluted tubule
D. Distal convoluted tubule
E. Bowman’s capsule

Answer A

8. Sympathetic drive to the heart is increased in which of the following conditions?


A. In exercise
B. In excitement
C. In hypotension
D. None of the above
E. All of the above

Answer E
9. Which of the following occurs during the isometric ventricular contraction?
A. The entry and exit valves of the ventricle are closed
B. Pressure in the aorta rises
C. Pressure in the atria falls
D. Left coronary blood flow rises
E. The rate of rise in pressure is greater in the right than in the left ventricle

Answer A

10. What is the main site of iron absorption?


A. Stomach
B. Rectum
C. Upper small intestine
D. Lower small intestine
E. Colon

Answer C
1. Which of the following is true regarding colour blindness?
A. Results from inability to detect one of the three primary light colours, red, yellow and blue
B. Where red and green are indistinguishable is due to failure of red and green cone systems
C. In which no colours can be detected is due to failure of all the cones systems
D. Is more common in women than men
E. Is a disability linked to the Y-chromosome

2. The hair cells in the semi-circular canals are stimulated by which of the following?
A. Movement of perilymph
B. Linear acceleration
C. Rotation at constant velocity
D. Gravity
E. Movement of endolymph relative to hair cells

3. intracranial pressure does not rise in which of the following situation?


A. Cerebral venous pressure rises
B. Forced expiration is made against a closed glottis
C. There is a bout of coughing
D. Cerebral blood flow increases
E. Arterial PCO2 falls below normal
4. Severe diarrhoea causes a decrease in all of the following except?
A. Body potassium
B. Body sodium
C. Extracellular fluid volume
D. Total peripheral resistance
E. Blood pH

5. The risk of developing gallstones increases in which of the following situations?


A. When cholesterol micelles are formed in the gall bladder
B. As the bile salt: cholesterol ratio increases
C. As the lecithin: cholesterol ratio increases
D. When supplementary bile salts are taken by mouth
E. In patients with haemolytic anaemia

6. Women having their first child after the age of 35 are associated with an increased risk of
which of the following?
A. Average blood loss than younger women
B. Incidence of ineffective uterine contractions during labour
C. Incidence of foetal abnormalities
D. Risk of spontaneous abortion
E. All of the above
7. Which of the following drugs causes hypokalaemia?
A. Amiloride
B. Carbenoxolone
C. Digoxin
D. Spironolactone
E. All of the above

8.in a normal adult woman weighing 75 kg, which of the following is not true regarding the
extracellular fluid (ECF)?
A. Contains plasma protein
B. Forms a lesser proportion of the total body weight in an obese than in a lean woman
C. Has a sodium concentration of 135–145 mmol/L
D. Has a total volume of 12–15 litres
E. Is isotonic throughout the body

9. gastric juice deficiency is likely to result in which of the following abnormality?


A. Pale stools
B. Local infections
C. Steatorrhoea
D. Milk intolerance
E. Anaemia
10. Which of the following is true regarding the ventricular filling?
A. Depends mainly on atrial contraction
B. Begins during isometric ventricular relaxation
C. Gives rise to a third heart sound in some healthy people
D. Can occur only when atrial pressure is greater than atmospheric pressure
E. Is most rapid in the second half of diastole
1. Which of the following is true regarding colour blindness?
A. Results from inability to detect one of the three primary light colours, red, yellow and blue
B. Where red and green are indistinguishable is due to failure of red and green cone systems
C. In which no colours can be detected is due to failure of all the cones systems
D. Is more common in women than men
E. Is a disability linked to the Y-chromosome

Answer A

2. The hair cells in the semi-circular canals are stimulated by which of the following?
A. Movement of perilymph
B. Linear acceleration
C. Rotation at constant velocity
D. Gravity
E. Movement of endolymph relative to hair cells

Answer A

3. intracranial pressure does not rise in which of the following situation?


A. Cerebral venous pressure rises
B. Forced expiration is made against a closed glottis
C. There is a bout of coughing
D. Cerebral blood flow increases
E. Arterial PCO2 falls below normal

Answer E

4. Severe diarrhoea causes a decrease in all of the following except?


A. Body potassium
B. Body sodium
C. Extracellular fluid volume
D. Total peripheral resistance
E. Blood pH

Answer D

5. The risk of developing gallstones increases in which of the following situations?


A. When cholesterol micelles are formed in the gall bladder
B. As the bile salt: cholesterol ratio increases
C. As the lecithin: cholesterol ratio increases
D. When supplementary bile salts are taken by mouth
E. In patients with haemolytic anaemia

Answer E
6.Women having their first child after the age of 35 are associated with an increased risk of
which of the following?
A. Average blood loss than younger women
B. Incidence of ineffective uterine contractions during labour
C. Incidence of foetal abnormalities
D. Risk of spontaneous abortion
E. All of the above

Answer E

7. Which of the following drugs causes hypokalaemia?


A. Amiloride
B. Carbenoxolone
C. Digoxin
D. Spironolactone
E. All of the above

Answer B

8. in a normal adult woman weighing 75 kg, which of the following is not true regarding the
extracellular fluid (ECF)?
A. Contains plasma protein
B. Forms a lesser proportion of the total body weight in an obese than in a lean woman
C. Has a sodium concentration of 135–145 mmol/L
D. Has a total volume of 12–15 litres
E. Is isotonic throughout the body
Answer E
9. gastric juice deficiency is likely to result in which of the following abnormality?
A. Pale stools
B. Local infections
C. Steatorrhoea
D. Milk intolerance
E. Anaemia
Answer E
10. Which of the following is true regarding the ventricular filling?
A. Depends mainly on atrial contraction
B. Begins during isometric ventricular relaxation
C. Gives rise to a third heart sound in some healthy people
D. Can occur only when atrial pressure is greater than atmospheric pressure
E. Is most rapid in the second half of diastole
Answer C
1.A 23-year-old woman develops a fever and has offensive vaginal discharge and abdominal
pain. You are concerned that she is septic and wish to administer intravenous antibiotics. Prior
to administration you wish to calculate her estimated glomerular filtration rate (eGFR) in order
to dose her antibiotic therapy appropriately.
Which of the following factors is included when calculating eGFR?
A Creatinine
B Diabetic status
C Height
D Medication
E Weight

2.A 67-year-old woman is admitted to hospital with frequency of urination and extreme thirst.
Her blood tests reveal deranged urea and electrolytes. Provisional diagnosis is diabetes
insipidus. Which hormone acts in the nephron, to increase the permeability of the collecting
ducts to water?
A Aldosterone
B Angiotensin
C Atrial natriuretic peptide
D Parathyroid hormone
E Vasopressin

3. Which of the following substances is unable to bind with fetal haemoglobin?


A 2,3-diphosphoglycerate
B Carbon dioxide
C Carbon monoxide
D Nitrous oxide
E Oxygen

4. A patient presents with amenorrhea and galactorrhea. Her PRL levels are elevated. She is
not and has never been pregnant. In addition to evaluating her for a prolactinoma, one also
needs to evaluate for other causes that would increase PRL such as elevated level of which of
the following?
(A) corticotropin-releasing hormone (CRH)
(B) dopamine
(C) gamma-aminobutyric acid (GABA)
(D) histamine type II receptor activation
(E) thyrotropin-releasing hormone (TRH)

5. The development of the oral contraceptive pill was a milestone for reproductive gynecology.
Addition of an ethinyl group at the 17C position of estradiol was critical in the development of
the oral contraceptive pill because it does which of the following?
(A) decreases biological activity
(B) increases androgenic activity
(C) increases hepatic degradation
(D) increases sex hormone-binding globulin (SHBG) affinity
(E) maintains biological activity after oral absorption

6. Which of the following statements best describes the role of FSH in menstruation?
(A) FSH increases its own receptor numbers on theca cells.
(B) FSH induces granulosa cell LH receptors within the dominant follicle.
(C) FSH induces theca cell aromatase.
(D) FSH stimulates follicular growth only in the early preantral stage.
(E) FSH stimulates granulosa cell androgen production.

7. Which of the following is mediated by progesterone thus implying successful ovulation?


(A) mammary ductal development
(B) proliferative endometrium
(C) shortened GnRH pulse interval
(D) thermogenic effect
(E) thin, stretchy cervical mucus

8.A 70-year-old man is operated on for aneurysm of aorta. Severe bleeding requires infusion of
forty units of blood. His recovery is complicated by a bleeding tendency and he is found to have
a very low level of fibrinogen. His treatment includes administration of heparin. What is the most
likely abnormality to be revealed by the laboratory investigations in this case?
A. Deficiency of vitamin K
B. Deficiency of factor VIII
C. Increased fibrinogen level
D. Disseminated intravascular coagulation
E. Deficiency of prothrombin

9. Which of the following statement is true regarding blood eosinophils?


A. Have agranular cytoplasm
B. Are about a quarter of all leucocytes
C. Are relatively scarce in the mucosa of the respiratory, urinary and alimentary tracts
D. Release cytokines
E. Increase in number in viral infections

10. Bronchial smooth muscle contracts in response to


A. Bronchial mucosal irritation
B. Local beta-adrenoceptor stimulation
C. Circulating noradrenaline
D. All of the above
E. None of the above
1. A 23-year-old woman develops a fever and has offensive vaginal discharge and abdominal
pain. You are concerned that she is septic and wish to administer intravenous antibiotics. Prior
to administration you wish to calculate her estimated glomerular filtration rate (eGFR) in order
to dose her antibiotic therapy appropriately.
Which of the following factors is included when calculating eGFR?
A Creatinine
B Diabetic status
C Height
D Medication
E Weight

A Creatinine
Estimated glomerular filtration rate is used as a marker of renal function and used in clinical
practice, e.g. for calculations of renal function in the use of nephrotoxic drugs. It is often
calculated using the modification of diet in renal disease (MDRD) equation which takes into
account the age, creatinine levels, gender and ethnic group. Weight, height, comorbidities and
medication are not used in this formula. It must be remembered that estimated glomerular
filtration rate (eGFR) is still an estimate and may be inaccurate in certain circumstances, e.g.
malnourished patients and amputees. It should not be used in children or pregnant women. For
Afro-Caribbean patients, the eGFR may be 21% higher than estimated. Normal GFR is > 90
mL/min/1.73 m2.
2. A 67-year-old woman is admitted to hospital with frequency of urination and extreme thirst.
Her blood tests reveal deranged urea and electrolytes. Provisional diagnosis is diabetes
insipidus. Which hormone acts in the nephron, to increase the permeability of the collecting
ducts to water?
A Aldosterone
B Angiotensin
C Atrial natriuretic peptide
D Parathyroid hormone
E Vasopressin

Answer E Vasopressin
Nephrons are the functional unit of the kidney and are responsible for the filtration of blood,
excretion of waste products and the regulation of water balance. The collecting duct is the
terminal part of the nephron and receives filtrate that has already passed through the
glomerulus (the main functional filtration unit), the loop of Henle and the proximal and distal
convoluted tubules. By the time the filtrate reaches the collecting duct it has already undergone
a process of reabsorption of substances such as sodium chloride, water, bicarbonate,
potassium and calcium. At the collecting duct vasopressin, also known as antidiuretic hormone,
increases permeability to water allowing for water molecules to be reabsorbed. Vasopressin
enables the production of concentrated urine by acting on the aquaporins (water channels) of
the collecting duct, allowing them to facilitate the passive reabsorption of water (Figure 7.2).
3. Which of the following substances is unable to bind with fetal haemoglobin?
A 2,3-diphosphoglycerate
B Carbon dioxide
C Carbon monoxide
D Nitrous oxide
E Oxygen

Answer A 2, 3-diphosphoglycerate
The oxygen dissociation curve for fetal haemoglobin (HbF) is a sigmoid shape. HbF consists of
two α chains and two γ chains. HbF has a higher affinity for oxygen and there is therefore a left
shift in the fetal oxygen dissociation curve. This difference in binding capacity between HbA
The reason for this shift is the reduced
binding of 2,3-diphosphoglyceric acid
(2,3-DPG). 2,3-DPG has a higher affinity for
the β chains in the adult haemoglobin (HbA).
and HbF ensures that HbF has a greater
affinity for oxygen than HbA. The P50 is
defined as the partial pressure of oxygen at
which the oxygen-carrying protein is 50%
saturated and this is lower in HbF due to the
reduced sensitivity of 2,3-DPG (Figure 7.4).
4. A patient presents with amenorrhea and galactorrhea. Her PRL levels are elevated. She is
not and has never been pregnant. In addition to evaluating her for a prolactinoma, one also
needs to evaluate for other causes that would increase PRL such as elevated level of which of
the following?
(A) corticotropin-releasing hormone (CRH)
(B) dopamine
(C) gamma-aminobutyric acid (GABA)
(D) histamine type II receptor activation
(E) thyrotropin-releasing hormone (TRH)

Asnwer (E) PRL secretion is dominated by the inhibitory action of hypothalamic dopamine.
Other mechanisms of PRL inhibition are GABA and histamine type II receptor activation. TRH is
a potent stimulator of PRL and may induce galactorrhea under conditions of primary
hypothyroidism, which should be checked for by measuring TSH.

5. The development of the oral contraceptive pill was a milestone for reproductive gynecology.
Addition of an ethinyl group at the 17C position of estradiol was critical in the development of
the oral contraceptive pill because it does which of the following?
(A) decreases biological activity
(B) increases androgenic activity
(C) increases hepatic degradation
(D) increases sex hormone-binding globulin (SHBG) affinity
(E) maintains biological activity after oral absorption

Answer (E) The major breakthrough in steroid contraception occurred in 1938, when it was
discovered that addition of an ethinyl group at the 17C position made estradiol active because of
its reduced rate of hepatic degradation after oral ingestion. This stability with oral intake was
necessary for hormonal contraception to be readily available to large numbers of women.

6. Which of the following statements best describes the role of FSH in menstruation?
(A) FSH increases its own receptor numbers on theca cells.
(B) FSH induces granulosa cell LH receptors within the dominant follicle.
(C) FSH induces theca cell aromatase.
(D) FSH stimulates follicular growth only in the early preantral stage.
(E) FSH stimulates granulosa cell androgen production.

Answer (B) FSH regulates ovarian steroidogenesis and stimulates folliculogenesis beyond the
early preantral stage. The central principle of gonadotropin-dependent ovarian steroidogenesis is
the “two-cell theory” of estrogen synthesis. This theory proposes that a developing follicle
requires both granulosa cells and surrounding stromal cells (theca interna) for estrogen synthesis.
Granulosa cells possess FSH receptors and respond to FSH by synthesizing aromatase and
increasing FSH receptor sites. Thecal cells contain LH receptors and produce androgens in the
presence of LH. Under these conditions, granulosa cells are able to convert androgens, produced
locally by thecal cells, to estrogens and are the major source of circulating E2.
7. Which of the following is mediated by progesterone thus implying successful ovulation?
(A) mammary ductal development
(B) proliferative endometrium
(C) shortened GnRH pulse interval
(D) thermogenic effect
(E) thin, stretchy cervical mucus
Answer (D) Progesterone produced during the luteal phase inhibits GnRH pulsatility and shifts
core body temperature upward from 0.6 to 1.0°F. Progesterone also produces secretory
endometrium; induces thick, opaque cervical mucus (preventing passage of sperm and
bacteria); and stimulates mammary alveolar development. Estrogen causes endometrial
proliferation and mammary ductal development.

8.A 70-year-old man is operated on for aneurysm of aorta. Severe bleeding requires infusion of
forty units of blood. His recovery is complicated by a bleeding tendency and he is found to have
a very low level of fibrinogen. His treatment includes administration of heparin. What is the most
likely abnormality to be revealed by the laboratory investigations in this case?
A.Deficiency of vitamin K
B. Deficiency of factor VIII
C. Increased fibrinogen level
D. Disseminated intravascular coagulation
E.Deficiency of prothrombin
Answer D
9. Which of the following statement is true regarding blood eosinophils?
A. Have agranular cytoplasm
B. Are about a quarter of all leucocytes
C. Are relatively scarce in the mucosa of the respiratory, urinary and alimentary tracts
D. Release cytokines
E. Increase in number in viral infections

Answer D

10. Bronchial smooth muscle contracts in response to


A. Bronchial mucosal irritation
B. Local beta-adrenoceptor stimulation
C. Circulating noradrenaline
D. All of the above
E. None of the above

Answer A
1. Which of the following disorders is correctly matched to an appropriate diagnostic test?
A) Addison's ‐ Fludrocortisone suppression test
B) Addison's ‐ Dexamethasone suppression test
C) Cushing's ‐ Synacthen® test
D) Conn's ‐ Saline (salt) suppression test
E) Conn's ‐ 24 hour urinary cortisol

2. Which major hormone of pregnancy is produced by the placenta from 16‐


hydroxydehydroepiandrosterone sulfate (16‐OH DHEAS)?
A) Estrone
B) Estriol
C) Pregnenolone
D) Dehydroepiandrosterone
E) Estradiol

3. Polycystic ovarian syndrome is classed as?


A) WHO type I Ovulation Disorders
B) WHO type II Ovulation Disorders
C) WHO type III Ovulation Disorders
D) WHO type IV Ovulation Disorders
E) WHO type V Ovulation Disorders
4. You see a 42 year old women in clinic who mentions she has been sweating and has
frequent
headaches. On examination you note her blood pressure is 195/105 and pulse rate is 110. You
suspect pheochromocytoma. Which of the following conditions is NOT associated with higher
risk of pheochromocytoma?
A) multiple endocrine neoplasia type 1
B) multiple endocrine neoplasia type 2
C) paraganglioma syndromes type 3
D) von Hippel‐Lindau (VHL) disease
E) neurofibromatosis type 1

5. Which of the following is true of nephrogenic diabetes insipidus?


A) There is deficiency of arginine vasopressin (AVP)
B) There is deficiency of anti‐diuretic hormone (ADH)
C) There is insensitivity to Vasopressin
D) There is deficiency of Insulin
E) There is insensitivity to Insulin

6. In girls what is the first sign of puberty?


A) Menarche
B) Peak growth velocity
C) Breast development
D) Axillary hair
E) Acne

7. Human placental lactogen (hPL) is structurally similar to which of the following hormones
A) Oxytocin
B) Estrone
C) Thyroid Stimulating Hormone (TSH)
D) Estradiol
E) Prolactin

8. Which of the following is responsible for the formation of Angiotensin I from Angiotensinogen
A) ACE
B) Aldosterone
C) Pepsin
D) Renin
E) Vasopressin

9. The zona glomerulosa produces which of the following hormones?


A) Cortisol
B) Calcitonin
C) Somatostatin
D) Dopamine
E) Aldosterone

10. The ovaries produce androgen and progesterone. What is the common precursor for both of
these hormones?
A) Cholesterol
B) Gylcerol
C) Inhibin A
D) Pyruvate
E) Sorbitol
1. Which of the following disorders is correctly matched to an appropriate diagnostic test?
A) Addison's ‐ Fludrocortisone suppression test
B) Addison's ‐ Dexamethasone suppression test
C) Cushing's ‐ Synacthen® test
D) Conn's ‐ Saline (salt) suppression test
E) Conn's ‐ 24 hour urinary cortisol
Correct Answer: Conn's ‐ Saline (salt) suppression test
Explanation: Addison's
Chronic Primary Adrenal Insufficency
Hormone abnormality: Cortisol & Aldosterone Deficiency
Key Clinical Features: Hypotension, hyponatraemia & hyperkalaemia
Testing: 9am Cortisol & Synacthen test
Cushing's
Hormone abnormality: Cortisol Excess
Key Clinical Features: Hypertension, weight gain & diabetes
Testing: Dexamethasone suppression test or 24‐hour urinary cortisol
Conn's
Hormone abnormality: Aldosterone Excess
Key Clinical Features: Hypertension & hypokalaemia
Testing: Renin/aldosterone ratio, saline (salt) suppression or fludrocortisone
suppression test
2. Which major hormone of pregnancy is produced by the placenta from 16‐
hydroxydehydroepiandrosterone sulfate (16‐OH DHEAS)?
A) Estrone
B) Estriol
C) Pregnenolone
D) Dehydroepiandrosterone
E) Estradiol
Correct Answer: Estriol
The placenta produces Estriol from 16‐OH DHEAS. Estriol is the major oestrogen (estrogen) of
pregnancy and the placenta is the primary site of production.
Pregnenolone is synthesised by the placenta from cholesterol and this is converted to dehydro
epiandrosterone (DHEA) in the fetal adrenal gland

3. Polycystic ovarian syndrome is classed as?


A) WHO type I Ovulation Disorders
B) WHO type II Ovulation Disorders
C) WHO type III Ovulation Disorders
D) WHO type IV Ovulation Disorders
E) WHO type V Ovulation Disorders
Correct Answer: WHO type II Ovulation Disorders
4. You see a 42 year old women in clinic who mentions she has been sweating and has
frequent headaches. On examination you note her blood pressure is 195/105 and pulse rate is
110. You suspect pheochromocytoma. Which of the following conditions is NOT associated with
higher risk of pheochromocytoma?
A) multiple endocrine neoplasia type 1
B) multiple endocrine neoplasia type 2
C) paraganglioma syndromes type 3
D) von Hippel‐Lindau (VHL) disease
E) neurofibromatosis type 1
Correct Answer: multiple endocrine neoplasia type 1
The conditions associated with increased risk of pheochromocytoma are:
MEN type 2
Paraganglioma syndromes types 1,3 and 4
VHL
Neurofibromatosis type 1

5. Which of the following is true of nephrogenic diabetes insipidus?


A) There is deficiency of arginine vasopressin (AVP)
B) There is deficiency of anti‐diuretic hormone (ADH)
C) There is insensitivity to Vasopressin
D) There is deficiency of Insulin
E) There is insensitivity to Insulin
Correct Answer: There is insensitivity to Vasopressin
Diabetes Insipidus is due to a deficiency of AVP (also called ADH or simply Vasopressin). This
deficiency is either a real deficiency due to lack of production of ADH in the hypothalamus/poste
rior pituitary when it is termed central, cranial or neurological diabetes insipidus.
In nephrogenic diabetes insipidous there are either normal or raised levels of ADH however the
ADH receptors in the kidney are dysfunctional and therefore the ADH produced is ineffective.
As a result of deficient ADH there is diuresis.

6. In girls what is the first sign of puberty?


A) Menarche
B) Peak growth velocity
C) Breast development
D) Axillary hair
E) Acne
Correct Answer: Breast development
Puberty is often classed into 5 stages using the Tanner classification.
Breast development typically begins between 10 and 11. The breast bud hs typically developed
by a mean age of 11.2 years and is considered stage 2.
Peak growth velocity, axillary hair growth and acne are Stage 3 (mean age 12.4 to 13.2)
Menarche is stage 4 (mean age 13.3)
7. Human placental lactogen (hPL) is structurally similar to which of the following hormones
A) Oxytocin
B) Estrone
C) Thyroid Stimulating Hormone (TSH)
D) Estradiol
E) Prolactin
Correct Answer: Prolactin
Although hormone similarities may not seem clinically relevant it is something the
RCOG can ask about in the part 1. This question may have used GH or Prolactin in place of hP
L as all 3 are structurally similar.

8. Which of the following is responsible for the formation of Angiotensin I from Angiotensinogen
A) ACE
B) Aldosterone
C) Pepsin
D) Renin
E) Vasopressin
Correct Answer: Renin
Renin cleaves the peptide bond on Angiotensinogen forming Angiotensin I
9. The zona glomerulosa produces which of the following hormones?
A) Cortisol
B) Calcitonin
C) Somatostatin
D) Dopamine
E) Aldosterone
Correct Answer: Aldosterone

10. The ovaries produce androgen and progesterone. What is the common precursor for both of
these hormones?
A) Cholesterol
B) Gylcerol
C) Inhibin A
D) Pyruvate
E) Sorbitol
Correct Answer: Cholesterol
1. In the non‐pregnant state which of the following hormones is secreted by the corpus luteum?
A) Estrone
B) Ethinylestradiol
C) Testosterone
D) Progesterone
E) LH

2. Cortisol is produced where?


A) Parafollicular cells
B) Zona glomerulosa
C) Chromaffin cells
D) Zona fasciculata
E) Anterior pituitary

3. A 15 year old girl is being investigated for primary amenorrhoea. She has normal FSH,LH
and E2 levels on hormone profiling and normal secondary sexual characteristics. An ultrasound
shows no uterus. What is the likely diagnosis?
A) Complete Androgen Insensitivity Syndrome (CAIS)
B) Testicular Feminisation Syndrome
C) Rokitansky‐Kuster‐Hauser syndrome
D) Turner Syndrome
E) Swyer Syndrome
4. At ovulation the surge in LH causes rupture of the mature oocyte via action on what?
A) Theca interna
B) Theca externa
C) Granulosa interna
D) Granulosa externa
E) Large luteal cells

5. Human Chorionic Gonadotrophin (HCG) is structurally similar to which of the following


hormones?
A) Thyroid Stimulating Hormone (TSH)
B) Growth Hormone (GH)
C) Prolactin
D) Oxytocin
E) Oestradiol

6. Which of the following is NOT a recognised cause of hyperprolactinaemia?


A) H2 antagonists
B) Hyperthyroidism
C) Chest wall surgery
D) PCOS
E) Pregnancy
7. Glucagon is stimulated by which of the following?
A) Increased Keto‐acids
B) Cholecystokinin
C) Somatostatin
D) Raised Urea
E) Increased free fatty acids

8. Prolactin is structurally similar to which of the following hormones?


A) Oestradiol
B) Follicle Stimulating Hormone (FSH)
C) Human Chorionic Gonadotrophin (HCG)
D) Growth Hormone (GH)
E) Anti‐diuretic Hormone (ADH)

9. Which of the following is typical of Conn's Syndrome?


A) hypocalcaemia, hypokalaemia & hypotension
B) hypocalcaemia, hyperkalaemia & hypotension
C) hypercalcaemia, hyperkalaemia & hypotension
D) hypocalcaemia, hyperkalaemia & hypertension
E) hypocalcaemia, hypokalaemia & hypertension
10. A patient develops hypocalcaemia as a result of pancreatitis. What is the appropriate
homeostatic response to hypocalcaemia?
A) Increased PTH, Increased 1,25 dihydroxycholecalciferol & Increased phosphate
B) Increased PTH, Increased 1,25 dihydroxycholecalciferol & Decreased phosphate
C) Increased PTH, Decreased 1,25 dihydroxycholecalciferol & Decreased phosphate
D) Decreased PTH, Decreased 1,25 dihydroxycholecalciferol & Increased phosphate
E) Decreased PTH, Increased 1,25 dihydroxycholecalciferol & Increased phosphate
1. In the non‐pregnant state which of the following hormones is secreted by the corpus luteum?
A) Estrone
B) Ethinylestradiol
C) Testosterone
D) Progesterone
E) LH
Correct Answer: Progesterone

2. Cortisol is produced where?


A) Parafollicular cells
B) Zona glomerulosa
C) Chromaffin cells
D) Zona fasciculata
E) Anterior pituitary
Correct Answer: Zona fasciculate

3. A 15 year old girl is being investigated for primary amenorrhoea. She has normal FSH,LH
and E2 levels on hormone profiling and normal secondary sexual characteristics. An ultrasound
shows no uterus. What is the likely diagnosis?
A) Complete Androgen Insensitivity Syndrome (CAIS)
B) Testicular Feminisation Syndrome
C) Rokitansky‐Kuster‐Hauser syndrome
D) Turner Syndrome
E) Swyer Syndrome
Correct Answer: Rokitansky‐Kuster‐Hauser syndrome

4. At ovulation the surge in LH causes rupture of the mature oocyte via action on what?
A) Theca interna
B) Theca externa
C) Granulosa interna
D) Granulosa externa
E) Large luteal cells
Correct Answer: Theca externa

5. Human Chorionic Gonadotrophin (HCG) is structurally similar to which of the following


hormones?
A) Thyroid Stimulating Hormone (TSH)
B) Growth Hormone (GH)
C) Prolactin
D) Oxytocin
E) Oestradiol
Correct Answer:
Thyroid Stimulating Hormone (TSH)
6. Which of the following is NOT a recognised cause of hyperprolactinaemia?
A)H2 antagonists
B) Hyperthyroidism
C) Chest wall surgery
D) PCOS
E)Pregnancy
Correct Answer:
Hyperthyroidism

7. Glucagon is stimulated by which of the following?


A)Increased Keto‐acids
B) Cholecystokinin
C) Somatostatin
D) Raised Urea
E)Increased free fatty acids
Correct Answer:
Cholecystokinin

8. Prolactin is structurally similar to which of the following hormones?


A) Oestradiol
B) Follicle Stimulating Hormone (FSH)
C) Human Chorionic Gonadotrophin (HCG)
D) Growth Hormone (GH)
E) Anti‐diuretic Hormone (ADH)
Correct Answer: Growth Hormone (GH)

9. Which of the following is typical of Conn's Syndrome?


A) hypocalcaemia, hypokalaemia & hypotension
B) hypocalcaemia, hyperkalaemia & hypotension
C) hypercalcaemia, hyperkalaemia & hypotension
D) hypocalcaemia, hyperkalaemia & hypertension
E) hypocalcaemia, hypokalaemia & hypertension
Correct Answer: hypocalcaemia, hypokalaemia & hypertension

10. A patient develops hypocalcaemia as a result of pancreatitis. What is the appropriate


homeostatic response to hypocalcaemia?
A) Increased PTH, Increased 1,25 dihydroxycholecalciferol & Increased phosphate
B) Increased PTH, Increased 1,25 dihydroxycholecalciferol & Decreased phosphate
C) Increased PTH, Decreased 1,25 dihydroxycholecalciferol & Decreased phosphate
D) Decreased PTH, Decreased 1,25 dihydroxycholecalciferol & Increased phosphate
E) Decreased PTH, Increased 1,25 dihydroxycholecalciferol & Increased phosphate
Correct Answer:
Increased PTH, Increased 1,25 dihydroxycholecalciferol & Decreased phosphate
1. Which of the following hormone acts on cartilage and liver to release igF-1?
A. Growth hormone
B. Prolactin
C. Somatostatin
D. TSH
E. ACTH

2. Hyperprolactinaemia with hypogonadism is found in which of the following conditions?


A. Chromophobe adenoma of the pituitary
B. Addison’s disease
C. Hyperthyroidism
D. Sheehan’s syndrome
E. None of the above

3. Which of the following statement about testicular hormones is true?


A. Inhibin increases plasma follicle stimulating hormone levels
B. Testosterone in plasma is partly bound to albumin
C. Testosterone is excreted in urine as 17-ketosteroids
D. None of the above
E. All the above
4. A 21-year-old athletic woman with diabetes on a low-dose oral contraceptive comes to your
clinic with irregular menses and bilateral breast discharge. On examination, the discharge is
expressed and galactorrhea is confirmed with fat globules seen microscopically. She currently
takes metoclopramide (Reglan) for delayed gastric emptying. A random serum prolactin level is
65 ng/mL. Which of the following is most likely responsible for her hyperprolactinemia?
(A) metoclopramide
(B) pregnancy
(C) oral contraceptive
(D) pituitary adenoma
(E) Exercise

5. A 25-year-old woman who underwent menar-che at 11 years of age presents with a history of
irregular menstrual cycles over the last 12 months, increased weight gain, and bilateral pelvic
pain.
Transvaginal ultrasound shows large cystic ad-nexa, with cysts measuring 7 to 9 cm in size. A
urine pregnancy test is negative. Her thyroid-stimulating hormone (TSH) level is 17 mlU/mL,
and prolactin level is 10 ng/mL. Which of the following is the treatment of choice for this patient
to regain normal menstrual cycles?
(A) monophasic birth control pills
(B) triphasic birth control pills
(C) levothyroxine treatment
(D) bromocriptine treatment
(E) gonadotropin-releasing hormone (GnRH) agonist treatment

6. A 16-year-old girl has not experienced menarche. Examination shows absence of breast
development and small but otherwise normal female pelvic organs. Which of the following
diagnostic tests is most useful in determining the etiology of the amenorrhea?
(A) serum follicle-stimulating hormone (FSH)
(B) serum estradiol
(C) serum testosterone
(D) magnetic resonance imaging (MRI) of the head
(E) ovarian biopsy

7. An adult genetic male with 17-alpha-hydro-xylase deficiency would have which of the
following findings?
(A) no breast development, uterus present, hypertension
(B) no breast development, uterus present, hypotension
(C) breast development, uterus absent, hypotension
(D) no breast development, uterus absent, hypertension
(E) breast development, uterus present, hypertension

8. A 4-year-old girl is brought in by her mother for evaluation of clitoral enlargement. She is tall
for her age, with no breast or axillary hair development. There is slight pubic hair growth on
examination and an enlarged clitoris with a single perineal opening. Karyotype is 46,XX. The
17-OHP level is 108 ng/mL. What is the most likely diagnosis?
(A) androgen insensitivity syndrome
(B) PCOS
(C) CAH with 21-hydroxylase deficiency
(D) ovarian thecoma
(E) germ cell ovarian tumor

9. What is the principal androgen used for placental estrogen synthesis?


(A) androstenedione
(B) testosterone
(C) DHEAS
(D) aldosterone
(E) Cortisol

10. What is the principal hormone responsible for 1,25-dihydroxy vitamin D3 synthesis?
(A) aldosterone
(B) cortisol
(C) thyroxine
(D) parathyroid hormone (PTH)
(E) insulin
1. Which of the following hormone acts on cartilage and liver to release igF-1?
A.Growth hormone
B. Prolactin
C. Somatostatin
D. TSH
E.ACTH
Answer A

2. Hyperprolactinaemia with hypogonadism is found in which of the following conditions?


A. Chromophobe adenoma of the pituitary
B. Addison’s disease
C. Hyperthyroidism
D. Sheehan’s syndrome
E. None of the above
Answer A

3. Which of the following statement about testicular hormones is true?


A. Inhibin increases plasma follicle stimulating hormone levels
B. Testosterone in plasma is partly bound to albumin
C. Testosterone is excreted in urine as 17-ketosteroids
D. None of the above
E. All the above
Answer B

4. A 21-year-old athletic woman with diabetes on a low-dose oral contraceptive comes to your
clinic with irregular menses and bilateral breast discharge. On examination, the discharge is
expressed and galactorrhea is confirmed with fat globules seen microscopically. She currently
takes metoclopramide (Reglan) for delayed gastric emptying. A random serum prolactin level is
65 ng/mL. Which of the following is most likely responsible for her hyperprolactinemia?
(A) metoclopramide
(B) pregnancy
(C) oral contraceptive
(D) pituitary adenoma
(E) Exercise
Answer (A) Pregnancy increases prolactin levels; however, this patient is unlikely to be
pregnant because she was taking the oral contraceptive but a urine pregnancy test is still
indicated. The oral contraceptive rarely increases prolactin to such high levels. Metoclopramide
is a potent dopamine antagonist that can act on the lac-totroph to ramp up secretion of
prolactin. A prolactin-producing pituitary adenoma is unlikely since most tumors present with
serum prolactin levels greater than 100 ng/mL. While athletic nipple stimulation (”jogger’s
nipples”) could cause a modest elevation in serum prolactin, elevations of the level given here
are unlikely.
5. A 25-year-old woman who underwent menar-che at 11 years of age presents with a history of
irregular menstrual cycles over the last 12 months, increased weight gain, and bilateral pelvic
pain.
Transvaginal ultrasound shows large cystic ad-nexa, with cysts measuring 7 to 9 cm in size. A
urine pregnancy test is negative. Her thyroid-stimulating hormone (TSH) level is 17 mlU/mL,
and prolactin level is 10 ng/mL. Which of the following is the treatment of choice for this patient
to regain normal menstrual cycles?
(A) monophasic birth control pills
(B) triphasic birth control pills
(C) levothyroxine treatment
(D) bromocriptine treatment
(E) gonadotropin-releasing hormone (GnRH) agonist treatment
Answer (C) Ovarian cysts and irregular menstrual cycles can both arise from compensated
hypothyroidism, which is revealed by this patient’s elevated TSH level. Levothyroxine is the
treatment of choice.
Monophasic or triphasic birth control pills would only mask the underlying problem despite the
fact that regular withdrawal bleeding can be achieved. This patient has a normal serum
prolactin level, and thus bromocriptine is not indicated.

6. A 16-year-old girl has not experienced menarche. Examination shows absence of breast
development and small but otherwise normal female pelvic organs. Which of the following
diagnostic tests is most useful in determining the etiology of the amenorrhea?
(A) serum follicle-stimulating hormone (FSH)
(B) serum estradiol
(C) serum testosterone
(D) magnetic resonance imaging (MRI) of the head
(E) ovarian biopsy
Answer (A) Primary amenorrhea is defined as absence of menarche by age 16 years, or by age
14 years without appearance of secondary sex characteristics, or within 2 years of the onset of
secondary sex changes. A practical approach used by some investigators assigns patients with
normal female external genitalia and primary amenorrhea into one of four groups based on
physical examination. A series of diagnostic steps unique to each category determines the
etiology of amenorrhea. The four groups of patients with primary amenorrhea are (1) no breast
development and uterus present, (2) breast development and uterus absent, (3) no breast
development and uterus absent, and (4) breast development and uterus present. In the
evaluation of a patient from group 1 (as described in this problem), a serum FSH level
can distinguish between absence of gonadal function (elevated FSH) such as gonadal
dysgenesis and diminished pulsatile GnRH release (suppressed FSH) such as delayed puberty
from hypothalamic suppression (Olympic-level gymnastic). Further investigation is similar to
that of delayed puberty. All individuals with secondary amenorrhea are included in group 4.

7. An adult genetic male with 17-alpha-hydro-xylase deficiency would have which of the
following findings?
(A) no breast development, uterus present, hypertension
(B) no breast development, uterus present, hypotension
(C) breast development, uterus absent, hypotension
(D) no breast development, uterus absent, hypertension
(E) breast development, uterus present, hypertension
Answer (D) Genetic males (46,XY) with enzymatic defects in the early pathways of
steroidogenesis (20,22 desmolase 17-alpha-hydroxylase, 17,20 desmolase) are extremely rare
(group 3 from answer 6).
Absence of sex steroid production accounts for elevated gonadotropins, low-circulating
testosterone levels, and lack of breast development. Müllerian duct regression due to testicular
MIF causes absence of uterine development. Neonates with these disorders often die from
cortisol deficiency. Adults with 17-alpha-hydroxylase deficiency have hypertension and
hypokalemic alkalosis and should undergo gonadectomy to prevent the risk of developing a
malignant gonadal tumor.

8. A 4-year-old girl is brought in by her mother for evaluation of clitoral enlargement. She is tall
for her age, with no breast or axillary hair development. There is slight pubic hair growth on
examination and an enlarged clitoris with a single perineal opening. Karyotype is 46,XX. The
17-OHP level is 108 ng/mL. What is the most likely diagnosis?
(A) androgen insensitivity syndrome
(B) PCOS
(C) CAH with 21-hydroxylase deficiency
(D) ovarian thecoma
(E) germ cell ovarian tumor
Answer (C) CAH accounts for the majority of cases of congenital sexual ambiguity with
enlarged clitorides.
These patients have normal 46,XX karyotypes and elevated adrenal androgens, which are
responsible for the external genitalia changes. Androgen insensitivity syndrome (AIS) is
associated with the lack of response to androgens and would present as female pheno-type
with male genotype.

9. What is the principal androgen used for placental estrogen synthesis?


(A) androstenedione
(B) testosterone
(C) DHEAS
(D) aldosterone
(E) Cortisol
Answer (C) DHEAS, derived primarily from the fetus but also from the mother, is the major
androgenic precursor for placental estrogen synthesis.

10. What is the principal hormone responsible for 1,25-dihydroxy vitamin D3 synthesis?
(A) aldosterone
(B) cortisol
(C) thyroxine
(D) parathyroid hormone (PTH)
(E) Insulin
Answer (D) PTH regulates calcium metabolism by stimulating calcium reabsorption from bone,
renal tubular calcium reabsorption, and 1-alpha-hydroxylation of 25-hydroxy vitamin D3. The
resulting 1,25- dihydroxy vitamin D3 is a potent stimulator of intestinal calcium absorption. An
increase in maternal PTH production during pregnancy creates a reservoir of free calcium ions,
which are actively transported across the placenta for calcification of the fetal skeleton. The
ability of the decidua to synthesize 1,25-dihydroxy vitamin D3 aids in this process.
StudyMRCOG
MRCOGPart 1
Biochemistry
Dr Chitra
1.
2.
3.
4.
5.
6.
7.

8.
9.
10.
1. Co-factor for the free radical scavenging enzyme glutathione peroxidase is-
A. Zinc
B. Iron
C. Selenium
D. Molybdenum
E. Copper.

2. The mineral which has synergistic action with vitamin E is-


A. Calcium
B. Magnesium
C. Manganese
D. Sulphur
E. Selenium.

3. Poor wound healing, impaired spermatogenesis, diarrhoea, dermatitis, alopecia are the
features of which mineral deficiency-
A. Iron
B. Copper
C. Zinc
D. Magnesium
E. Chromium.
4. Which of following vitamin act as antioxidant and its deficiency cause mild hemolytic anemia
in newborn
a) A
b) D
c) E
d) K
e) B1

5. Which of following vitamin act as antioxidant and its deficiency cause mild hemolytic anemia
in newborn
a) A
b) D
c) E
d) K
e) B1

6. Deficiency of following vitamin deficiency cause paraesthesia:


a) B2
b) B3
c) Pantothenic acid
d) K
e) B1
7. Deficiency of following vitamin deficiency cause megaloblastic anemia:
a) B9
b) B12
c) Pantothenic acid
d) Both a and b
e) B1

8. Which of the following is not true regarding 1,25(OH)2 vitamin D?


A. 25-hydroxylation occurs in the kidney
B. Release is stimulated by hypophosphatemia
C. Promotes phosphate and calcium absorption from the gut
D. Attaches to cytosolic receptors
E. Concentrations are decreased in chronic renal failure

9. Which of the following is true concerning 1,25-(OH)2D3 (vitamin D)?


A. Facilitates calcium and phosphate reabsorption from bone
B. Is more active than 24,25-(OH)2 vitamin D
C. Levels are low during lactation
D. Reduces the absorption of calcium and phosphate from the gut
E. Stimulates the excretion of calcium and phosphate into renal tubules
10. Which of the following is correct regarding vitamin B12?
A. Is a fat-soluble vitamin
B. Absorption takes place throughout the small intestine
C. Is essential for the metabolism of folic acid in the human
D. Deficiency leads to microcytic anaemia
E. Deficiency is common in non-vegetarians
1. Co-factor for the free radical scavenging enzyme glutathione peroxidase is-
A. Zinc
B. Iron
C. Selenium
D. Molybdenum
E. Copper.
Answer C

2. The mineral which has synergistic action with vitamin E is-


A. Calcium
B. Magnesium
C. Manganese
D. Sulphur
E. Selenium.
Answer E

3. Poor wound healing, impaired spermatogenesis, diarrhoea, dermatitis, alopecia are the
features of which mineral deficiency-
A. Iron
B. Copper
C. Zinc
D. Magnesium
E. Chromium.
Answer C

4. Which of following vitamin act as antioxidant and its deficiency cause mild hemolytic anemia
in newborn
a)A
b) D
c) E
d) K
e)B1
Answer C

5. Which of following vitamin act as antioxidant and its deficiency cause mild hemolytic anemia
in newborn
a) A
b) D
c) E
d) K
e) B1
Answer C
6. Deficiency of following vitamin deficiency cause paraesthesia:
a) B2
b) B3
c) Pantothenic acid
d) K
e) B1
Answer C

7. Deficiency of following vitamin deficiency cause megaloblastic anemia:


a) B9
b) B12
c) Pantothenic acid
d) Both a and b
e) B1
Answer D

8. Which of the following is not true regarding 1,25(OH)2 vitamin D?


A. 25-hydroxylation occurs in the kidney
B. Release is stimulated by hypophosphatemia
C. Promotes phosphate and calcium absorption from the gut
D. Attaches to cytosolic receptors
E. Concentrations are decreased in chronic renal failure
Answer A

9. Which of the following is true concerning 1,25-(OH)2D3 (vitamin D)?


A.Facilitates calcium and phosphate reabsorption from bone
B. Is more active than 24,25-(OH)2 vitamin D
C. Levels are low during lactation
D. Reduces the absorption of calcium and phosphate from the gut
E.Stimulates the excretion of calcium and phosphate into renal tubules
Answer B

10. Which of the following is correct regarding vitamin B12?


A.Is a fat-soluble vitamin
B. Absorption takes place throughout the small intestine
C. Is essential for the metabolism of folic acid in the human
D. Deficiency leads to microcytic anaemia
E.Deficiency is common in non-vegetarians
Answer C
1. Cell membrane primarily composed of:
a) Phosphatidylcholine
b) Phosphatidylethanolamine
c) Phosphatidylinisitol
d) Phosphatidylserine
e) All of above

2. Under aerobic condition all respiratory substance end in :


a) Glycolysis
b) Glyconeogenisis
c) Citric acid cycle
d) Pentose pathway
e) Fatty acid oxidation

3. Each turn of kreb`s cycle produce ATP:


a) 12
b) 14
c) 10
d) 6
e) 1
4.In order to take part in Kreb`s cycle pyruvate must be converted into:
a) Acetyl CoA
b) Oxaloacetate
c) Citrate
d) Alpha ketoglutarate
e) Malate

5.Night blindness is caused deficiency of:


a) A
b) D
c) E
d) K
e) B1

6.Deficiency of following vitamin deficiency cause dermatitis enteritis :


a) B2
b) B3
c) Pantothenic acid
d) Biotin
e) B1
7.Glycogenolysis catalyzed by:
a) Glycogen phosphorylase
b) Glycogen synthase
c) Glycogen reductase
d) Glucose 6 phosphotase
e) Glucose 1 phosphotase

8.In prolonged fasting state there is:


a) Utilization of glucose
b) Utilization of fat
c) Mobilization of glycogen
d) Mobilization of amino acid
e) None of above

9. False statement regarding glycolysis is:


a) Net gain of reaction is 2 ATP and 2 NADH molecule
b) In aerobic condition total 8 ATP produce
c) 2 ATP are consumed in early stage of cycle
d) Cannot work in anaerobic condition
e) In anaerobic condition pyruvate is converted into lactic acid
1. Cell membrane primarily composed of:
a) Phosphatidylcholine
b) Phosphatidylethanolamine
c) Phosphatidylinisitol
d) Phosphatidylserine
e) All of above
Answer: e

2. Under aerobic condition all respiratory substance end in :


a) Glycolysis
b) Glyconeogenisis
c) Citric acid cycle
d) Pentose pathway
e) Fatty acid oxidation
Answer: c

3. Each turn of kreb`s cycle produce ATP:


a) 12
b) 14
c) 10
d) 6
e) 1
ans-10

4.In order to take part in Kreb`s cycle pyruvate must be converted into:
a)Acetyl CoA
b) Oxaloacetate
c) Citrate
d) Alpha ketoglutarate
e)Malate .
ans-a

5.Night blindness is caused deficiency of:


a)A
b) D
c) E
d) K
e)B1
Ans- a

6.Deficiency of following vitamin deficiency cause dermatitis enteritis :


a) B2
b) B3
c) Pantothenic acid
d) Biotin
e) B1
Answ- d

7.Glycogenolysis catalyzed by:


a)Glycogen phosphorylase
b) Glycogen synthase
c) Glycogen reductase
d) Glucose 6 phosphotase
e)Glucose 1 phosphotase
Ans-a

8.In prolonged fasting state there is:


a) Utilization of glucose
b) Utilization of fat
c) Mobilization of glycogen
d) Mobilization of amino acid
e) None of above
Answ-d

9. False statement regarding glycolysis is:


a) Net gain of reaction is 2 ATP and 2 NADH molecule
b)In aerobic condition total 8 ATP produce
c) 2 ATP are consumed in early stage of cycle
d) Cannot work in anaerobic condition
e)In anaerobic condition pyruvate is converted into lactic acid
Answer: d
1. Acetazolamide is a carbonic anhydrase inhibitor and is used in the treatment of all the
following conditions except which one?
(A) Dehydration
(B) Glaucoma
(C) Epilepsy
(D) Altitude sickness
(E) Congestive heart failure

2. Which of the following is true regarding carbohydrate metabolism?


A.The principal carbohydrate used in body metabolism is galactose
B. Glycolysis is the process of glycogen formation
C. The pentose shunt is active in all cells of the body except red blood cells (RBCs)
D.The tricarboxylic acid (TCA) cycle is the common pathway for the oxidation of dietary
carbohydrates, fats and proteins to CO2 and H2O
E. Acetoacetic acid and beta-hydroxybutyric acid are types of fat

3. Which of the following is an essential amino acid?


A. Arginine
B. Methionine
C. Glycine
D. Tryptophan
E. Valine
4. An iron overload is not seen in which of the following conditions?
A. Thalassaemia major
B. Polycythaemia rubra vera
C. Myelodysplasia
D. Haemochromatosis
E. None of the above

5. A 41-year-old woman presents with severe, sharp epigastric abdominal pain that radiates to
her back and with nausea and vomiting. Laboratory results indicate a serum triglyceride level
of 5000 mg/dL. She is diagnosed with pancreatitis, in part owing to her elevated serum triacyl-
glycerol levels. To form triacylglycerol from diacylglycerol, which of the following compounds is
also required?
(A) Glycerol
(B) Glycerol 3-phosphate
(C) Fatty acyl CoA
(D) Acetyl CoA
(E) Malonyl CoA

6. A 17-year-old woman who recently began taking birth control pills presents to the emergency
room with cramping abdominal pain, anxiety, paranoia, and hallucinations. A surgical
evaluation, including ultrasound and computed tomography scan, fails to demonstrate an acute
abdominal process. A urinalysis reveals an increase in urine porphyrins. Which of the fol-
lowing is the most likely?
(A) Congenital erythropoietic porphyria
(B) Variegate porphyria
(C) Porphyria cutanea tarda
(D) Acute intermittent porphyria
(E) Erythropoietic protoporphyria

7. A 73-year-old woman is admitted to the intensive care unit for septic shock from a urinary
tract infection. The critical care fellow is concerned she may not have an appropriate stress
response and orders a cosyntropin test. Which hormone does this test evaluate?
(A) Oxytocin
(B) Vasopressin
(C) Cortisol
(D) Corticotropin-releasing hormone (CRH)
(E) Adrenocorticotropic hormone (ACTH)

8. A 23-year-old woman is referred to an endocrinologist for weight gain, especially around


the waist. She also has striae over the abdomen and a rounded appearance to her face. She is
found to have Cushing disease. Which of the following would most likely be found in this
patient, compared with someone who does not have Cushing disease?
(A) Increased synthesis of immunoglobulins
(B) Increased protein synthesis
(C) Inhibition of lipolysis
(D) Increased gluconeogenesis
(E) Reduced liver glycogen stores

9. Two couples present to the emergency room with severe nausea, vomiting, and diarrhea.
One of the patients admits that she had a dinner party and served a salad containing
mushrooms she had picked during a hike in the forest earlier that day. Inhibition of which
enzyme or process explains the clinical manifestations of a-amanitin poisoning seen in these
patients?
(A) RNA polymerase II
(B) RNA polymerase I
(C) RNA splicing
(D) RNA polyadenylation
(E) RNA polymerase III

10. Which of the following is not an intermediate product of the citric acid cycle?
A. Alpha-ketoglutarate
B. Acetyl coenzyme A
C. Citrate
D. Oxaloacetate
E. Succinyl coenzyme A
1. Acetazolamide is a carbonic anhydrase inhibitor and is used in the treatment of all the
following conditions except which one?
(A) Dehydration
(B) Glaucoma
(C) Epilepsy
(D) Altitude sickness
(E) Congestive heart failure
The answer is A. Acetazolamide is a potent carbonic anhydrase inhibitor and helps to reduce
conditions of volume overload (not volume decrease, which would be brought about by dehy-
dration). In the eye, carbonic anhydrase inhibitors lead to a decrease in the secretion of
aqueous
humor, which reduces intraocular pressure. In patients with epilepsy, these inhibitors block the
activity of the central nervous system neuron carbonic anhydrase, which decreases excessive
neuronal discharge. In the treatment of individuals with altitude sickness, the mechanism of the
carbonic anhydrase inhibitor appears to be related to the acid-base effects of the drug. Patients
with congestive heart failure take these inhibitors, and the effect of the inhibitors is to act as a
diuretic, which helps to manage and reduce intravascular volume.

2. Which of the following is true regarding carbohydrate metabolism?


A. The principal carbohydrate used in body metabolism is galactose
B. Glycolysis is the process of glycogen formation
C. The pentose shunt is active in all cells of the body except red blood cells (RBCs)
D.The tricarboxylic acid (TCA) cycle is the common pathway for the oxidation of dietary
carbohydrates, fats and proteins to CO2 and H2O
E.Acetoacetic acid and beta-hydroxybutyric acid are types of fat
Answer D

3. Which of the following is an essential amino acid?


A.Arginine
B. Methionine
C. Glycine
D. Tryptophan
E.Valine
Answer B

4. An iron overload is not seen in which of the following conditions?


A. Thalassaemia major
B. Polycythaemia rubra vera
C. Myelodysplasia
D. Haemochromatosis
E. None of the above
Answer B
5. A 41-year-old woman presents with severe, sharp epigastric abdominal pain that radiates to
her back and with nausea and vomiting. Laboratory results indicate a serum triglyceride level
of 5000 mg/dL. She is diagnosed with pancreatitis, in part owing to her elevated serum triacyl-
glycerol levels. To form triacylglycerol from diacylglycerol, which of the following compounds is
also required?
(A) Glycerol
(B) Glycerol 3-phosphate
(C) Fatty acyl CoA
(D) Acetyl CoA
(E) Malonyl CoA
The answer is C. Triacylglycerol is formed when a diacylglycerol reacts with a fatty acyl CoA.
Glycerol and glycerol 3-phosphate form the backbone of the triacylglycerol. Acetyl CoA and
malonyl CoA are involved in fatty acid synthesis, and not directly in triacylglycerol synthesis.

6. A 17-year-old woman who recently began taking birth control pills presents to the emergency
room with cramping abdominal pain, anxiety, paranoia, and hallucinations. A surgical
evaluation, including ultrasound and computed tomography scan, fails to demonstrate an acute
abdominal process. A urinalysis reveals an increase in urine porphyrins. Which of the following
is the most likely?
(A) Congenital erythropoietic porphyria
(B) Variegate porphyria
(C) Porphyria cutanea tarda
(D) Acute intermittent porphyria
(E) Erythropoietic protoporphyria
The answer is D. Acute intermittent porphyria is an autosomal dominant disease resulting from
the deficiency of porphobilinogen deaminase (also known as hydroxymethylbilane synthase).
Often these intermittent attacks are provoked by drugs such as gonadal steroids, barbiturates,
or alcohol. These drugs are metabolized by cytochrome P-450 systems, which contain heme.
The presence of these drugs induces cytochrome P-450 synthesis (which includes an increase
in heme biosynthesis). The induction of heme biosynthesis is the event that leads to an
accumulation of the toxic intermediate. The other choices, including congenital erythropoietic
porphyria, porphyria cutanea tarda, variegate porphyria, and erythropoietic protoporphyria, are
considered erythropoietic porphyries, which are characterized by photosensitivity and rarely
exhibit abdominal pain.

7. A 73-year-old woman is admitted to the intensive care unit for septic shock from a urinary
tract infection. The critical care fellow is concerned she may not have an appropriate stress
response and orders a cosyntropin test. Which hormone does this test evaluate?
(A) Oxytocin
(B) Vasopressin
(C) Cortisol
(D) Corticotropin-releasing hormone (CRH)
(E) Adrenocorticotropic hormone (ACTH)
The answer is C. The primary stress hormone in the body is cortisol. Normally, CRH from the
hypothalamus stimulates the release of ACTH from the anterior pituitary. ACTH then acts on the
adrenal gland to produce cortisol. Cosyntropin is a synthetic ACTH injected to stimulate the
release of cortisol to evaluate an appropriate stress response. Thus, a cosyntropin test is meas-
uring the release of cortisol. Oxytocin is involved in labor during birth and milk ejection after-
ward. Vasopressin comes from the posterior pituitary and controls overall volume status and
blood pressure.

8. A 23-year-old woman is referred to an endocrinologist for weight gain, especially around


the waist. She also has striae over the abdomen and a rounded appearance to her face. She is
found to have Cushing disease. Which of the following would most likely be found in this
patient, compared with someone who does not have Cushing disease?
(A) Increased synthesis of immunoglobulins
(B) Increased protein synthesis
(C) Inhibition of lipolysis
(D) Increased gluconeogenesis
(E) Reduced liver glycogen stores
The answer is D. Cushing disease leads to excessive cortisol release due to an adenoma in the
pituitary gland, leading to the release of ACTH, which stimulates cortisol release from the adre-
nal gland. Excess levels of cortisol lead to a variety of debilitating symptoms, which include cen-
tral obesity, a ‘‘buffalo hump,’’ a round face (often referred to as a ‘‘moon face’’), excessive
sweating, dilation of capillaries (telangiectasia), thinning of the skin with purple or red striae,
hirsutism, sexual dysfunction, and mental changes. Cortisol is a ‘‘stress’’ hormone and is pre-
paring the tissues for survival during the stressful period. This includes an increase in lipolysis
and gluconeogenesis, and glycogen storage. Steroids, including cortisol, suppress the immune
response and are often administered exogenously to control autoimmune diseases. Thus, im-
munoglobulin synthesis would not be increased. Protein synthesis is also not increased in
response to cortisol.

9. Two couples present to the emergency room with severe nausea, vomiting, and diarrhea.
One of the patients admits that she had a dinner party and served a salad containing
mushrooms she had picked during a hike in the forest earlier that day. Inhibition of which
enzyme or process explains the clinical manifestations of a-amanitin poisoning seen in these
patients?
(A) RNA polymerase II
(B) RNA polymerase I
(C) RNA splicing
(D) RNA polyadenylation
(E) RNA polymerase III
The answer is A. The cellular toxin a-amanitin from the Amanita phalloides mushroom (death
cap mushroom) specifically inhibits RNA polymerase II, the enzyme required for mRNA synthe-
sis. Loss of transcriptional activity results in severe gastrointestinal symptoms, liver toxicity, and
sometimes death. Eukaryotic RNA polymerase I is responsible for the synthesis of rRNA,
whereas RNA polymerase III produces tRNA. Small ribonucleoproteins are required for RNA
splicing. RNA poly(A) polymerase normally adds stretches of adenine residues to the 30 end of
mRNA. The only enzyme inhibited by a-amanitin is RNA polymerase II.

10. Which of the following is not an intermediate product of the citric acid cycle?
A.Alpha-ketoglutarate
B. Acetyl coenzyme A
C. Citrate
D. Oxaloacetate
E.Succinyl coenzyme A
Answer B
StudyMRCOG
MRCOG Part 1
Genetics
Compiled by StudyMRCOG Mentor’s Team

These questions are complied from various sources/materials for MRCOG/MRCPI


study purpose, STUDYMRCOG doesn't own any credit or doesn't hold any copy rights
on this & used only for educational purpose.
1. Which of the following images is Acrocentric

A
B D
C

2. Which of the following statement is true regarding human chromosomes?


A. Banding with quinicrine fluorescent stain can be used to identify X chromosome
B. Banding can be used to determine polymorphism in populations
C. Telocentric chromosomes have a centrally placed centromere
D. Terminal fragments called ‘satellites’ are present in the metacentric chromosomes
E. Telocentric chromosomes are present in human
3. Which of the following is true regarding Down syndrome?
A. Shows X-linked pattern of inheritance
B. There may be mild to moderate mental retardation
C. Is characterised by the presence of muscle hypertonia
D. Larger than normal space between the second and third toe
E. The commonest type of congenital heart defect which may be present is coarctation of aorta

4. 46 XX karyotype is associated with which of the condition?


A. Testicular feminisation syndrome
B. Klinefelter’s syndrome
C. Muscular dystrophy
D. Constitutional hirsutism
E. None of the above

5. Which of the following is the leading cause of Down Syndrome?


A) Nondisjunction maternal gamete
B) Nondisjunction paternal gamete
C) Mitotic Nondisjunction after conception
D) Robertsonian Translocation
E) Ring Chromosome
6.A 39-year-old multiparous woman is 13 weeks pregnant. She has serum screening as part of
the combined test. Analysis shows an elevated level of α-fetoprotein and a normal level of
pregnancy-associated plasma protein A.
What diagnosis are the screening results suggestive of?
A Down’s syndrome
B Edwards’ syndrome
C Multiple pregnancy
D Neural tube defect
E Normal pregnancy

7.A couple are both known to carry the trait for a haemoglobinopathy. They decline any
invasive testing when they conceive their first pregnancy. At an anomaly scan at 20 weeks’
gestation the fetus is found to have severe hydrops. In utero death occurs at 22 weeks'
gestation.
What is the most likely cause of the fetal demise?
A Alpha-thalassaemia with deletion of 4α-globin genes
B Beta-thalassaemia major
C Glucose-6-dehydrogenase deficiency
D Haemoglobin H disease
E Sickle cell disease
8. You are counseling a 16-year-old G1P0 girl and her 16-year-old boyfriend. Two weeks ago
they received a quad screen result with a 1:50 risk for Down syndrome. She has noninformative
family history and medical history.
You begin counseling by explaining that the number of chromosomes in a fetus is how many?
(A) 23
(B) 46
(C) 48
(D) 47 for a female fetus
(E) 21 for Down syndrome

9. A 19-year-old woman with a complaint of never having had menses comes to your office.
Physical examination shows that she is 1.37 m tall and weighs 94 lb. She lacks breast and
pubic hair development.
There is webbing of her neck and cubitus valgus.
Which of the following would be the simplest, yet most useful, initial test to begin her
evaluation?
(A) serum estrogen level
(B) prolactin
(C) thyroid index
(D) serum follicle-stimulating hormone (FSH) and luteinizing hormone (LH)
(E) a cardiogram
10. A couple presents to your office during the ninth week of pregnancy. Their previous child
was diagnosed ith trisomy 18, so they are extremely anxious and would like prenatal diagnosis
as early as possible.
They have many questions about chorionic villus sampling (CVS).
A mother with blood group AB has an AB child. She would like to establish paternity through
blood typing. Which blood type excludes a male from being the potential biologic father?
(A) AA
(B) BB
(C) BO
(D) AO
(E) OO
1. Which of the following images is Acrocentric

A C D
B

Answer B
2. Which of the following statement is true regarding human chromosomes?
A. Banding with quinicrine fluorescent stain can be used to identify X chromosome
B. Banding can be used to determine polymorphism in populations
C. Telocentric chromosomes have a centrally placed centromere
D. Terminal fragments called ‘satellites’ are present in the metacentric chromosomes
E. Telocentric chromosomes are present in human
Answer B

3. Which of the following is true regarding Down syndrome?


A. Shows X-linked pattern of inheritance
B. There may be mild to moderate mental retardation
C. Is characterised by the presence of muscle hypertonia
D. Larger than normal space between the second and third toe
E. The commonest type of congenital heart defect which may be present is coarctation of aorta
Answer B

4. 46 XX karyotype is associated with which of the condition?


A. Testicular feminisation syndrome
B. Klinefelter’s syndrome
C. Muscular dystrophy
D. Constitutional hirsutism
E. None of the above
Answer D

5. Which of the following is the leading cause of Down Syndrome?


A)Nondisjunction maternal gamete
B) Nondisjunction paternal gamete
C) Mitotic Nondisjunction after conception
D) Robertsonian Translocation
E)Ring Chromosome
Correct Answer:
Nondisjunction maternal gamete
Explanation: Nondisjunction accounts for approximately 95% of Down Syndrome.
88% is due to nondisjunction of the maternal gamete
8% is due to the paternal gamete
Mitotic Nondisjunction after conception leads to mosaicism.
Translocation Down Syndromes account for 4.5% of Down Syndromes

6. A 39-year-old multiparous woman is 13 weeks pregnant. She has serum screening as part of
the combined test. Analysis shows an elevated level of α-fetoprotein and a normal level of
pregnancy-associated plasma protein A.
What diagnosis are the screening results suggestive of?
A Down’s syndrome
B Edwards’ syndrome
C Multiple pregnancy
D Neural tube defect
E Normal pregnancy
Answer D Neural tube defect
The combined test uses serum markers together with fetal nuchal translucency measurement in
order to obtain an estimate as to how likely it is that a particular fetus is affected by a
chromosomal or genetic disorder. Alpha-fetoprotein (AFP) is produced by the fetal liver and yolk
sac. High levels of AFP are suggestive of neural tube defects, such as spina bifida, or more
rarely anencephaly. This raised level is a consequence of flow of AFP from the open neural tube
of the fetus, into the amniotic fluid (and secondarily maternal serum). Pregnancy-associated
plasma protein A (PAPP-A) is produced by the fetus and also by the placenta. Reduced levels
of PAPP-A in maternal serum can be suggestive of a fetus with an aneuploidy, such as Down’s
syndrome. Intrauterine growth restriction has also been associated with reduced levels of
PAPP-A.

7. A couple are both known to carry the trait for a haemoglobinopathy. They decline any
invasive testing when they conceive their first pregnancy. At an anomaly scan at 20 weeks’
gestation the fetus is found to have severe hydrops. In utero death occurs at 22 weeks'
gestation.
What is the most likely cause of the fetal demise?
A Alpha-thalassaemia with deletion of 4α-globin genes
B Beta-thalassaemia major
C Glucose-6-dehydrogenase deficiency
D Haemoglobin H disease
E Sickle cell disease
Answer A Alpha-thalassaemia with deletion of 4α-globin genes
Alpha-thalassaemia is an autosomal recessive condition associated with deletions in genes
responsible for the production of α-globin chains. This couple both have α-thalassaemia trait,
which means they each have either 2α-globin gene deletions on the same chromosomes or two
chromosomes each with an α-globin gene deletion. Severity of α-thalassaemia is largely
dependent on the number of α-globin genes affected. Individuals with the α-thalassaemia trait
are usually minimally affected other than a mild anaemia. However in fetuses with deletions in
all of the 4α-globin chains there is a complete absence of α-globin production. In early fetal life
embryonic globin and gamma chains form functional units. At later gestations there is formation
of haemoglobin Bart’s (excess gamma chains which form tetramers) which has a high affinity
for oxygen and therefore little oxygen delivery capacity. Hydrops fetalis is seen in fetuses with
this profound absence of α-globin. Previously thought to be incompatible with life some fetuses
may now survive with intrauterine transfusion.

8. You are counseling a 16-year-old G1P0 girl and her 16-year-old boyfriend. Two weeks ago
they received a quad screen result with a 1:50 risk for Down syndrome. She has noninformative
family history and medical history.
You begin counseling by explaining that the number of chromosomes in a fetus is how many?
(A) 23
(B) 46
(C) 48
(D) 47 for a female fetus
(E) 21 for Down syndrome
(B) There are 23 pairs of chromosomes in the normal human cell. Twenty-two are alike in males
and females and are called autosomes. The remaining pair are sex chromosomes, the X and
the Y, anddetermine genetic sex. In Figure 4–1, a 47,XX + 21 chromosome complement is
illustrated.
9. A 19-year-old woman with a complaint of never having had menses comes to your office.
Physical examination shows that she is 1.37 m tall and weighs 94 lb. She lacks breast and
pubic hair development.
There is webbing of her neck and cubitus valgus.
Which of the following would be the simplest, yet most useful, initial test to begin her
evaluation?
(A) serum estrogen level
(B) prolactin
(C) thyroid index
(D) serum follicle-stimulating hormone (FSH) and luteinizing hormone (LH)
(E) a cardiogram
Answer (D) An evaluation of serum gonadotropins would demonstrate ovarian failure, one of the
hallmarks of this syndrome. Karyotyping is not always necessary.

10. A couple presents to your office during the ninth week of pregnancy. Their previous child
was diagnosed ith trisomy 18, so they are extremely anxious and would like prenatal diagnosis
as early as possible.
They have many questions about chorionic villus sampling (CVS).
A mother with blood group AB has an AB child. She would like to establish paternity through
blood typing. Which blood type excludes a male from being the potential biologic father?
(A) AA
(B) BB
(C) BO
(D) AO
(E) OO
Answer (E) The father could not be type O, unless of the very rare Bombay phenotype, since
the child would have been AO or BO and could not be AB.
1. What is the inheritance of sickle cell disease(SCD)?
A) single gene autosomal dominant
B) polygenic autosomal dominant
C) single gene autosomal recessive
D) polygenic autosomal recessive
E) X‐linked dominant

2. Which of the following is true regarding Down syndrome?


A. Shows X-linked pattern of inheritance
B. There may be mild to moderate mental retardation
C. Is characterised by the presence of muscle hypertonia
D. Larger than normal space between the second and third toe
E. The commonest type of congenital heart defect which may be present is coarctation of aorta

3. A 34-year-old woman delivers a male baby with Down’s syndrome. Chromosomal analysis
following his birth is suggestive of familial Down’s syndrome. What chromosomal event best
describes the aetiology of Familial Down’s syndrome?
A Microdeletion
B Nonsense mutation
C Reciprocal translocation
D Robertsonian translocation
E Triplet repeat expansion
5. A 25-year-old G1P0, 20 weeks’ gestation has a family history of classic hemophilia. Her uncle
on her mother’s side and her brother are affected. A perinatal ultrasound notes a male fetus.
What is the chance that her son (in utero) will inherit hemophilia?
(A) 25%
(B) 50%
(C) 33%
(D) 100%
(E) Impossible to say

A.16 year old girl has been referred to clinic as she has amenorrhoea. She reports never havin
periods. Her mother and 2 sisters menarche was at age 12. On examination you note the
patient is 152cm tall and BMI 29.0 and secondary sexual characteristics are not developed. Her
FSH is elevated. You organise a Karyotype and are phoned as the result is abnormal, what
does the result show?
A) 46X
B) 47XXX
C) 47XXY
D) 46XX
E) 45X

7. A 2-year-old boy with Down syndrome requires intubation in the intensive care unit
due to difficulty breathing. He is afflicted with congenital heart disease associated with the dis-
ease, and he dies shortly after admission. What is the most common genetic cause of Down
syndrome?
(A) Meiotic nondisjunction
(B) Autosomal dominant inheritance
(C) X-linked recessive inheritance
(D) Decreased maternal age
(E) Monosomy 21

8. What cells in the spermatogenesis process can undergo mitotic


division?
A. Primary spermatocytes
B. Secondary spermatocytes
C. Spermatids
D. Spermatogonia
E. Spermatozoa

9. A woman with blood type O gave birth to an AB infant. She and her partner are quite
concerned that there may be a mix-up in the nursery. What is the most likely diagnosis?
(A) Lyon hypothesis
(B) chimerism
(C) Bombay phenotype
(D) laboratory error
(E) a maternal blocking antibody

10. A 27-year-old female developed insulin dependent diabetes mellitus. Her uncle and
grandmother also had diabetes mellitus. What is the most likely mode of inheritance for her
condition?
A. Autosomal co-dominant
B. Autosomal dominant
C. Autosomal recessive
D. Polygenic
E. Single gene defect
1. What is the inheritance of sickle cell disease(SCD)?
A) single gene autosomal dominant
B) polygenic autosomal dominant
C) single gene autosomal recessive
D) polygenic autosomal recessive
E) X‐linked dominant
Answer C

2. Which of the following is true regarding Down syndrome?


A.Shows X-linked pattern of inheritance
B. There may be mild to moderate mental retardation
C. Is characterised by the presence of muscle hypertonia
D. Larger than normal space between the second and third toe
E.The commonest type of congenital heart defect which may be present is coarctation of aorta
Answer B

3. A 34-year-old woman delivers a male baby with Down’s syndrome. Chromosomal analysis
following his birth is suggestive of familial Down’s syndrome. What chromosomal event best
describes the aetiology of Familial Down’s syndrome?
A Microdeletion
B Nonsense mutation
C Reciprocal translocation
D Robertsonian translocation
E Triplet repeat expansion
Answer D Robertsonian translocation
Familial Down’s syndrome refers to trisomy 21 that occurs following a Robertsonian
translocation. These translocations only occur in humans at chromosomes 13, 14, 15, 21 and
22. These chromosomes are acrocentric, i.e. they have very short arms. This form of
translocation occurs when there is fusion of one acrocentric chromosome onto another
acrocentric chromosome, so that their long arms become fused, with the loss of the insignificant
genetic material found in the short arms. This may result in either a balanced Robertsonian
translocation, whereby there is the no overall loss or excess of chromosomal material, or an
unbalanced translocation. In the latter there may be an extra copy of one of the chromosomes,
i.e. a trisomy or a monosomy. Down’s syndrome caused by a Robertsonian translocation is
usually caused by the inheritance of two normal copies of chromosome 21 (one from each
parent) and the inheritance of a balanced translocation chromosome from one parent. This
translocation chromosome, typically chromosome 14, has a copy of chromosome 21 fused to its
long arm, leading to trisomy 21. Couples where one partner carries such a translocation
chromosome have around a 10% chance of each of their pregnancies being affected by Down’s
syndrome.

5. A 25-year-old G1P0, 20 weeks’ gestation has a family history of classic hemophilia. Her uncle
on her mother’s side and her brother are affected. A perinatal ultrasound notes a male fetus.
What is the chance that her son (in utero) will inherit hemophilia?
(A) 25%
(B) 50%
(C) 33%
(D) 100%
(E) Impossible to say
Answer (A) Classic hemophilia is X-linked. The mother has a 50% chance of being a carrier.
Since her brother
has the disease, we know the patient has the potential of being positive. Since the fetus is a
male, it has
a 50% chance of getting the affected X chromosome. Thus 50% chance of 50% is 25%.

A.16 year old girl has been referred to clinic as she has amenorrhoea. She reports never havin
periods. Her mother and 2 sisters menarche was at age 12. On examination you note the patient
is 152cm tall and BMI 29.0 and secondary sexual characteristics are not developed. Her FSH is
elevated. You organise a Karyotype and are phoned as the result is abnormal, what does the
result show?
A) 46X
B) 47XXX
C) 47XXY
D) 46XX
E) 45X
Correct Answer: 45X
Explanation: The scenario describes Turner Syndrome. There is a loss of an X‐chromosome so
the patient only has 45 instead of 46 chromosomes. The Karyotype can either be written 45X or
45XO.
It is relatively common affecting 1 in 2500 females at birth (though it has a good postnatal
prognosis 99% of Turner Syndrome conceptions will result in spontaneous abortion or stillbirth)
47XXX describes an extra X chromosome (triple X syndrome). These patients usually exhbit no
clinical signs though there is an association with tall stature and microcephaly.
47 XXY is male phenotype seen in Klinefelter's
46XX is normal female karyotype
46X doesn't exist.

7. A 2-year-old boy with Down syndrome requires intubation in the intensive care unit
due to difficulty breathing. He is afflicted with congenital heart disease associated with the dis-
ease, and he dies shortly after admission. What is the most common genetic cause of Down
syndrome?
(A) Meiotic nondisjunction
(B) Autosomal dominant inheritance
(C) X-linked recessive inheritance
(D) Decreased maternal age
(E) Monosomy 21
Answer A Meiotic nondisjunction is the failure of chromosomes to separate and move
to opposite poles of the dividing cell. This results in two copies of the same chromosome (chro-
mosome 21 in this case) in one gamete, and nullisomy (no chromosome 21) in the other ga-
mete. If the gamete with two copies of chromosome 21 is fertilized by a gamete with another
chromosome 21, trisomy 21 will result. This is the most common cause of Down syndrome
(monosomy 21 is incompatible with life). Down syndrome does not occur due to autosomal
dominant inheritance. There is no dominant gene that offspring can inherit that would lead to
this disorder. Because Down syndrome is due to trisomy 21, it is clearly not X-linked. The inci-
dence of Down syndrome is essentially equal between boys and girls. Increased (not
decreased) maternal age is associated with increased incidence of meiotic nondisjunction.

8. What cells in the spermatogenesis process can undergo mitotic


division?
A. Primary spermatocytes
B. Secondary spermatocytes
C. Spermatids
D. Spermatogonia
E. Spermatozoa
The correct answer is spermatogonia.

9. A woman with blood type O gave birth to an AB infant. She and her partner are quite
concerned that there may be a mix-up in the nursery. What is the most likely diagnosis?
(A) Lyon hypothesis
(B) chimerism
(C) Bombay phenotype
(D) laboratory error
(E) a maternal blocking antibody
Answer C
The Bombay phenotype is very rare and has inactive alleles for H antigen. If no H is
formed,neither A nor B antigen can be formed, and the patient’s blood will be type O even if the
A or B gene is
present. Laboratory error is always possible, but given the caution with which blood typing is
done, itis unlikely. Blocking antibodies will not disguise a blood type.

10. A 27-year-old female developed insulin dependent diabetes mellitus. Her uncle and
grandmother also had diabetes mellitus. What is the most likely mode of inheritance for her
condition?
A. Autosomal co-dominant
B. Autosomal dominant
C. Autosomal recessive
D. Polygenic
E. Single gene defect
Answer D
1. Which of the following technique is not directly used for used for identifying DNA?
A. Denaturing gradient gel electrophoresis
B. Northern blotting
C. Polymerase chain reaction
D. Southern blotting
E. None of the above

2. Which of the following is true regarding hermaphroditism?


A. Chromosomal sex is usually female (46,XX)
B. End-organ resistance is a feature
C. External sex is usually female
D. Mosaics do not occur
E. Either the primordial follicles or the seminiferous tubules are present

3. Which of the following disorders have an autosomal recessive pattern of inheritance?


A. Christmas disease
B. Neurofibromatosis
C. Colour blindness
D. Tay Sachs disease
E. Achondroplasia
4. increased numbers of chromosomes occur in which of the following condition?
A. Fragile X syndrome
B. Down’s syndrome
C. Phenylketonuria
D. Turners syndrome
E. Cri du chat syndrome

5. Which of the following conditions is not associated with genetic anticipation?


A. Cystic fibrosis
B. Fragile X syndrome
C. Huntington’s chorea
D. Myotonic dystrophy
E. None of the above

6. 46 XY karyotype is associated with which of the following condition?


A. MRKH syndrome
B. Testicular feminisation syndrome
C. Klinefelter’s syndrome
D. All the above
E. None of the above
7.A 32-year-old woman is 15 weeks pregnant with her second pregnancy. She opts to have
antenatal screening and has blood taken as part of the quadruple test. The result shows
reduced levels of α-fetoprotein and unconjugated oestriol with elevated β-human chorionic
gonadotrophin.
Which of the following is the most likely explanation for the screening results:
A Down’s syndrome
B Edwards’ syndrome
C Multiple pregnancy
D Neural tube defect
E Normal pregnancy

8.A couple are both known to carry the trait for a haemoglobinopathy. They decline any
invasive testing when they conceive their first pregnancy. At an anomaly scan at 20 weeks’
gestation the fetus is found to have severe hydrops. In utero death occurs at 22 weeks'
gestation.
What is the most likely cause of the fetal demise?
A Alpha-thalassaemia with deletion of 4α-globin genes
B Beta-thalassaemia major
C Glucose-6-dehydrogenase deficiency
D Haemoglobin H disease
E Sickle cell disease
9.In baby with Down's syndrome , the probability of extra chromosome 21 being maternal in
origin
A) 100%
B) 95-100%
C) 80-85%
D) 70-79%
E) 50%

10. A 46 XY karyotype associated with a female phenotype is found in which of the following?
A A child whose mother has been treated with stilboestrol
B A patient with testicular feminisation syndrome (androgen insensitivity)
C An infant whose mother has thyrotoxicosis
D An infant whose mother is treated with prednisolone
E Noonan's syndrome
1. Which of the following technique is not directly used for used for identifying DNA?
A. Denaturing gradient gel electrophoresis
B. Northern blotting
C. Polymerase chain reaction
D. Southern blotting
E. None of the above
Answer D

2. Which of the following is true regarding hermaphroditism?


A.Chromosomal sex is usually female (46,XX)
B. End-organ resistance is a feature
C. External sex is usually female
D. Mosaics do not occur
E.Either the primordial follicles or the seminiferous tubules are present
Answer A

3. Which of the following disorders have an autosomal recessive pattern of inheritance?


A. Christmas disease
B. Neurofibromatosis
C. Colour blindness
D. Tay Sachs disease
E. Achondroplasia
Answer D

4. increased numbers of chromosomes occur in which of the following condition?


A. Fragile X syndrome
B. Down’s syndrome
C. Phenylketonuria
D. Turners syndrome
E. Cri du chat syndrome
Answer B

5. Which of the following conditions is not associated with genetic anticipation?


A. Cystic fibrosis
B. Fragile X syndrome
C. Huntington’s chorea
D. Myotonic dystrophy
E. None of the above
Answer A

6. 46 XY karyotype is associated with which of the following condition?


A. MRKH syndrome
B. Testicular feminisation syndrome
C. Klinefelter’s syndrome
D. All the above
E. None of the above
Answer B

7. A 32-year-old woman is 15 weeks pregnant with her second pregnancy. She opts to have
antenatal screening and has blood taken as part of the quadruple test. The result shows
reduced levels of α-fetoprotein and unconjugated oestriol with elevated β-human chorionic
gonadotrophin.
Which of the following is the most likely explanation for the screening results:
A Down’s syndrome
B Edwards’ syndrome
C Multiple pregnancy
D Neural tube defect
E Normal pregnancy
Answer A
Down’s syndrome, trisomy 21, is one of a number of conditions that may be screened for using
serum markers, as part of the quadruple test. Reduced serum α-fetoprotein (AFP) is associated
with pregnancies affected by Down’s syndrome. AFP is produced by the fetal liver and yolk sac.
Reduced levels of AFP are thought to be secondary to the smaller size of fetuses affected by
Down’s syndrome. Beta-human chorionic gonadotrophin levels are often elevated in Down’s
syndrome pregnancies. The quadruple test does not aim to provide a definitive diagnosis, but is
a screening test with both false positive and false negative results.

8. A couple are both known to carry the trait for a haemoglobinopathy. They decline any
invasive testing when they conceive their first pregnancy. At an anomaly scan at 20 weeks’
gestation the fetus is found to have severe hydrops. In utero death occurs at 22 weeks'
gestation.
What is the most likely cause of the fetal demise?
A Alpha-thalassaemia with deletion of 4α-globin genes
B Beta-thalassaemia major
C Glucose-6-dehydrogenase deficiency
D Haemoglobin H disease
E Sickle cell disease
Answer A Alpha-thalassaemia with deletion of 4α-globin genes
Alpha-thalassaemia is an autosomal recessive condition associated with deletions in genes
responsible for the production of α-globin chains. This couple both have α-thalassaemia trait,
which means they each have either 2α-globin gene deletions on the same chromosomes or two
chromosomes each with an α-globin gene deletion. Severity of α-thalassaemia is largely
dependent on the number of α-globin genes affected. Individuals with the α-thalassaemia trait
are usually minimally affected other than a mild anaemia. However in fetuses with deletions in all
of the 4α-globin chains there is a complete absence of α-globin production. In early fetal life
embryonic globin and gamma chains form functional units. At later gestations there is formation
of haemoglobin Bart’s (excess gamma chains which form tetramers) which has a high affinity
for oxygen and therefore little oxygen delivery capacity. Hydrops fetalis is seen in fetuses with
this profound absence of α-globin. Previously thought to be incompatible with life some fetuses
may now survive with intrauterine transfusion.

9.In baby with Down's syndrome , the probability of extra chromosome 21 being maternal in
origin
A) 100%
B) 95-100%
C) 80-85%
D) 70-79%
E) 50%
Answer C

10.A 46 XY karyotype associated with a female phenotype is found in which of the following?
A A child whose mother has been treated with stilboestrol
B A patient with testicular feminisation syndrome (androgen insensitivity)
C An infant whose mother has thyrotoxicosis
D An infant whose mother is treated with prednisolone
E Noonan's syndrome
Answer B
1. Which of the following is true concerning the Müllerian ducts?
A. Are derived from coelomic epithelium
B. Form the vas deferens
C. Form urogenital sinus in their lowest part
D. Fuse from above downwards
E. Grow medial to the Wolffian ducts

2. Which of the following structures do not arise from the Wolffian ducts?
A. Epididymis
B. Epoöphoron
C. Gartner’s duct
D. Paroöphoron
E. Round ligament

3. at birth which of the following changes occur in the foetus?


A. Pulmonary vascular resistance decreases
B. The aortic pressure decreases
C. The left ventricular pressure decreases
D. The right atrial pressure increases
E. The right ventricular pressure increases
4.The cells of which structure form the chorionic villi?
A Neural crest
B Endoderm
C Primitive streak
D Hypoblast
E Extraembryonic mesoderm

5.Which of the following is not a derivative of the vitelline vein?


A Lower inferior vena cava
B Inferior mesenteric vein
C Superior mesenteric vein
D Portal vein
E Hepatic vein

6. Which of the following is true regarding the ductus venosus?


A. Carries blood with a higher PO2 than umbilicalarterial blood
B. Gives rise to the ligamentum teres
C. Is a shunt preventing blood from passing to the foetal lungs
D. Is derived from the anterior cardinal vein
E. Is part of the embryonic heart
7. During ovulation, the secondary oocyte
resides at what specific stage of meiosis?
(A) Prophase of meiosis I
(B) Prophase of meiosis II
(C) Metaphase of meiosis I
(D) Metaphase of meiosis II
(E) Meiosis is completed at the time of Ovulation

8. When does a secondary oocyte complete its second meiotic division to become a mature
ovum?
(A) At ovulation
(B) Before ovulation
(C) At fertilization
(D) At puberty
(E) Before birth

9. Which of the following components plays the most active role in invading the endome-
trium during blastocyst implantation?
(A) Epiblast
(B) Syncytiotrophoblast
(C) Hypoblast
(D) Extraembryonic somatic mesoderm
(E) Extraembryonic visceral mesoderm

10. Which process establishes the three definitive germ layers?


(A) Neurulation
(B) Gastrulation
(C) Craniocaudal folding
(D) Lateral folding
(E) Angiogenesis
1. Which of the following is true concerning the Müllerian ducts?
A.Are derived from coelomic epithelium
B. Form the vas deferens
C. Form urogenital sinus in their lowest part
D. Fuse from above downwards
E.Grow medial to the Wolffian ducts
Answer A

2. Which of the following structures do not arise from the Wolffian ducts?
A. Epididymis
B. Epoöphoron
C. Gartner’s duct
D. Paroöphoron
E. Round ligament
Answer E

3. at birth which of the following changes occur in the foetus?


A. Pulmonary vascular resistance decreases
B. The aortic pressure decreases
C. The left ventricular pressure decreases
D. The right atrial pressure increases
E. The right ventricular pressure increases
Answer A

4.The cells of which structure form the chorionic villi?


A Neural crest
B Endoderm
C Primitive streak
D Hypoblast
E Extraembryonic mesoderm
Answer E Extraembryonic mesoderm
It is the cells of the extraembryonic mesoderm that develop into the chorionic villi. The neural
crest gives rise to the nervous system and melanocytes. Endoderm is one of the germ cell
layers and produces the gastrointestinal tract, respiratory tract and endocrine organs. The
primitive streak develops by the end of the second week in the bilaminar embryonic disc and
functions to determine symmetry of the developing embryo. The hypoblast is a part of the inner
cell mass and lies beneath the epiblast. It gives rise to extraembryonic endoderm.

5. Which of the following is not a derivative of the vitelline vein?


A Lower inferior vena cava
B Inferior mesenteric vein
C Superior mesenteric vein
D Portal vein
E Hepatic vein
Answer B Inferior mesenteric vein
The vitelline veins take blood away from the yolk sac. The vitelline veins give rise to the hepatic
veins, the inferior part of the inferior vena cava, the superior mesenteric vein and the portal
vein. The inferior mesenteric vein is not a derivative of the vitelline vein.

6. Which of the following is true regarding the ductus venosus?


A.Carries blood with a higher PO2 than umbilicalarterial blood
B. Gives rise to the ligamentum teres
C. Is a shunt preventing blood from passing to the foetal lungs
D. Is derived from the anterior cardinal vein
E.Is part of the embryonic heart
Answer A

7. During ovulation, the secondary oocyte resides at what specific stage of meiosis?
(A) Prophase of meiosis I
(B) Prophase of meiosis II
(C) Metaphase of meiosis I
(D) Metaphase of meiosis II
(E) Meiosis is completed at the time of Ovulation
Answer D. The secondary oocyte is arrested in metaphase of meiosis II about 3 hours before
ovulation, and it remains in this meiotic stage until fertilization.
8. When does a secondary oocyte complete its second meiotic division to become a mature
ovum?
(A) At ovulation
(B) Before ovulation
(C) At fertilization
(D) At puberty
(E) Before birth
Answer C. At ovulation, a secondary oocyte begins meiosis II, but this division is arrested at
metaphase.
The secondary oocyte will remain arrested in metaphase until a sperm penetrates it at fertiliza-
tion. Therefore, the term “mature ovum’” is somewhat of a misnomer because it is a secondary
oocyte that is fertilized, and, once fertilized, the new diploid cell is known as a zygote. If fertil-
ization does not occur, the secondary oocyte degenerates.

9. Which of the following components plays the most active role in invading the endome-
trium during blastocyst implantation?
(A) Epiblast
(B) Syncytiotrophoblast
(C) Hypoblast
(D) Extraembryonic somatic mesoderm
(E) Extraembryonic visceral mesoderm
Answer B. The syncytiotrophoblast plays the most active role in invading the endometrium of
the mother’s uterus. During the invasion, endometrial blood vessels and endometrial glands are
eroded and a lacunar network is formed.

10. Which process establishes the three definitive germ layers?


(A) Neurulation
(B) Gastrulation
(C) Craniocaudal folding
(D) Lateral folding
(E) Angiogenesis
Answer B. Gastrulation establishes the three primary germ layers during week 3 of
development. Neurulation is the process by which neuroectoderm forms the neural plate, which
eventually folds to form the neural tube.
1. In the production of male gametes, which of the following cells remains dormant for 12
years?
(A) Primordial germ cell
(B) Primary spermatocyte
(C) Secondary spermatocyte
(D) Spermatid
(E) Sperm

2. Which of the following is the origin of the mitochondrial DNA of all human adult cells?
(A) Paternal only
(B) Maternal only
(C) A combination of paternal and maternal
(D) Either paternal or maternal
(E) Unknown origin

3. In oogenesis, which of the following events occurs immediately following the completions
of meiosis II?
(A) Degeneration of the zona pellucida
(B) Sperm penetration of the corona radiata
(C) Formation of a female pronucleus
(D) Appearance of the blastocyst
(E) Completion of cleavage
4. At the end of week 2 of development (day 14), what is the composition of the embryonic
disk?
(A) Epiblast only
(B) Epiblast and hypoblast
(C) Ectoderm and endoderm
(D) Ectoderm, mesoderm, and endoderm
(E) Epiblast, mesoderm, and hypoblast

5. The lateral mesoderm is divided into two distinct layers by the formation of the
(A) extraembryonic coelom
(B) intraembryonic coelom
(C) cardiogenic region
(D) notochord
(E) yolk sac

6. Somites may differentiate into which of the following?


(A) Urogenital ridge
(B) Kidneys
(C) Notochord
(D) Epimeric and hypomeric muscles
(E) Epithelial lining of the gastrointestinal tract
7. Intermediate mesoderm will give rise to the
(A) neural tube
(B) heart
(C) kidneys and gonads
(D) somites
(E) Notochord

8. A woman has her pregnancy suddenly terminated due to intrauterine fetal death.
At autopsy, the fetus shows severe pallor, generalized edema, and hepatosplenomegaly. Which
of the following would you suspect?
(A) VATER
(B) β-Thalassemia minor
(C) β-Thalassemia major
(D) Hydrops fetalis
(E) VACTERL

9. Which structure is derived from the same embryonic primordium as the dorsal root ganglia?
(A) Gonads
(B) Kidney
(C) Pineal gland
(D) Liver
(E) Adrenal medulla
10. The specialized group of mesenchymal cells that aggregate to form blood islands centrally
and primitive blood vessels peripherally are called
(A) fibroblasts
(B) cardiac progenitor cells
(C) angioblasts
(D) myoblasts
(E) osteoblasts
1. In the production of male gametes, which of the following cells remains dormant for 12
years?
(A) Primordial germ cell
(B) Primary spermatocyte
(C) Secondary spermatocyte
(D) Spermatid
(E) Sperm
Answer A. Primordial germ cells migrate from the wall of the yolk sac during week 4 of
embryonic life and enter the gonad of a genetic male, where they remain dormant until puberty
(about age 12 years), when hormonal changes in the young man stimulate the production of
sperm.

2. Which of the following is the origin of the mitochondrial DNA of all human adult cells?
(A) Paternal only
(B) Maternal only
(C) A combination of paternal and maternal
(D) Either paternal or maternal
(E) Unknown origin
Answer B. The mitochondrial DNA of all human adult cells is of maternal origin only. In human
fertilization, the entire sperm enters the secondary oocyte cytoplasm. However, sperm
mitochondria degenerate along with the sperm’s tail. Therefore, only mitochondria present
within the secondary oocyte (maternal) remain in the fertilized zygote
3. In oogenesis, which of the following events occurs immediately following the completions
of meiosis II?
(A) Degeneration of the zona pellucida
(B) Sperm penetration of the corona radiata
(C) Formation of a female pronucleus
(D) Appearance of the blastocyst
(E) Completion of cleavage
Answer C. The secondary oocyte is arrested in metaphase of meiosis II, and it will remain in
this meiotic stage until fertilization occurs. Following fertilization, the secondary oocyte
completes meiosis II, forming a mature ovum and a polar body. The nucleus of the mature ovum
is called the female pronucleus, which fuses with the male pronucleus to form a zygote.

4. At the end of week 2 of development (day 14), what is the composition of the embryonic
disk?
(A) Epiblast only
(B) Epiblast and hypoblast
(C) Ectoderm and endoderm
(D) Ectoderm, mesoderm, and endoderm
(E) Epiblast, mesoderm, and hypoblast
Answer B. The embryoblast consists of the two distinct cell layers (epiblast and hypoblast) at
the end of development week 2 (day 14) and forms the bilaminar embryonic disk.
5. The lateral mesoderm is divided into two distinct layers by the formation of the
(A) extraembryonic coelom
(B) intraembryonic coelom
(C) cardiogenic region
(D) notochord
(E) yolk sac
Answer B. The lateral mesoderm is a subdivision of intraembryonic mesoderm and initially is a
solid plate of mesoderm. The intraembryonic coelom forms in the middle of the lateral
mesoderm, thereby dividing it into the intraembryonic somatic mesoderm and intraembryonic
visceral mesoderm.

6. Somites may differentiate into which of the following?


(A) Urogenital ridge
(B) Kidneys
(C) Notochord
(D) Epimeric and hypomeric muscles
(E) Epithelial lining of the gastrointestinal tract
Answer D. Approximately 35 pairs of somites form. They are derived from a specific subdivision
of intraembryonic mesoderm called paraxial mesoderm. Somites differentiate into the
components called sclerotome (cartilage and bone of the vertebral column), myotome
(epimeric and hypomeric muscle), and dermatome (dermis and subcutaneous area of skin).
7. Intermediate mesoderm will give rise to the
(A) neural tube
(B) heart
(C) kidneys and gonads
(D) somites
(E) Notochord
Answer C. Intermediate mesoderm is a subdivision of intraembryonic mesoderm that forms a
longitudinal dorsal ridge called the urogenital ridge from which the kidneys and gonads develop.

8. A woman has her pregnancy suddenly terminated due to intrauterine fetal death.
At autopsy, the fetus shows severe pallor, generalized edema, and hepatosplenomegaly. Which
of the following would you suspect?
(A) VATER
(B) β-Thalassemia minor
(C) β-Thalassemia major
(D) Hydrops fetalis
(E) VACTERL
Answer D. Hydrops fetalis is the most severe form of α-thalassemia, and is a direct result of the
lack or decreased synthesis of the α-globin chain of hemoglobin α2β2.

9. Which structure is derived from the same embryonic primordium as the dorsal root ganglia?
(A) Gonads
(B) Kidney
(C) Pineal gland
(D) Liver
(E) Adrenal medulla
Answer E. Both the chromaffin cells of the adrenal medulla and the dorsal root ganglia are
derived from neural crest cells.

10. The specialized group of mesenchymal cells that aggregate to form blood islands centrally
and primitive blood vessels peripherally are called
(A) fibroblasts
(B) cardiac progenitor cells
(C) angioblasts
(D) myoblasts
(E) Osteoblasts
Answer B. The angioblasts are the mesenchymal cells that form blood vessels in embryonic
development, as well as embryonic blood cells.
1. Which of the following is true regarding meiosis?
A. Meiosis II is a reductional division
B. Exchange of paternal and maternal DNA takes place in meiosis II
C. Separation of whole chromosomes occur during meiosis II
D. Only one round of DNA replication occurs during two successive phases of meiosis
E. None of the above

2. Which of the following tissues is paired with the appropriate primary germ cell layer of origin?
A. Endometrium—mesoderm
B. Pineal gland—ectoderm
C. Tongue epithelium—mesoderm
D. All the above
E. None of the above

3. Which of the following structures do not arise from the Wolffian ducts?
A. Epididymis
B. Epoöphoron
C. Gartner’s duct
D. Paroöphoron
E. Round ligament
4.Which of the following best describes the process of compaction?
A It leads to the formation of the trophoblast
B It leads to the formation of the cytotrophoblast
C It is a reaction of the chromosomes during meiosis
D It leads to the formation of the morula after the 16 cell stage
E It refers to the reaction of the sperm head on penetration of the ovum

5.The cells of which structure form the chorionic villi?


A Neural crest
B Endoderm
C Primitive streak
D Hypoblast
E Extraembryonic mesoderm

6.Which of the following is not a derivative of the vitelline vein?


A Lower inferior vena cava
B Inferior mesenteric vein
C Superior mesenteric vein
D Portal vein
E Hepatic vein
7. “a neonate is admitted to the hospital at 14 days of life with the complaints of failure to thrive,
tachypnoea and difficulty in feeding. He is diagnosed as
having a circulatory defect.”
administration of prostaglandin antagonists soon after birth can be used for therapeutic closure
of which patent structure of foetal origin?
A. Ductus venosus
B. Foramen ovale
C. Ductus arteriosus
D. Fossa ovalis
E. Ligamentum venosum

8.Which of the following tissues is paired with the appropriate primary germ cell layer of
origin?
A. Endometrium—mesoderm
B. Pineal gland—ectoderm
C. Tongue epithelium—mesoderm
D. All the above
E. None of the above

9. Which of the following is true concerning the development of the genital system in a female?
A. The paramesonephric ducts develop from coelomic epithelium on the urogenital ridge
B. The sex of the embryo is determined at the 7th week of development
C. Male embryos have only the mesonephric duct whereas the female embryos have only the
paramesonephric ducts
D. Mitosis in oogonia is not completed by the end of the first year of life
E. The ovary develops in the medulla of the primitive Gonad

10. Which of the following circulatory changes occur at the time of birth?
A. A 20-fold increase in lung blood flow
B. A rise in right atrial pressure
C. Anatomical closure of the ductus arteriosus
D. Flap closure of the foramen ovale
E. Anatomical closure of the ductus venosus
1. Which of the following is true regarding meiosis?
A.Meiosis II is a reductional division
B. Exchange of paternal and maternal DNA takes place in meiosis II
C. Separation of whole chromosomes occur during meiosis II
D. Only one round of DNA replication occurs during two successive phases of meiosis
E.None of the above
Answer D
2. Which of the following tissues is paired with the appropriate primary germ cell layer of origin?
A.Endometrium—mesoderm
B. Pineal gland—ectoderm
C. Tongue epithelium—mesoderm
D. All the above
E.None of the above
Answer B
3. Which of the following structures do not arise from the Wolffian ducts?
A. Epididymis
B. Epoöphoron
C. Gartner’s duct
D.Paroöphoron
E.Round ligament
Answer E
4.Which of the following best describes the process of compaction?
A It leads to the formation of the trophoblast
B It leads to the formation of the cytotrophoblast
C It is a reaction of the chromosomes during meiosis
D It leads to the formation of the morula after the 16 cell stage
E It refers to the reaction of the sperm head on penetration of the ovum
Answer D It leads to the formation of the morula after the 16 cell stage
Compaction refers to the stage of cell division when the cells flatten out and it becomes
impossible to determine cell outlines. This occurs between the 16 and 32 cell stage when the
embryo becomes a morula. The reaction of the sperm with the ovum is known as capacitation.

5.The cells of which structure form the chorionic villi?


A Neural crest
B Endoderm
C Primitive streak
D Hypoblast
E Extraembryonic mesoderm
Answer E Extraembryonic mesoderm
It is the cells of the extraembryonic mesoderm that develop into the chorionic villi. The neural
crest gives rise to the nervous system and melanocytes. Endoderm is one of the germ cell
layers and produces the gastrointestinal tract, respiratory tract and endocrine organs. The
primitive streak develops by the end of the second week in the bilaminar embryonic disc and
functions to determine symmetry of the developing embryo. The hypoblast is a part of the inner
cell mass and lies beneath the epiblast. It gives rise to extraembryonic endoderm.

6. Which of the following is not a derivative of the vitelline vein?


A Lower inferior vena cava
B Inferior mesenteric vein
C Superior mesenteric vein
D Portal vein
E Hepatic vein
Answer B Inferior mesenteric vein
The vitelline veins take blood away from the yolk sac. The vitelline veins give rise to the hepatic
veins, the inferior part of the inferior vena cava, the superior mesenteric vein and the portal
vein. The inferior mesenteric vein is not a derivative of the vitelline vein.

7. “a neonate is admitted to the hospital at 14 days of life with the complaints of failure to thrive,
tachypnoea and difficulty in feeding. He is diagnosed as
having a circulatory defect.”
administration of prostaglandin antagonists soon after birth can be used for therapeutic closure
of which patent structure of foetal origin?
A. Ductus venosus
B. Foramen ovale
C. Ductus arteriosus
D.Fossa ovalis
E.Ligamentum venosum
Answer C

8.Which of the following tissues is paired with the appropriate primary germ cell layer of
origin?
A. Endometrium—mesoderm
B. Pineal gland—ectoderm
C. Tongue epithelium—mesoderm
D. All the above
E. None of the above
Answer B

9. Which of the following is true concerning the development of the genital system in a female?
A.The paramesonephric ducts develop from coelomic epithelium on the urogenital ridge
B. The sex of the embryo is determined at the 7th week of development
C.Male embryos have only the mesonephric duct whereas the female embryos have only the
paramesonephric ducts
D. Mitosis in oogonia is not completed by the end of the first year of life
E.The ovary develops in the medulla of the primitive Gonad
Answer A
10. Which of the following circulatory changes occur at the time of birth?
A.A 20-fold increase in lung blood flow
B. A rise in right atrial pressure
C. Anatomical closure of the ductus arteriosus
D. Flap closure of the foramen ovale
E.Anatomical closure of the ductus venosus
Answer D
9. Herpes family does not include which of the following?

c.papilloma virus
1. Which of the following micro-organisms does not cause latent infection?
A. Cytomegalovirus (CMV)
B. Chlamydia trachomatis
C. Hepatitis A
D. Mycobacterium tuberculosis
E. Varicella zoster virus

2.A 50-year-old woman has ongoing pelvic pain; she has had a coil in situ for the last 8 years.
She has a pelvic mass; histological sampling of the mass at laparoscopy shows a suppurative
and granulomatous inflammatory process with the presence of sulphur granules.
Which is the most likely causative agent?
A Actinomyces israelii
B Chlamydia trachomatis
C Gardnerella vaginalis
D Neisseria gonorrhoeae
E Neisseria meningitidis

3.A florist presents with a subcutaneous lesion on the hand, which she thinks resulted from a
jab wound she received while she was making a sphagnum moss-wire frame for a floral wreath.
The nodule has ulcerated and not healed despite use of antibacterial cream, and a new nodule
is forming above the original lesion. What is most likely to be an appropriate treatment for this
infection?
(A) Oral itraconazole or potassium iodide
(B) Miconazole cream
(C) Cortisone cream
(D) Oral griseofulvin
(E) Penicillin

4. The germination of tetanus spores in a wound is inhibited by which of the following?


A. Injection of anti-toxin
B. Injection of toxoid
C. Tissue trauma
D. Reduced blood supply
E. Decreased oxygen supply

5. Which of the following is not true regarding Bacteroides?


A. Anaerobic
B. Gram-positive
C. Non-spore-forming
D. Sensitive to metronidazole
E. A cause of bacterial vaginosis

6. Which of the following diseases are caused by herpes simplex virus?


A. Cervical warts
B. Acute gingivostomatitis
C. Shingles
D. All the above
E. None of the above

7. Which of the following hepatitis viruses is RNa virus?


A. HAV
B. HCV
C. HDV
D. All the above
E. None of the above

8. Which of the following is the causative agent of Kaposi’s sarcoma?


A. HIV
B. Human herpes virus 4
C. Human herpes virus 8
D. None of the above
E. All of the above

9. Which of the following predispose to microbial invasion?


A. Ciliary dyskinesia
B. Cystic fibrosis
C. Neutrophil deficiency
D. All the above
E. None of the above

10. Which of the following is not a notifiable disease?


A. Rubella
B. Chicken pox
C. Food poisoning
D. Measles
E. Meningococcal meningitis
1. Which of the following micro-organisms does not cause latent infection?
A. Cytomegalovirus (CMV)
B. Chlamydia trachomatis
C. Hepatitis A
D. Mycobacterium tuberculosis
E. Varicella zoster virus
Answer C

2. A 50-year-old woman has ongoing pelvic pain; she has had a coil in situ for the last 8 years.
She has a pelvic mass; histological sampling of the mass at laparoscopy shows a suppurative
and granulomatous inflammatory process with the presence of sulphur granules.
Which is the most likely causative agent?
A Actinomyces israelii
B Chlamydia trachomatis
C Gardnerella vaginalis
D Neisseria gonorrhoeae
E Neisseria meningitidis
Answer A Actinomyces israelii
The Gram-positive bacteria Actinomyces israelii is a commensal of the colon, mouth and
vagina. It is the commonest cause of actinomycosis, a chronic, suppurative and granulomatous
inflammatory infection. The majority of cases of actinomycosis affect the cervicofacial area,
classically presenting as painless facial lumps; however, thoracic, abdominal and pelvic forms
do occasionally occur. Diagnosis of pelvic actinomycosis is usually made from histological
samples taken during surgery and has been associated with intrauterine contraceptive devices
which have been in situ for long periods of time. The presence of sulphur granules is
characteristic of actinomyces infection.

3. A florist presents with a subcutaneous lesion on the hand, which she thinks resulted from a
jab wound she received while she was making a sphagnum moss-wire frame for a floral wreath.
The nodule has ulcerated and not healed despite use of antibacterial cream, and a new nodule
is forming above the original lesion. What is most likely to be an appropriate treatment for this
infection?
(A) Oral itraconazole or potassium iodide
(B) Miconazole cream
(C) Cortisone cream
(D) Oral griseofulvin
(E) Penicillin
The answer is A. This is a classic case of lymphocutaneous sporotrichosis in which a gardener
or florist is infected via a puncture wound. The drug of choice is either itraconazole or
potassium iodide (administered orally in milk). Topical antifungals are not effective, and the
cortisone cream would probably enhance the spread of the disease. Griseofulvin localizes in
the keratinized tissues and would not halt the subcutaneous spread of this infection. Penicillin
would have no effect because Sporothrix is not a bacterium.
4. The germination of tetanus spores in a wound is inhibited by which of the following?
A. Injection of anti-toxin
B. Injection of toxoid
C. Tissue trauma
D. Reduced blood supply
E. Decreased oxygen supply
Answer B

5. Which of the following is not true regarding Bacteroides?


A. Anaerobic
B. Gram-positive
C. Non-spore-forming
D. Sensitive to metronidazole
E. A cause of bacterial vaginosis
Answer B

6. Which of the following diseases are caused by herpes simplex virus?


A. Cervical warts
B. Acute gingivostomatitis
C. Shingles
D. All the above
E. None of the above
StudyMRCOG
MRCOGPart 1
Pathology
Dr Chitra
1. Type of collagen found in basement membranes is
a.Type 1
b.Type 2
c.Type 3
d.Type 4
e.Type 5
Answer d- type 4 found in basement membrane
type 2 found in cartilage & vitreous humour
type 3 found in granulation tissue,embryonic tissue ,uterus & keloids.

2.Features of DIC are all ,except


a.Reduced platelets
b.Reduced fibrinogen
c.Increased FDPs
d.Increased PT & APTT
e.Increased fibrinogen
Answer e-increased fibrinogen
In DIC fibrionogen is decreased.

3.Which is the right pair?


a.α-feto protein – choriocarcinoma
b.β-HCG –yolk sac tumour
c.CEA- thyroid follicular cells
d.Ca-125—epithelial ovarian carcinoma
e.calcitonin- colonic cvarcinoma

Answer d. Ca -125-epithelial ovarian carcinoma


Other correct pairs are
α fetoprotein-yolk sac tumour
β HCG– choriocarcinoma
CEA—colonic carcinoma
calcitonin—thyroid follicular cells

4.Increase in number of cells in tissues is called


a.Hyperplasia
b.Hypoplasia
c.Metaplasia
d.Atrophy
e.hypertrophy

Answer a—hyperplasia is increase in number of cells.


hypertrophy is increase in cell size
atrophy-decrease in size of cell
hypoplasia-decrease in number of cells
Metaplasia-change of mature cells from one type to another

5.A neoplasm of the female genital tract occurring in an 18 year old girl whose mother was
treated with Diethylstilbesterol during pregnancy is likely to be
a. Brenner cell tumour of ovary
b. Teratome of ovary
c. Sarcoma Botryoides of vagina
d. Clear cell carcinoma of vagina
e. Squamous cell carcinoma of vulva
Answer d- clear cell carcinoma of vagina

6.If choriocarcinoma metastasizes ,it spreads predominantly by which route?


a.Direct invasion
b.Hematogenous
c.Lymphatic
d.Surface implantation
e.transcoelomic
Answer b—hematogenous

7.Which of the following clotting factors remain unchanged in normal pregnancy?


a.Factor vii
b.Factors xi & xiii
c.Factors ii,v & ix
d.Factors vii,viii,x & xi
e.Factors vii,x,xi & xiii

Answer b—factors xi & xiii

8.Wound healing by secondary intention takes place


a.When there is irreparable skin loss
b.When the wound becomes infected
c.When the wound does not break apart
d.When the wound edges are brought together
e.All of the above

Answer B—when the wound becomes infected

9.A 70 years old lady,presented to her GP with c/o progressive abdominal distension
,shortness of breath & decreased appetite.Also there was h/o weight loss. The GP referred
her to the oncology department & ordered Ca-125 levels,which turned out to be 500u/ml.What
is the most likely diagnosis in this case?
a.Gastric cancer
b.Colorectal cancer
c.Hepatocellular cancer
d.Pancreatic cancer
e.Primary peritoneal cancer

Answer E—primary peritoneal cancer

10. Which tissue is not capable of regeneration?


a.Epidermis
b.Bone marrow
c.Myocardium
d.Liver
e.skin

Answer C-myocardium
1.A 27-year-old nulliparous woman and her husband have a series of routine investigations to
investigate primary subfertility. She has a hysterosalpingogram which shows she has a
bicornuate uterus.
Which obstetric phenomenon is of increased prevalence in women with a bicornuate uterus?
A Breech presentation
B Stillbirth
C Postpartum haemorrhage
D Placenta praevia
E Placenta accreta

2. Which of the following is characteristic of the cellular changes seen in dysplasia?


A Absence of mitotic figures on microscopy
B Decreased mitotic activity
C Hyperchromatism
D Irreversibility
E Uniformity in cell shape

3.Near the external os of the cervix, what is found as a normal transition from columnar
epithelium?
(A) keratinized epithelium
(B) squamous epithelium
(C) transitional epithelium
(D) cuboidal epithelium
(E) cervical erosion

4. A 44-year-old G5P5005 patient who is currently using oral contraceptive pills to control
menorrhagia had a hysterectomy for uterine enlargement. You suspect adenomyosis by
history. Which histological description supports the diagnosis of adenomyosis?
(A) the metaplastic change of glandular epithelium to muscle fibers in the uterus
(B) the same pattern and location as endometriosis
(C) the presence of endometrial glands and stroma deep within uterine muscle
(D) a premalignant change of the endometrium
(E) a premalignant change of the uterine muscle

5. The cells in the layers surrounding each oocyte produce ovarian hormones. Which of the
following is the correct order of cell layers surrounding an ovarian follicle from the oocyte
outward?
(A) zona pellucida, granulosa, theca interna
(B) granulosa, theca interna, zona pellucida
(C) theca interna, zona pellucida, granulosa
(D) theca interna, granulosa, zona pellucida
(E) zona pellucida, theca interna, granulosa

6. Histologically, the presence of which of the following would determine that an ovarian
teratoma is malignant?
(A) squamous cells
(B) all three germ cell lines
(C) immature fetal-like cells
(D) neural ectoderm
(E) an ovarian capsule

7. Chemical mediators concerned in production of an inflammatory response include which of


the following?
A. Globulin permeability factor
B. Bradykinin
C. 5-Hydroxytryptamine
D. All the above
E. None of the above

8. Which of the following pathogens is commonly isolated from intra-abdominal pus?


A. Actinomyces
B. Bacillus
C. Clostridia
D. None of the above
E. All the above
9. Which of the following tissues is not capable of cellular regeneration?
A. Bone marrow
B. Epidermis
C. Liver
D. Myocardium
E. skin

10. Which obstetric complication has an increased prevalence in women with bicornuate
uterus?
A. Breech presentation
B. Postpartum haemorrhage
C. Placenta accreta
D. Placenta praevia
E. Spontaneous miscarriage
1. A 27-year-old nulliparous woman and her husband have a series of routine investigations to
investigate primary subfertility. She has a hysterosalpingogram which shows she has a
bicornuate uterus.
Which obstetric phenomenon is of increased prevalence in women with a bicornuate uterus?
A Breech presentation
B Stillbirth
C Postpartum haemorrhage
D Placenta praevia
E Placenta accreta

Answer A Breech presentation


A bicornuate uterus has an incidence of 0.1–0.5%. It is caused when the müllerian
(paramesonephric) ducts incompletely fuse during embryonic development. This results in two
separate, but communicating, endometrial cavities. The extent to which there is incomplete
fusion of the müllerian ducts can result in varying degrees of abnormality. This varies from
uterus didelphus where there are two entirely separate uterine cavities and a septate uterus
where there is essentially a single uterine cavity interrupted by a thin septum. The presence of
a bicornuate uterus is associated with recurrent miscarriage, breech presentation and preterm
delivery. Figure 13.2 shows structural abnormalities of the uterus.
Royal College of Obstetricians and Gynaecologists. Query Bank: Recurrent miscarriage and
bicornuate uterus. London: RCOG, 2010. http://www.rcog.org.uk/womens-health/clinical-
guidance/recurrent-miscarriage-and-bicornuate-uterus-query-bank
2. Which of the following is characteristic of the cellular changes seen in dysplasia?
A Absence of mitotic figures on microscopy
B Decreased mitotic activity
C Hyperchromatism
D Irreversibility
E Uniformity in cell shape

Answer C Hyperchromatism
Dysplasia is a term used to describe abnormal development of immature cells in a tissue,
whereby there are abnormalities in the cellular architecture and appearance. Dysplastic cells
can be thought of as showing some of cellular changes that occur in cancer cells. These
atypical features infer malignant potential although not all dysplastic tissue will go on to
become malignant. In dysplastic cells a series of visible characteristic changes occur which
include:
• Increased mitotic activity
• Hyperchromatism: prominent cell nucleus due to increased chromatin
• Nuclear pleomorphism: abnormalities in the shape and size of the nucleus
• Anisocytosis: increased cell size
• Poikilocytosis: unusually shaped cells
Dysplasia may be reversible if in its early stages, especially if any causative stimulus is
removed. A potentially reversible form of dysplasia is present in CIN I, where dysplastic
changes are only seen in the basal third of the squamous epithelium; in these cases there is
high likelihood of reversible changes and monitoring may be appropriate.

3. Near the external os of the cervix, what is found as a normal transition from columnar
epithelium?
(A) keratinized epithelium
(B) squamous epithelium
(C) transitional epithelium
(D) cuboidal epithelium
(E) cervical erosion

Answer (B) The cervix is covered with glandular epithelium in childhood. The cervical
epithelium will undergo change (metaplasia) as it is exposed to estrogen. The columnar
epithelium is replaced by squamous epithelium. The area that changes is called the
transformation zone, and the leading edge of the area of change is the squamocolumnar
junction. Squamous changes are thought to begin at the squamocolumnar junction where
active metaplasia is occurring. This area must be sampled by a Pap smear, and it must be
completely seen with a colposcope, or biopsied during diagnostic procedures to evaluate
abnormal Pap smears.

4. A 44-year-old G5P5005 patient who is currently using oral contraceptive pills to control
menorrhagia had a hysterectomy for uterine enlargement. You suspect adenomyosis by
history. Which histological description supports the diagnosis of adenomyosis?
(A) the metaplastic change of glandular epithelium to muscle fibers in the uterus
(B) the same pattern and location as endometriosis
(C) the presence of endometrial glands and stroma deep within uterine muscle
(D) a premalignant change of the endometrium
(E) a premalignant change of the uterine muscle

Answer (C) Adenomyosis is a condition in which endometrial glands and stroma are found
within the myometrium on histologic examination. These structures must be one or more low-
power microscopic fields below the surface. It is not malignant. Endometrial glands do not
undergo metaplasia to muscle, nor does muscle undergo metaplasia to glands. Endometriosis
is the term that refers to ectopic endometrium in any location outside the uterus. Adenomyosis
used to be referred to as endometriosis interna.

5. The cells in the layers surrounding each oocyte produce ovarian hormones. Which of the
following is the correct order of cell layers surrounding an ovarian follicle from the oocyte
outward?
(A) zona pellucida, granulosa, theca interna
(B) granulosa, theca interna, zona pellucida
(C) theca interna, zona pellucida, granulosa
(D) theca interna, granulosa, zona pellucida
(E) zona pellucida, theca interna, granulosa

Answer (A) The oocyte is surrounded by the zona pellucida, the granulosa, and the theca.
During each menstrual cycle, the follicles selected for ovulation grow until an egg is ovulated by
erupting through the surface of the ovary surrounded by some of the follicle cells. If the early
eggs are not surrounded by follicular cells to form primordial follicles, they resorb.

6. Histologically, the presence of which of the following would determine that an ovarian
teratoma is malignant?
(A) squamous cells
(B) all three germ cell lines
(C) immature fetal-like cells
(D) neural ectoderm
(E) an ovarian capsule

Answer (C) Immature teratomas are malignant, containing embryonic-like tissues. Mature
teratomas may contain all three germ lines but they may or may not be malignant. Mature
teratomas can be malignant, usually because they have malignant elements of mature skin
(squamous epithelium) and neural ectoderm. The neural tissue is most useful in grading the
virulence of the tumor (those with large areas of neuroblast are most virulent).

7. Chemical mediators concerned in production of an inflammatory response include which of


the following?
A. Globulin permeability factor
B. Bradykinin
C. 5-Hydroxytryptamine
D. All the above
E. None of the above

Answer D

8. Which of the following pathogens is commonly isolated from intra-abdominal pus?


A. Actinomyces
B. Bacillus
C. Clostridia
D. None of the above
E. All the above

Answer C

9. Which of the following tissues is not capable of cellular regeneration?


A. Bone marrow
B. Epidermis
C. Liver
D. Myocardium
E. skin

Answer D

10. Which obstetric complication has an increased prevalence in women with bicornuate
uterus?
A. Breech presentation
B. Postpartum haemorrhage
C. Placenta accreta
D. Placenta praevia
E. Spontaneous miscarriage

Answer A
1.A 31-year-old nulliparous woman has heavy bleeding at 8 weeks’ gestation. An early
pregnancy scan is suggestive of a molar pregnancy, and no fetus is observed. What is the
typical genotype of a complete molar pregnancy?
A 45 XO
B 46 XX
C 46 XXX
D 69 XXY
E 92 XXXY

2.A 65-year-old patient presents with a vulvar lesion. The pathology report of the vulvar biopsy
is returned with the following description: There is hyperplasia of keratinocytes in the prickle
cell layer (stratum spinosum) thickening the epidermis. This is descriptive of which of the
following?
(A) Atrophic vulvitis
(B) Syphilitic ulceration
(C) Acanthosis
(D) Lichen sclerosus
(E) Parakeratosis

3. Which of the following is not true regarding polymerase chain reaction?


A. DNA or RNA can be used as the template
B. Helps in diagnosis of infection
C. In diagnostic PCR,the exact sequence at both ends of the target region must be known
D. Polymorphisms in the viral genome may result in amplification failure
E. Takes several days to complete

4.In addition to mast cells, which of the following cells produces histamine?
A Basophils
B Erythrocytes
C Macrophages
D Monocytes
E Neutrophils

5.On a cytologic specimen, which of the following findings would be most suspicious of herpes
virus infection?
(A) intranuclear inclusion bodies
(B) intracytoplasmic inclusions
(C) copious glassy cytoplasm
(D) Donovan bodies
(E) multiple round nucleoli

6. Which of the following is not a predisposing factor for atherosclerosis?


A. Cigarette smoking
B. Diabetes mellitus
C. Hormone replacement therapy
D. Hypertriglyceridemia
E. Systemic arterial hypertension

7.Within what timeframe from injury do macrophages replace neutrophils in cutaneous wound
healing?
A 1–2 hours
B 6–12 hours
C 18–24 hours
D 48–92 hours
E 7–10 days

8.A patient has a screening mammograph that shows a lesion that is high risk for carcinoma.
While waiting for her biopsy to be scheduled she has done some reading on the web regarding
breast cancer.
She is confused by the number of different types of breast cancer and asks which is the most
common pathologic type of breast cancer?
(A) ductal
(B) lobular
(C) Paget’s
(D) inflammatory
(E) adenoid cystic
9. Which of the following is cause of generalised lymphadenopathy (lAP)?
A. HIV seroconversion illness
B. Q fever
C. Syphilis
D. Toxoplasma gondii
E. All the above

10. A 55-year-old woman is admitted to the emergency department 1 week after a work-related
crush injury. On physical examination, she is febrile and appears dehydrated. After
catheterisation, she passes a small amount of very dark urine. The urine dipstick test for blood
is positive but no red blood cells are seen on microscopy. Her serum biochemistry shows:
Creatinine = 120 μmol/L
Serum potassium = 5.7 mmol/L
Creatinine kinase = > 50,000 U/L
Which of the following is the most likely diagnosis?
A Poststreptococcal glomerulonephritis
B Renal infarction
C Renal papillary necrosis
D Rhabdomyolysis
E Ureteral lithiasis
1. A 31-year-old nulliparous woman has heavy bleeding at 8 weeks’ gestation. An early
pregnancy scan is suggestive of a molar pregnancy, and no fetus is observed. What is the
typical genotype of a complete molar pregnancy?
A 45 XO
B 46 XX
C 46 XXX
D 69 XXY
E 92 XXXY

Answer B 46 XX
Gestational trophoblastic disease describes a variety of conditions ranging from complete and
partial molar pregnancies to malignant choriocarcinoma. Histologically there is cystic swelling
of chorionic villi.
A complete molar pregnancy is usually caused by a single sperm combining with an egg which
is devoid of DNA. The genotype is usually diploid 46 XX, as a result of mitosis of the fertilising
sperm. Occasionally the genotype is 46 XY.A partial molar pregnancy is usually caused by two
sperm or one which duplicates and has a triploid genotype 69 XXY, or quadraploid XXXY.
Gestational trophoblasticdisease is a premalignant condition. Overall approximately 1–2% of
hydatiform pregnancies develop into choriocarcinoma and complete moles are more likely
to do so. They are diagnosed histologically by cystic swelling of chorionic villi and
marked trophoblastic proliferation.
Royal College of Obstetricians and Gynaecologists. The Management of Gestational
Trophoblastic Disease.
Green-top Guideline 38. London: RCOG, 2010.

2. A 65-year-old patient presents with a vulvar lesion. The pathology report of the vulvar biopsy
is returned with the following description: There is hyperplasia of keratinocytes in the prickle
cell layer (stratum spinosum) thickening the epidermis. This is descriptive of which of the
following?
(A) Atrophic vulvitis
(B) Syphilitic ulceration
(C) Acanthosis
(D) Lichen sclerosus
(E) Parakeratosis

Answer (C) Acanthosis is found with syphilis, lichen planus, venereal warts, and cancer, as well
as other conditions. Clinically, it refers to a hyperplasia of keratinocytes, causing a thickening of
the prickle layer of the epidermis, which clinically appears as a diffusely thickened or localized
plaque. Thickening of the superficial horny layer of the skin is called hyperkeratosis. The
presence of nucleated surface cells is called parakeratosis. Thinning or atrophy means fewer
cells and cell layers present.
Ulceration means there is absence of epithelium.

3. Which of the following is not true regarding polymerase chain reaction?


A. DNA or RNA can be used as the template
B. Helps in diagnosis of infection
C. In diagnostic PCR,the exact sequence at both ends of the target region must be known
D. Polymorphisms in the viral genome may result in amplification failure
E. Takes several days to complete

Answer E

4. In addition to mast cells, which of the following cells produces histamine?


A Basophils
B Erythrocytes
C Macrophages
D Monocytes
E Neutrophils

Answer A Basophils
Histamine (5-hydroxytryptamine) is a significant contributor to the immediate response in acute
inflammation and is a vasoactive amine. It is predominantly produced by local mast cells, but
also by basophils and platelets. Histamine is typically stored in mast cell granules and released
during a process called degranulation following mast cell activation. This activation may occur
in response to stimuli, such as mast cell antibody binding, following complement activation and
in direct response to injury. The main action of histamine is vascular dilatation,
however it is also involved in increasing vascular permeability.

5. On a cytologic specimen, which of the following findings would be most suspicious of herpes
virus infection?
(A) intranuclear inclusion bodies
(B) intracytoplasmic inclusions
(C) copious glassy cytoplasm
(D) Donovan bodies
(E) multiple round nucleoli

Answer (A) Intranuclear inclusion bodies, irregular nucleoli, and multinuclei are characteristic of
herpes simplex virus. These findings are sometimes detected by biopsy or Tzanck staining of a
cytologic smear, or sometimes with routine Pap smear (50% sensitivity). Donovan bodies are
found with granuloma inguinale and sulfur granules with actinomyces.

6. Which of the following is not a predisposing factor for atherosclerosis?


A.Cigarette smoking
B. Diabetes mellitus
C. Hormone replacement therapy
D. Hypertriglyceridemia
E.Systemic arterial hypertension
Answer C
7.Within what timeframe from injury do macrophages replace neutrophils in cutaneous wound
healing?
A 1–2 hours
B 6–12 hours
C 18–24 hours
D 48–92 hours
E 7–10 days

Answer D 48–92 hours


The process of cutaneous wound healing occurs with initial inflammation, followed by
proliferation and then maturation. Healing by primary intention follows surgical incision closed
by suturing. The initial surgical incision causes platelets to rapidly gather to form a clot which
instigates the inflammatory response. Healing by secondary intention occurs when there is a
traumatic wound causing large loss of cells and tissues. Neutrophils appear with 24 hours of
the insult. Fibroblasts proliferate in the first 24–72 hours forming granulation tissue which fills
the wound within 1 week. Neutrophils are replaced by macrophages within 48–96 hours.
Leucocyte infiltration and increased vascularity is present for up to 14 days.

8.A patient has a screening mammograph that shows a lesion that is high risk for carcinoma.
While waiting for her biopsy to be scheduled she has done some reading on the web regarding
breast cancer.
She is confused by the number of different types of breast cancer and asks which is the most
common pathologic type of breast cancer?
(A) ductal
(B) lobular
(C) Paget’s
(D) inflammatory
(E) adenoid cystic

Answer (A) The pathologic types of breast cancer are identified by their histologic appearance.
About 75% of breast cancers will arise from ductal epithelium. No other cell type in the breast
accounts for more than 10% of pathologic types. The histologic subtype has less bearing on
prognosis than stage of the cancer at treatment, which is the most important prognostic factor
for survival.

9. Which of the following is cause of generalised lymphadenopathy (lAP)?


A. HIV seroconversion illness
B. Q fever
C. Syphilis
D. Toxoplasma gondii
E. All the above

Answer A
10. A 55-year-old woman is admitted to the emergency department 1 week after a work-related
crush injury. On physical examination, she is febrile and appears dehydrated. After
catheterisation, she passes a small amount of very dark urine. The urine dipstick test for blood
is positive but no red blood cells are seen on microscopy. Her serum biochemistry shows:
Creatinine = 120 μmol/L
Serum potassium = 5.7 mmol/L
Creatinine kinase = > 50,000 U/L
Which of the following is the most likely diagnosis?
A Poststreptococcal glomerulonephritis
B Renal infarction
C Renal papillary necrosis
D Rhabdomyolysis
E Ureteral lithiasis

Answer D Rhabdomyolysis
This woman’s urinalysis shows myoglobinuria, which typically follows significant trauma to
muscle tissue. Although its presence may have minimal sequelae in severe cases, there may
be rhabdomyolysis. Myoglobin is normally renally-excreted, but in excessive amounts it causes
obstruction of the distal tubule and acute renal failure.
In rhabdomyolysis there is excessive release of the intracellular contents of muscle cells
leading to hyperkalaemia and metabolic acidosis. Hypocalaemia may also occur. In severe
cases there may be disseminated intravascular coagulation.
1. Rigors are characteristic feature of which of the following?
A. Acute cholecystitis
B. Acute pancreatitis
C. Acute pyelonephritis
D. Hodgkin’s disease
E. Uretericcalculi

2.A 43-year-old woman was diagnosed at 15 years of age with type 1 diabetes mellitus. Her
disease has been poorly controlled. She develops a non-healing ulcer of her foot at age 35
years. By 40 years of age, she has an increasing serum urea and a urinalysis shows a specific
gravity of 1.012, pH 6.5, 1+ protein, no blood, 1+ glucose, negative leukocyte esterase,
negative nitrite, and no ketones. Which of the following renal diseases is she most likely to
have?
A Crescentic glomerulonephritis
B Hyperplastic arteriolosclerosis
C Nodular glomerulosclerosis
D Papillary necrosis
E Pyelonephritis

3.Which of the following is a risk factor for the development of ovarian cancer?
A Early menopause
B History of breastfeeding
C Nulliparity
D Oral contraceptive use
E Physical activity

4. During the menstrual cycle, the histologic appearance of the endometrium will change
significantly.
During the first half of the menstrual cycle, the endometrium becomes thicker and rebuilds
largely in response to which of the following?
(A) progesterone
(B) follicle-stimulating hormone (FSH)
(C) estrogen
(D) luteinizing hormone (LH)
(E) gonadotropin-releasing hormone (GnRH)

5. Although thought to be primarily a conduit between the ovaries and the uterus, the fallopian
tubes have been found to have a more prominent role in conception. The same cellular lining
that helps in fertility also makes the patient more vulnerable for chronic infection from many of
the sexual infectious diseases. Which of the following best describes the normal lining of the
fallopian tube?
(A) squamous epithelium
(B) transitional epithelium
(C) cuboidal epithelium
(D) columnar epithelium with cilia
(E) fibrous connective tissue

6. An ovary is removed for frozen section pathologic examination. The ovary is enlarged, with
small surface excrescences. Pathologic examination reveals numerous cysts lined by serous
epithelium with six to eight cell layers piled on top of one another to form the cyst walls. The
cells show marked cytologic atypia, and nests of similar cells are present in the ovarian stroma.
Round laminated calcium bodies are also seen. What diagnosis does this histologic description
indicate?
(A) normal proliferative phase follicle
(B) corpus luteum cyst
(C) ovarian endometriosis
(D) borderline ovarian carcinoma
(E) cystadenocarcinoma

7. Which of the following pathologic features is most helpful in distinguishing complete


hydatidiform mole from normal placenta?
(A) trophoblastic proliferation
(B) absence of blood vessels
(C) hydropic degeneration of villi
(D) cellular atypia
(E) sex chromatin positivity
8. Which of the following disease is not associated with HlA-B8?
A. Graves’disease
B. Insulin-dependent diabetes mellitus
C. Multiple sclerosis
D. Myasthenia gravis
E. Sjogren’s syndrome

9. Within what timeframe from injury do macrophages replace neutrophils in case of cutaneous
wound healing?
A. 2–3 hours
B. 6–12 hours
C. 18–24 hours
D. 48–96 hours
E. 8–10 days

10. Apoptosis is characterized by which of the following?


A. Cell swelling
B. Karyorrhexis
C. Release of inflammatory mediators
D. None of the above
E. All the above
1. Rigors are characteristic feature of which of the following?
A. Acute cholecystitis
B. Acute pancreatitis
C. Acute pyelonephritis
D. Hodgkin’s disease
E. Uretericcalculi
Answer C

2. A 43-year-old woman was diagnosed at 15 years of age with type 1 diabetes mellitus. Her
disease has been poorly controlled. She develops a non-healing ulcer of her foot at age 35
years. By 40 years of age, she has an increasing serum urea and a urinalysis shows a specific
gravity of 1.012, pH 6.5, 1+ protein, no blood, 1+ glucose, negative leukocyte esterase,
negative nitrite, and no ketones. Which of the following renal diseases is she most likely to
have?
A Crescentic glomerulonephritis
B Hyperplastic arteriolosclerosis
C Nodular glomerulosclerosis
D Papillary necrosis
E Pyelonephritis
Answer C Nodular glomerulosclerosis
Nodular glomerulosclerosis, also known as diabetic nephropathy, is a common complication of
diabetes mellitus. The disease is characterised by thickening of the glomerulus basement
membrane, mesangial sclerosis and glomerulosclerosis. These features are the sequelae of the
known end-organ damage associated with diabetes mellitus as a consequence of
microvascular disease and chronic hyperglycaemia.
This form of nephropathy is typically diagnosed in individuals with known diabetes mellitus, non-
healing ulcers, proteinuria and retinopathy. The diagnosis is characterised by a progressively
declining glomerular filtration rate, long-standing albuminuria and hypertension. The mainstay of
treatment is improved glycaemic control, management of hypertension (often with the use of
ACE inhibitors), dietary protein restriction and in some cases renal dialysis.

3. Which of the following is a risk factor for the development of ovarian cancer?
A Early menopause
B History of breastfeeding
C Nulliparity
D Oral contraceptive use
E Physical activity
Answer C Nulliparity
There are a series of risk factors for ovarian cancer. The main risk factors for the development
of ovarian cancer are increasing age, the presence of gene mutations such as BRCA and
HNPCC and a family history of the disease. Several additional risk factors relate to ovarian
activity, i.e. ovarian cancer appears to be less common in women who have interrupted
ovulation during their reproductive years. The malignancy is more common in women who have
an early menarche and a late menopause. Women who have had no pregnancies are at a
greater risk of developing ovarian cancer; the more children a woman has had the lower the
risk. Protective factors include those who have used the contraceptive pill and those with
a history of breastfeeding.

4. During the menstrual cycle, the histologic appearance of the endometrium will change
significantly.
During the first half of the menstrual cycle, the endometrium becomes thicker and rebuilds
largely in response to which of the following?
(A) progesterone
(B) follicle-stimulating hormone (FSH)
(C) estrogen
(D) luteinizing hormone (LH)
(E) gonadotropin-releasing hormone (GnRH)
Answer (C) In the first half of the menstrual cycle, the endometrium is influenced by estrogen.
The endometrium develops from its basal layer (basalis). Estrogen makes the lining proliferate
(hence, the proliferative phase). At midcycle, estrogen production continues, but with ovulation,
progesterone is also produced, which causes coiling of the endometrial arteries and compaction
of the endometrium. Progesterone also causes endometrial glands to secrete (hence, secretory
phase). Although it is true that gonadotropins (GnRH, FSH, and LH) serve as signals for this
process, they have little direct effect on endometrium.
In the normal postmenopausal woman, gonadotropins are elevated, but the endometrium does
not change because no estrogen or progesterone is produced (see Figure 2–2).
5. Although thought to be primarily a conduit between the ovaries and the uterus, the fallopian
tubes have been found to have a more prominent role in conception. The same cellular lining
that helps in fertility also makes the patient more vulnerable for chronic infection from many of
the sexual infectious diseases. Which of the following best describes the normal lining of the
fallopian tube?
(A) squamous epithelium
(B) transitional epithelium
(C) cuboidal epithelium
(D) columnar epithelium with cilia
(E) fibrous connective tissue
Answer (D) Each portion of the female genital tract has a characteristic epithelial lining. The
fallopian tube is lined by ciliated columnar epithelium. Many of the tubal cells appear ciliated,
while others are secretory or absorptive. The cilia and mucus facilitate egg transport. The
secretory cells provide additional nutrients to the blastocyst as it works its way to the
endometrial cavity. However, this columnar epithelial also increases the easy of either
Chlamydia or GC being able to establish an infectious process.

6. An ovary is removed for frozen section pathologic examination. The ovary is enlarged, with
small surface excrescences. Pathologic examination reveals numerous cysts lined by serous
epithelium with six to eight cell layers piled on top of one another to form the cyst walls. The
cells show marked cytologic atypia, and nests of similar cells are present in the ovarian stroma.
Round laminated calcium bodies are also seen. What diagnosis does this histologic description
indicate?
(A) normal proliferative phase follicle
(B) corpus luteum cyst
(C) ovarian endometriosis
(D) borderline ovarian carcinoma
(E) cystadenocarcinoma
Answer (E) In serous cystadenocarcinoma, more than
three cell layers of stratification exist in the epithelial cell
lining. The individual cells are atypical, and there is
invasion of the ovarian stroma and/or protrusion from the
capsule (excrescences). A borderline malignant tumor
has three or fewer cells in the lining of the cyst and no
evidence of invasion. It is the thickness of the lining,
invasion, and atypia that makes the diagnosis of ovarian
carcinoma. The calcifications are called psammoma
bodies and are suggestive but not diagnostic of an
ovarian malignancy (see Figure 2–4).
7. Which of the following pathologic features is most helpful in distinguishing complete
hydatidiform mole from normal placenta?
(A) trophoblastic proliferation
(B) absence of blood vessels
(C) hydropic degeneration of villi
(D) cellular atypia
(E) sex chromatin positivity
Answer (B) One of the distinctions between complete mole and normal placenta is the lack of
blood vessels in the moles. In most cases, complete moles are 46, XX with euploidy and are
therefore sex-chromatin positive. One-half of normal placentas are sex-chromatin positive.
Microscopically, moles have edematous villi (hydropic degeneration) and both moles and
placentas have proliferating syncytiocytotrophoblast, although random mix of cyto- and
syncytiotrophoblast is more common in moles. However, some placentas, especially of hydropic
fetuses, can show this also. Cellular atypia is common in both normal and molar placentas.

8. Which of the following disease is not associated with HlA-B8?


A. Graves’disease
B.Insulin-dependent diabetes mellitus
C. Multiple sclerosis
D. Myasthenia gravis
E.Sjogren’s syndrome
Answer C
9. Within what timeframe from injury do macrophages replace neutrophils in case of cutaneous
wound healing?
A. 2–3 hours
B. 6–12 hours
C. 18–24 hours
D. 48–96 hours
E. 8–10 days
Answer D

10. Apoptosis is characterized by which of the following?


A. Cell swelling
B. Karyorrhexis
C. Release of inflammatory mediators
D. None of the above
E. All the above
Answer B
StudyMRCOG
MRCOGPart 1
Pharrmccology
Dr Chitra
1.Atosiban is
A.progesterone agonist
B.progesterone antagonist
C.used to stimulate uterine contractions
D.oxytocin antagonist
E.oxytocin agonist

Ans –d –oxytocin antagonist

2.Use of Lithium in pregnancy is associated with which abnormality in the fetus?


A.Tetralogy of Fallots
B.Dandy Walker syndrome
C.Ebsteins Anatoly
D.Transposition of great arteries
E.Limb reduction deformity

Ans. C— Ebsteins anomaly

3.For OAB ,a new drug called Mirabegron is used.Which is the single mechanism of action for
this drug.
A.SSRI
B.SNRI
C.selective B3 adrenergic agonist
D.selective B1 agonist
E.Selective B2 agonist
Ans. C—selective B3 adrenergic agonist

4.Contraindication for the use of Oxybutynin is


A.Bronchial Asthana
B.Diabetes Mellitus
C.Narrow angle glaucoma
D.Parkinsonism
E.Peptic ulcer
Ans c— narrow angle glaucoma

5.Which antihypertensive causes depression in post natal period?


A.labetalol
B.hydrallazine
C.lisinopril
D.nifedipine
E.Methyldopa
Ans. E—methyldopa
6.Mechanism of action of clomiphene citrate is
A.selective estrogen receptor modulator
B.selective progesterone receptor modulator
C.androgenic steroid
D.estrogenic steroid
E.Gonadotropin analogue

Ans. A— selective estrogen receptor modulator

7.Mechanism of action of Ulipristal acetate.


A.selective progesterone receptor modulator
B.oxytocin antagonist
C.selective estrogen receptor modulator
D.prostagalndin analogue
E.partial progesterone receptor blockage

Ans. A— selective progesterone receptor modulator

8.Which of the following causes Kernicterus?


A. Sulfonamides
B .methotrexate
C.erythromycin
D.penicillin
E.gentamicin

Ans. A - sulfonamides

9.Why is oxytocin not used orally?


A.will be destroyed by gastric juice.
B.causes strong uterine contractions
C.metabolised by liver enzymes
D.causes gastric irritation
E.action is delayed

Ans. A— destroyed by gastric juice

10.Which of the following may reduce anticoagulant action of Warfarin?


A.aspirin
B.diazepam
C.ranitidine
D.ciprofloxacin
E.oral contraceptive pills

Ans. E. Oral contraceptive pills


1. Which of the following antihypertensives are ACe inhibitors?
A. Lisinopril
B. Losartan
C. Propranolol
D. All the above
E. None of the above

2. Which of the following is true regarding aciclovir (acyclovir)?


A. Is effective against cytomegalovirus (CMV)
B. Acts via viral thymidine cycle
C. Stops herpetic neuralgia
D. All the above
E. None of the above

3. Which of the following drugs is most associated with coronary artery spasm?
A) Oxytocin
B) Ergometrine
C) Nifedipine
D) Verapamil
E) Atosiban
4. An important feature of congestive heart failure (CHF) regarding drug action is
(a) Impaired blood flow to the intestine
(B) Increased protein binding of various drugs
(C) Increased volume of distribution
(D) Increased drug elimination
(e) Altered drug kinetics

5. Which subfamily of cytochrome P-450s is responsible for the highest fraction of clinically
important drug interactions resulting from metabolism?
(a) CYP1A
(B) CYP2A
(C) CYP3A
(D) CYP4A
(e) CYP5A

6. You administer to a patient an oral maintenance dose of drug calculated to achieve a steady-
state plasma concentration of 5 mcg/L. After dosing the patient for a time sufficient to reach
steady state, the average plasma concentration of drug is 10 mcg/L.
A decrease in which of the following parameters explains this higher than anticipated plasma
drug concentration?
(a) Bioavailability
(B) Volume of distribution
(C) Clearance
(D) Half-life

7. A 66-year-old woman with a long history of heavy smoking presents to her doctor with
complaints of shortness of breath and chronic coughing that has been present for about 2 years
and has been worsening in frequency. The doctor decides to prescribe a bronchodilator agent
that has minimal cardiac side effects, since the patient also has an extensive cardiac history.
Which medication did the doctor likely prescribe?
(A) Albuterol
(B) Prazosin
(C) Atenolol
(D) Ipratropium
(E) Pseudoephedrine

8. Ephedra (ephedrine) causes increased blood pressure by


(A) Indirect action on cholinergic receptors
(B) Blockade of adrenergic receptors
(C) Stimulation of release of epinephrine
(D) Inhibition of reuptake of catecholamines
(E) Direct action on dopamine receptors
9. What significant side effect of terazosin should the doctor warn a 69-year-old patient about?
(A) Bronchospasm
(B) Postural hypotension
(C) Heart failure
(D) Sedation
(E) Drug abuse

10. Prolonged apnea may occur following the administration of succinylcholine to a patient with
a hereditary deficiency of which of the following enzymes?
(A) Glucose-6-phosphate dehydrogenase
(B) Plasma cholinesterase
(C) Monoamine oxidase
(D) Cytochrome P4503A
(E) Acetylcholinesterase
1. Which of the following antihypertensives are ACe inhibitors?
A. Lisinopril
B. Losartan
C. Propranolol
D. All the above
E. None of the above
Answer A

2. Which of the following is true regarding aciclovir (acyclovir)?


A. Is effective against cytomegalovirus (CMV)
B. Acts via viral thymidine cycle
C. Stops herpetic neuralgia
D. All the above
E. None of the above
Answer B

3. Which of the following drugs is most associated with coronary artery spasm?
A) Oxytocin
B) Ergometrine
C) Nifedipine
D) Verapamil
E) Atosiban
Correct Answer: Ergometrine
Explanation: Ergometrine is a vasoconstrictor and has been used by cardiologists in the past
(as ergonovine/Methergin ®) as a provocation test for coronary artery spasm.

4. An important feature of congestive heart failure (CHF) regarding drug action is


(a) Impaired blood flow to the intestine
(B) Increased protein binding of various drugs
(C) Increased volume of distribution
(D) Increased drug elimination
(e) Altered drug kinetics
the answer is C. Because of the patient’s edema and ascites, the apparent volume of
distribution will be increased, which may require small adjustments in his usual medica-
tion doses. Edematous states do not influence gastrointestinal (GI) blood flow, nor do they
affect drug–protein interactions. Drug elimination may be slowed with congestive heart
failure (CHF) exacerbation, not increased. Drug kinetics are generally not changed by
edematous states.

5. Which subfamily of cytochrome P-450s is responsible for the highest fraction of clinically
important drug interactions resulting from metabolism?
(a) CYP1A
(B) CYP2A
(C) CYP3A
(D) CYP4A
(e) CYP5A
the answer is C. The CYP3A subfamily is responsible for roughly 50% of the total cytochrome
P450 activity present in the liver and is estimated to be responsible for approximately half of
all clinically important untoward drug interactions resulting from metabolism.

6. You administer to a patient an oral maintenance dose of drug calculated to achieve a steady-
state plasma concentration of 5 mcg/L. After dosing the patient for a time sufficient to reach
steady state, the average plasma concentration of drug is 10 mcg/L.
A decrease in which of the following parameters explains this higher than anticipated plasma
drug concentration?
(a) Bioavailability
(B) Volume of distribution
(C) Clearance
(D) Half-life
the answer is C. Steady-state plasma concentration of drug 5 (dose rate)/(clearance).
Thus, a decrease in clearance will increase the plasma drug concentration, whereas
an increase in any of the other three parameters will decrease the steady-state plasma
concentration.
7. A 66-year-old woman with a long history of heavy smoking presents to her doctor with
complaints of shortness of breath and chronic coughing that has been present for about 2 years
and has been worsening in frequency. The doctor decides to prescribe a bronchodilator agent
that has minimal cardiac side effects, since the patient also has an extensive cardiac history.
Which medication did the doctor likely prescribe?
(A) Albuterol
(B) Prazosin
(C) Atenolol
(D) Ipratropium
(E) Pseudoephedrine
The answer is D. Ipratropium bromide is used extensively for chronic obstructive pulmo-
nary disease (COPD), which is the most likely diagnosis in this case. It acts by antagoniz-
ing muscarinic receptors in bronchial smooth muscle, thereby causing bronchodilation.
Albuterol is also used for the treatment of COPD; however, it can cause adverse cardiac
effects such as tachycardia and is not recommended in this case. Prazosin is an α-blocker
used for benign prostatic hypertrophy (BPH). Atenolol is a β-blocker used for hyperten-
sion. Pseudoephedrine is an α-agonist used for nasal congestion.

8. Ephedra (ephedrine) causes increased blood pressure by


(A) Indirect action on cholinergic receptors
(B) Blockade of adrenergic receptors
(C) Stimulation of release of epinephrine
(D) Inhibition of reuptake of catecholamines
(E) Direct action on dopamine receptors
The answer is C. Ephedrine acts indirectly to release norepinephrine from nerve terminals,
causing effects similar to those of catecholamines, including elevated blood pressure.
This potentially dangerous agent has been removed from the OTC market because of an
increasing number of deaths being reported as caused by this agent. An example of an
indirect-acting cholinergic agonist is edrophonium, which is used for diagnosis of myas-
thenia gravis. Some adrenoceptor blockers, such as atenolol, are used for the treatment
of hypertension. Catecholamine reuptake inhibition is a property of some antidepressant
medications. Dopamine receptor agonists are used in the treatment of Parkinson disease.

9. What significant side effect of terazosin should the doctor warn a 69-year-old patient about?
(A) Bronchospasm
(B) Postural hypotension
(C) Heart failure
(D) Sedation
(E) Drug abuse
The answer is B. α1-Adrenoceptor agonists such as terazosin may cause significant
postural hypotension and should be prescribed carefully in the elderly population.
Bronchospasm is a possible side effect of β-blockers. β-Blockers can also produce heart
failure in some patients. Sedation is common with the use of some agents such as pro-
pranolol. Drug abuse can be observed in patients using centrally acting adrenoreceptor
agonists such as amphetamine.

10. Prolonged apnea may occur following the administration of succinylcholine to a patient with
a hereditary deficiency of which of the following enzymes?
(A) Glucose-6-phosphate dehydrogenase
(B) Plasma cholinesterase
(C) Monoamine oxidase
(D) Cytochrome P4503A
(E) Acetylcholinesterase
The answer is B. Plasma cholinesterase is responsible for the rapid inactivation of
succinylcholine.
1. A 45-year-old woman with a long history of alcohol abuse is being treated for cirrhosis-
associated ascites. Her internist decided to give her amiloride, a diuretic helpful in edema
caused by cirrhosis. What common side effect should be monitored in this patient?
(A) Hypernatremia
(B) Hypocalcemia
(C) Hyperphosphatemia
(D) Hypermagnesemia
(E) Hyperkalemia

2. An 87-year-old woman who is taking multiple medications for her “heart disease” is
prescribed gentamicin for diverticulitis. After a few days of taking the antibiotic, she complains of
dizziness and tinnitus. What “heart medication” might she be on?
(A) Spironolactone
(B) Hydrochlorothiazide
(C) Mannitol
(D) Ethacrynic acid
(E) Urea

3. A patient with a long history of cardiovascular disease develops worsening ventricular


arrhythmias. Which of the following drugs is most likely to be the cause of the arrhythmia?
(A) Quinidine
(B) Propanolol
(C) Dobutamine
(D) Methyldopa

4. A 55-year-old woman is admitted to the surgical intensive care unit after having a coronary
artery bypass grafting of four of her coronary vessels. Overnight she develops hypotension, and
her cardiac output, as measured by the Swan-Ganz catheter, is significantly lower than it had
been post surgery. You decide to give her a dose of milrinone. This results in an increase in her
cardiac output. How does this medication work?
(A) It is a cholinergic agonist
(B) It reduces left ventricular filling pressure
(C) It potentiates cardiac phosphodiesterase type 3
(D) It decreases cyclic AMP (cAMP)
(E) It decreases intracellular calcium

5. A 56-year-old retired school teacher with a treated blood pressure of 125/82 mmHg comes in
for a semi-annual exam. You have a set of blood chemistries run and his low-density lipoprotein
cholesterol (LDLc) is 230 and his high-density lipoprotein cholesterol (HDLc) is 54. You place
him on a drug and ask him to return in 1 month. On his return, his LDLc is reduced to 189 but
he complains of a cramping pain in his gastrocnemius in both the legs. The drug most likely to
have caused this adverse effect is which of the following?
(A) Ezetimibe
(B) Rosuvastatin
(C) Hydrochlorothiazide
(D) Niacin
(E) Gemfibrozil

5. Fentanyl is approximately how many times more potent than morphine?


A)2
B) 5
C) 10
D)20
E) 100

6. Which of the following best describes the mechanism of action of Ondansetron?


A) Histamine H1‐receptor agonist
B) Histamine H1‐receptor antagonist
C) Serotonin 5‐HT3 receptor antagonist
D) Dopamine D2 receptor agonist
E) Dopamine D2 receptor antagonist

7. Which of the following is an appropriate treatment for heroin (diamorphine) overdose?


A) Flumazenil
B) Naloxone
C) Protamine
D) Parvolex
E) Octiplex

8.A 33 year old women presents to clinic for infertility investigations. She has a significant
psychiatric history. Her blood tests reveal a raised Prolactin. Which of her medications below is
NOT known to cause this
A) Amitriptyline
B) Zopiclone
C) Chlorpromazine
D) Risperidone
E) Cocodamol

9. Which of the following best describes Ulipristal?


A) Synthetic Progesterone
B) Synthetic Oestragen
C) Selective Progesterone Receptor Modulator
D) Selective Oestragen Receptor Modulator
E) GnRH Antagonist

10. Tamoxifen is associated with an increased risk of which of the following?


A) Osteoporosis in post‐menopausal women
B) Endometrial cancer
C) Gynaecomastia
D) Ovarian cancer
E) Induced lactation
1. A 45-year-old woman with a long history of alcohol abuse is being treated for cirrhosis-
associated ascites. Her internist decided to give her amiloride, a diuretic helpful in edema
caused by cirrhosis. What common side effect should be monitored in this patient?
(A) Hypernatremia
(B) Hypocalcemia
(C) Hyperphosphatemia
(D) Hypermagnesemia
(E) Hyperkalemia
The answer is E. Hyperkalemia, a potentially life-threatening side effect, should be recognized
as a possible result of amiloride use. Hyponatremia, not hypematremia, can be observed with
amiloride. This agent does not affect calcium or phosphorus balance to a significant degree.
Triamterene, another potassium-sparing diuretic, can cause increased urinary excretion of
magnesium; amiloride is not known to produce this effect.

2. An 87-year-old woman who is taking multiple medications for her “heart disease” is
prescribed gentamicin for diverticulitis. After a few days of taking the antibiotic, she complains of
dizziness and tinnitus. What “heart medication” might she be on?
(A) Spironolactone
(B) Hydrochlorothiazide
(C) Mannitol
(D) Ethacrynic acid
(E) Urea
The answer is D. Ototoxicity, as demonstrated by tinnitus and dizziness, is a common side effect
of loop diuretics, especially ethacrynic acid. This effect is magnified when aminoglycoside
antibiotics are added to the regimen. Spironolactone is not associated with tinnitus.
Hydrochlorothiazide can cause gout in susceptible individuals. Mannitol and urea are osmotic
diuretics and are not indicated in patients with heart disease, especially congestive heart failure
(CHF).

3. A patient with a long history of cardiovascular disease develops worsening ventricular


arrhythmias. Which of the following drugs is most likely to be the cause of the arrhythmia?
(A) Quinidine
(B) Propanolol
(C) Dobutamine
(D) Methyldopa
The answer is A. Quinidine is associated with QT interval prolongation and torsade de point
arrhythmias. Propranolol can cause heart block. Dobutamine rarely causes ventricular
arrhythmias.

4. A 55-year-old woman is admitted to the surgical intensive care unit after having a coronary
artery bypass grafting of four of her coronary vessels. Overnight she develops hypotension, and
her cardiac output, as measured by the Swan-Ganz catheter, is significantly lower than it had
been post surgery. You decide to give her a dose of milrinone. This results in an increase in her
cardiac output. How does this medication work?
(A) It is a cholinergic agonist
(B) It reduces left ventricular filling pressure
(C) It potentiates cardiac phosphodiesterase type 3
(D) It decreases cyclic AMP (cAMP)
(E) It decreases intracellular calcium
The answer is B. Milrinone reduces left ventricular filling pressure and thus enhances cardiac
output. It is related to the anticholinergic agent biperiden. Milrinone inhibits cardiac
phosphodiesterase type 3. It increases cyclic AMP (cAMP), and therefore intracellular calcium.

5. A 56-year-old retired school teacher with a treated blood pressure of 125/82 mmHg comes in
for a semi-annual exam. You have a set of blood chemistries run and his low-density lipoprotein
cholesterol (LDLc) is 230 and his high-density lipoprotein cholesterol (HDLc) is 54. You place
him on a drug and ask him to return in 1 month. On his return, his LDLc is reduced to 189 but
he complains of a cramping pain in his gastrocnemius in both the legs. The drug most likely to
have caused this adverse effect is which of the following?
(A) Ezetimibe
(B) Rosuvastatin
(C) Hydrochlorothiazide
(D) Niacin
(E) Gemfibrozil
The answer is B. A common adverse effect of statins like rosuvastatin is muscle pain and
cramping. Ezetimibe reduces LDLc by interfering with absorption of dietary cholesterol.
Hydrochlorothiazide can increase both plasma cholesterol and triglycerides. Niacin can
also reduce LDLc and elevated HDLc. The major action of gemfibrozil is to reduce serum
triglycerides.

5. Fentanyl is approximately how many times more potent than morphine?


A)2
B) 5
C) 10
D)20
E) 100
Correct Answer: 100
Explanation: Fentanyl is approximately 80 to 100 times more potent than morphine

6. Which of the following best describes the mechanism of action of Ondansetron?


A) Histamine H1‐receptor agonist
B) Histamine H1‐receptor antagonist
C) Serotonin 5‐HT3 receptor antagonist
D) Dopamine D2 receptor agonist
E) Dopamine D2 receptor antagonist
Correct Answer: Serotonin 5‐HT3 receptor antagonist
Explanation: Ondansetron is a Serotonin (5‐HT3) receptor antagonist
7.Which of the following is an appropriate treatment for heroin (diamorphine) overdose?
A) Flumazenil
B) Naloxone
C) Protamine
D) Parvolex
E) Octiplex
Correct Answer: Naloxone
Explanation: Opoid overdose is typically treated using naloxone. Doctors should be aware that
Naloxone is short acting. This is particularly important if the patient has respiratory depression
as they may respond to treatment then relapse as the initial dose wears off.
Flumazenil is used for benzodiazepine overdose.
Parvolex (N‐acetyl cysteine) is used for paracetamol overdose.
Protamine can be used for Heparin reversal.
Octiplex can be used for Warfarin reversal

8.A 33 year old women presents to clinic for infertility investigations. She has a significant
psychiatric history. Her blood tests reveal a raised Prolactin. Which of her medications below is
NOT known to cause this
A) Amitriptyline
B) Zopiclone
C) Chlorpromazine
D) Risperidone
E) Cocodamol
Correct Answer: Zopiclone
Explanation: Drug Cause of Hyperprolactinaemia include:
Atypical antipsychotics eg risperidone
Phenothiazines eg chlorpromazine
Butyrophenones eg haloperidol
Thioxanthenes
Metoclopramide
Dopamine synthesis inhibitors eg ‐Methyldopa
Catecholamine depletors eg Reserpine
Opiates eg Codeine
H2 antagonists eg Cimetidine, Ranitidine
Amitriptyline
SSRI's eg Fluoxetine
Calcium channel blockers eg Verapamil
Oestrogens
TRH

9. Which of the following best describes Ulipristal?


A) Synthetic Progesterone
B) Synthetic Oestragen
C) Selective Progesterone Receptor Modulator
D) Selective Oestragen Receptor Modulator
E) GnRH Antagonist
Correct Answer: Selective Progesterone Receptor Modulator
Explanation: Ulipristal is a SPRM (selective progesterone receptor modulator) that is thought to
have its effect by inhibition of ovulation.

10. Tamoxifen is associated with an increased risk of which of the following?


A) Osteoporosis in post‐menopausal women
B) Endometrial cancer
C) Gynaecomastia
D) Ovarian cancer
E) Induced lactation
Correct Answer: Endometrial cancer
Explanation: Tamoxifen is an oestrogen antagonist in breast tissue but it acts as a partial
oestrogen agonist on the endometrium increasing the risk of hyperplasia and has been linked to

endometrial cancer.
Tamoxifen improves bone density in post‐menopausal women.
It is an off license treatment for gynaecomastia.
Reduced lactation is a potential side effect of Tamoxifen.
1.A 17-year-old girl discovers she is pregnant despite taking the oral contraceptive pill. She has
recently been prescribed a new medication by her general practitioner.
Which of the following drugs is most likely to have interacted with the efficacy of her
contraception?
A Carbamazepine
B Cimetidine
C Erythromycin
D Metronidazole
E Sulphamethoxazole

2.Which of the following is a recognised side effect of heparin usage?


A Hirsutism
B Hyperaldosteronism
C Hypokalaemia
D Osteomalacia
E Thrombocytopaenia

3.A 25-year-old nulliparous woman, with a lifelong history of tonic clonic seizures, sees her
neurologist as she wishes to start a family. In addition to her current anticonvulsant therapy
which additional drug is now required?
A A second anticonvulsant
B Ferrous sulphate
C Folic acid
D Low-molecular weight heparin
E Vitamin K

4. A 35-year-old woman, with a history of previous multiple pulmonary embolisms, is now 8


weeks pregnant.
Which is the anticoagulant of choice during her pregnancy?
A Aspirin 300 mg
B Heparin infusion
C Low-molecular weight heparin
D Warfarin
E None of the above

5. A 27-year-old woman is treated for severe bronchitis at 38 weeks’ gestation. Her baby, born
at 41 weeks’ gestation, has neonatal haemolysis.
Which drug taken by the mother for bronchitis is the cause of the baby’s neonatal haemolysis?
A Amoxicillin
B Chloramphenicol
C Co-trimoxazole
D Doxycycline
E Erythromycin
6.A 32-year-old woman suffers a 1500 mL postpartum haemorrhage 15 minutes after delivery.
She is given several drugs to contract the uterus and the bleeding stops. One hour later, she is
found to have blood pressure of 178/110 mmHg. Which drug is most likely to be responsible for
this clinical finding?
A Carboprost
B Ergometrine
C Misoprostol
D Oxytocin
E Ritodrine
7. Atosiban is:
a) Oxytocin receptor blocker
b) Progesterone antagonist
c) Use as tocolytic
d) Both a and c
e) Progesterone agonist
8. After starting Warfarin there is thrombosis in superficial vein. This is due to:
a) Inhibition of factor VII
b) Inhibition of factor X
c) Inhibition of factor VII
d) Inhibition of reduction of vitamin K oxide form
e) Inhibition of protein C and S before inhibition of reductase enzyme
9. Hemorrhage due to heparin can be reversed by:
a) Protamine
b) Vitamin K
c) Factor VII
d) Normal saline
e) Danaparoid

10. Primigravida 30 week 3 days started on antihypertensinve methyldopa 1 month back. Now
she complaint of edema on both legs. What are side effects of methyldopa:
a) Sedation
b) Reduced libido
c) Edema
d) Post partum depression
e) All of above

11. Side effect because of which clonidine is not used today is:
a) Sedation
b) Reduced libido
c) Rebound hypertension
d) Post partum depression
e) All of above
1.A 17-year-old girl discovers she is pregnant despite taking the oral contraceptive pill. She has
recently been prescribed a new medication by her general practitioner.
Which of the following drugs is most likely to have interacted with the efficacy of her
contraception?
A Carbamazepine
B Cimetidine
C Erythromycin
D Metronidazole
E Sulphamethoxazole
ANS- A

2.Which of the following is a recognised side effect of heparin usage?


A Hirsutism
B Hyperaldosteronism
C Hypokalaemia
D Osteomalacia
E Thrombocytopaenia
ANS-E

3.A 25-year-old nulliparous woman, with a lifelong history of tonic clonic seizures, sees her
neurologist as she wishes to start a family. In addition to her current anticonvulsant therapy
which additional drug is now required?
A A second anticonvulsant
B Ferrous sulphate
C Folic acid
D Low-molecular weight heparin
E Vitamin K
ANS-C

4.A 35-year-old woman, with a history of previous multiple pulmonary embolisms, is now 8
weeks pregnant.
Which is the anticoagulant of choice during her pregnancy?
A Aspirin 300 mg
B Heparin infusion
C Low-molecular weight heparin
D Warfarin
E None of the above
ANS-C

5. A 27-year-old woman is treated for severe bronchitis at 38 weeks’ gestation. Her baby, born
at 41 weeks’ gestation, has neonatal haemolysis.
Which drug taken by the mother for bronchitis is the cause of the baby’s neonatal haemolysis?
A Amoxicillin
B Chloramphenicol
C Co-trimoxazole
D Doxycycline
E Erythromycin
ANS-C

6.A 32-year-old woman suffers a 1500 mL postpartum haemorrhage 15 minutes after delivery.
She is given several drugs to contract the uterus and the bleeding stops. One hour later, she is
found to have blood pressure of 178/110 mmHg. Which drug is most likely to be responsible for
this clinical finding?
A Carboprost
B Ergometrine
C Misoprostol
D Oxytocin
E Ritodrine
ANS-B

7. Atosiban is:
a) Oxytocin receptor blocker
b) Progesterone antagonist
c) Use as tocolytic
d) Both a and c
e) Progesterone agonist
Answer: a

8. After starting Warfarin there is thrombosis in superficial vein. This is due to:
a)Inhibition of factor VII
b) Inhibition of factor X
c) Inhibition of factor VII
d) Inhibition of reduction of vitamin K oxide form
e)Inhibition of protein C and S before inhibition of reductase enzyme
Answer: e

9. Hemorrhage due to heparin can be reversed by:


a)Protamine
b) Vitamin K
c) Factor VII
d) Normal saline
e)Danaparoid
Answer: a

10. Primigravida 30 week 3 days started on antihypertensinve methyldopa 1 month back. Now
she complaint of edema on both legs. What are side effects of methyldopa:
a) Sedation
b) Reduced libido
c)Edema
d) Post partum depression
e)All of above
Answer: e

11. Side effect because of which clonidine is not used today is:
a)Sedation
b) Reduced libido
c) Rebound hypertension
d) Post partum depression
e)All of above
Answer: c
StudyMRCOG
MRCOG Part 1
Clinical Management
Compiled by StudyMRCOG Mentor’s team

These questions are complied from various sources/materials for MRCOG/MRCPI


study purpose, STUDYMRCOG doesn't own any credit or doesn't hold any copy rights
on this & used only for educational purpose.
1. Which of the following statement is true about evidence-based medicine?
A. Combines clinical expertise and external evidence
B. Does not involve health economic assessment
C. Is restricted to randomised placebo-controlled trials
D. Is used to cut down waiting lists
E. Tries to rely on subjective measurements of disease outcomes

2.a team wishes to audit their departmental results on the use of anticoagulation in patients
with obstetric thromboembolic disease. What is the most appropriate next step to be taken up
by the team who is undertaking audit in this case?
A. Data analysis
B. Data collection
C. Identify standards
D. Implement change
E. Needs assessment

3. A 55-year-old woman presents to the clinic enquiring about the use of hormone replacement
therapy (HRT). She had a hysterectomy 8 years ago for fibroids. She has no contraindications
or other past medical history, except a strong family history of osteoporosis. Her main
symptoms are hot flushes and vaginal dryness. What would be your first treatment option?
a) A selective estrogen receptor modulator
b) Combined sequential HRT
c) Estrogen-only HRT
d) Oral calcium therapy only
e) Selective serotonin reuptake inhibitor

4. A 17-year-old woman presents to the sexual health clinic with vulval ulceration and difficulty
in passing urine. She is sexually active and has had unprotected intercourse with her new
boyfriend. She takes the combined oral contraceptive pill. What is the most likely diagnosis?
a) Candida albicans
b) Herpes simplex virus
c) Herpes varicella virus
d) Human papillomavirus
e) Syphilis

5. A nulliparous woman presents with spontaneous rupture of membranes at 41 weeks of


gestation. At 18:00h, her cervical dilatation is 3 cm. A further vaginal examination at 22:00h
reveals that her cervical dilatation is still 3 cm. At 02:10h, the fetus is in the occipitoposterior
position and uterine activity is present. What is the most appropriate action?
a) Administer prostaglandin per vaginam
b) Caesarean section
c) Commence intravenous oxytocin
d) Membrane sweep
e) Repeat vaginal examination after 4 hours
6. Which of the following is not a known side effect of Tranexamic acid?
A Allergic skin reactions
B Diarrhoea
C Pulmonary haemorrhage
D Visual disturbances
E Vomiting

7. Which of the following is the site of production of the hormone insulin?


A Adrenal cortex
B Adrenal medulla
C Pancreatic beta cells
D Pancreatic alpha cells
E Pancreatic delta cells

8. A 33 year old women with known stage III cervical cancer presents to A&E with lower
abdominal and unilateral flank pain. From the following list what is the likely diagnosis?
A. Renal stones
B. Ischaemia secondary to Uterine artery obstruction
C. UTI
D. Urethral obstruction
E. Ureteric Obstruction
9. You review a 28 year old patient in the fertility clinic. She has a diagnosis of PCOS. She has
been trying to conceive for 2 years. Her BMI is 26 kg/m2. She is a non‐smoker. She has been
taking Clomiphene and metformin for the past 6 months. What is the next most appropriate
treatment?
A. Gonadotrophins
B. Pulsatile GnRH
C. Dopamine agonist
D. IVF
E. Continue current treatment for further 6 months

10. A 26 year old patient sustains a 3b perineal tear following delivery of her 1st baby. Following
repair she asks you about her prognosis. What is the chance she will be asymptomatic in 12
months time?
A) 10%
B) 30%
C) 50%
D) 70%
E) 90%
1. Which of the following statement is true about evidence-based medicine?
A. Combines clinical expertise and external evidence
B. Does not involve health economic assessment
C. Is restricted to randomised placebo-controlled trials
D. Is used to cut down waiting lists
E. Tries to rely on subjective measurements of disease outcomes
Answer A

2.a team wishes to audit their departmental results on the use of anticoagulation in patients
with obstetric thromboembolic disease. What is the most appropriate next step to be taken up
by the team who is undertaking audit in this case?
A. Data analysis
B. Data collection
C. Identify standards
D. Implement change
E. Needs assessment
Answer C

3. A 55-year-old woman presents to the clinic enquiring about the use of hormone replacement
therapy (HRT). She had a hysterectomy 8 years ago for fibroids. She has no contraindications
or other past medical history, except a strong family history of osteoporosis. Her main
symptoms are hot flushes and vaginal dryness. What would be your first treatment option?
a) A selective estrogen receptor modulator
b) Combined sequential HRT
c) Estrogen-only HRT
d) Oral calcium therapy only
e) Selective serotonin reuptake inhibitor
The answer is estrogen-only HRT. HRT provides the most effective method of treating
climacteric hot flushes and vaginal dryness, as well as postmenopausal osteoporosis.
Estrogen-only HRT is appropriate following a hysterectomy, as the risk of endometrial
carcinoma is not present.

4. A 17-year-old woman presents to the sexual health clinic with vulval ulceration and difficulty
in passing urine. She is sexually active and has had unprotected intercourse with her new
boyfriend. She takes the combined oral contraceptive pill. What is the most likely diagnosis?
a) Candida albicans
b) Herpes simplex virus
c) Herpes varicella virus
d) Human papillomavirus
e) Syphilis
The answer is herpes simplex virus. A painful genital ulcer has developed in a sexually active
patient who has not used barrier contraception. Syphilis is usually associated with the presence
of a painless ulcer. Candida is not a sexually transmitted disease and rarely presents with
ulceration. Human papillomavirus causes warts, and herpes varicella zoster causes chicken
pox and shingles.

5. A nulliparous woman presents with spontaneous rupture of membranes at 41 weeks of


gestation. At 18:00h, her cervical dilatation is 3 cm. A further vaginal examination at 22:00h
reveals that her cervical dilatation is still 3 cm. At 02:10h, the fetus is in the occipitoposterior
position and uterine activity is present. What is the most appropriate action?
a) Administer prostaglandin per vaginam
b) Caesarean section
c) Commence intravenous oxytocin
d) Membrane sweep
e) Repeat vaginal examination after 4 hours
The answer is commence intravenous oxytocin. The membranes have ruptured already so
amniotomy is not required. No progression has been made during the first stage of labour.
Therefore, the patient should be administered intravenous oxytocin.

6. Which of the following is not a known side effect of Tranexamic acid?


A Allergic skin reactions
B Diarrhoea
C Pulmonary haemorrhage
D Visual disturbances
E Vomiting
Answer C
Allergic skin reactions occur in 0.1‐1%.
Colour vision and visual disturbances have been reported but are of unknown frequency.
Diarrhoea and vomiting occur in 1‐10% of patients on tranexamic acid.

7. Which of the following is the site of production of the hormone insulin?


A Adrenal cortex
B Adrenal medulla
C Pancreatic beta cells
D Pancreatic alpha cells
E Pancreatic delta cells
Answer C
Insulin is synthesised in and released from the beta cells of the islets of Langerhans in the
pancreas.

8. A 33 year old women with known stage III cervical cancer presents to A&E with lower
abdominal and unilateral flank pain. From the following list what is the likely diagnosis?
A. Renal stones
B. Ischaemia secondary to Uterine artery obstruction
C. UTI
D. Urethral obstruction
E. Ureteric Obstruction
Answer: Ureteric Obstruction
About one third of patients with stage IIIB cancer will develop ureteric obstruction.

9. You review a 28 year old patient in the fertility clinic. She has a diagnosis of PCOS. She has
been trying to conceive for 2 years. Her BMI is 26 kg/m2. She is a non‐smoker. She has been
taking Clomiphene and metformin for the past 6 months. What is the next most appropriate
treatment?
A. Gonadotrophins
B. Pulsatile GnRH
C. Dopamine agonist
D. IVF
E. Continue current treatment for further 6 months
Correct Answer: Gonadotrophins
Explanation: Clomiphene shouldn't be continued for more than 6 months. The second line
options are Gonadotrophins or tubal drilling.

10. A 26 year old patient sustains a 3b perineal tear following delivery of her 1st baby. Following
repair she asks you about her prognosis. What is the chance she will be asymptomatic in 12
months time?
A) 10%
B) 30%
C) 50%
D) 70%
E) 90%
Answer 70%
Explanation: There is a 60‐80% chance she will be asymptomatic in 12 month
1.a surgical team presented their data demonstrating an increased rate of post-surgical wound
infection following gastro-intestinal surgery compared with published standards from the Royal
College of Surgeons. What is the most appropriate next step to be taken up by the team who is
undertaking audit in this case?
A. Data analysis
B. Data collection
C. Identify standards
D. Implement change
E. Needs assessment

2.at a recent directorate meeting, an obstetrician has been nominated to undertake the next
clinical audit. What is the most appropriate next step for the team undertaking audit in this
case?
A. Data analysis
B. Data collection
C. Needs assessment
D. Identify standards
E. Implement change

3. A 16-year-old woman presents with secondary amenorrhoea. She is healthy, with no past
medical history. Her BMI is 17. What is the most appropriate initial investigation?
a) Bone mineral density scan
b) Dehydroepiandrosterone sulphate (DHEAS) measurement
c) Follicle-stimulating hormone measurement
d) Karyotype
e) Thyroid function test

4. Whilst you are attending to a patient in the antenatal ward, the patient collapses and
becomes unresponsive. You open their airway but they are not breathing. What should you do
next?
a) Commence artificial ventilation
b) Commence cardiac compressions
c) Get help
d) Give a precordial thump
e) Left lateral tilt

5. Polyglactin sutures are used extensively in surgical procedures, particularly to ligate vessels.
What are the key features of polyglactin sutures?
a) Braided, absorbable and synthetic
b) Braided, non-absorbable and synthetic
c) Non-braided, absorbable and natural
d) Non-braided, absorbable and synthetic
e) Non-braided, non-absorbable and natural
6. You are performing a diagnostic laparoscopy on a patient. What is the appropriate distension
pressure upon completion of trocar insertion?
A. 2‐5 mmHg
B. 12‐15 mmHg
C. 20‐25 mmHg
D. 35‐50 mmHg
E. 55‐65 mmHg

7.When consenting someone for laparoscopy you discuss the risk of vascular injury. The
incidence of vascular injury during laparoscopy according to the Green‐top guidelines is?
A) 0.2/1000
B) 0.4/1000
C) 2/1000
D) 4/1000
E) 4/100.

8.A 26 year old women presents for her 12 week scan. She has been pregnant once before but
had a 1st trimester miscarriage. She reports no problems with this pregnancy and has had no
vaginal bleeding or spotting. The scan shows no fetal cardiac activity and a small gestational
sac. What is the likely diagnosis?
A. Recurrent Miscarriage
B. Complete Miscarriage
C. Missed Miscarriage
D. Inevitable Miscarriage
E. Threatened Miscarriage

9. A patient has been seeing you due to itching during pregnancy and you have diagnosed
cholestatic jaundice. What is the most appropriate advice regarding testing LFTs postnatally?
A. No postnatal testing required
B. Test 24 hours postnatally
C. Test 48 hours postnatally
D. Test 7 days postnatally
E. Test 10 days postnatally

10. A 32 year old women who is 25 weeks pregnant presents with vaginal bleeding and
cramping lower abdominal pain. On examination the cervix is closed. Fetal cardiac activity is
noted on ultrasound. What is the likely diagnosis?
A. Antepartum Haemorrhage
B. Threatened Miscarriage
C. Inevitable Miscarriage
D. Septic Miscarriage
E. None of the above
1.a surgical team presented their data demonstrating an increased rate of post-surgical wound
infection following gastro-intestinal surgery compared with published standards from the Royal
College of Surgeons. What is the most appropriate next step to be taken up by the team who is
undertaking audit in this case?
A. Data analysis
B. Data collection
C. Identify standards
D. Implement change
E. Needs assessment
Answer D

2.at a recent directorate meeting, an obstetrician has been nominated to undertake the next
clinical audit. What is the most appropriate next step for the team undertaking audit in this
case?
A.Data analysis
B. Data collection
C. Needs assessment
D. Identify standards
E.Implement change
Answer C
3. A 16-year-old woman presents with secondary amenorrhoea. She is healthy, with no past
medical history. Her BMI is 17. What is the most appropriate initial investigation?
a) Bone mineral density scan
b) Dehydroepiandrosterone sulphate (DHEAS) measurement
c) Follicle-stimulating hormone measurement
d) Karyotype
e) Thyroid function test
The answer is follicle-stimulating hormone measurement.
The likely diagnosis is amenorrhoea caused by weight loss, so follicle-stimulating hormone
measurements would be appropriate. Since she is clinically euthyroid, thyroid function tests
would be of limited value.

4. Whilst you are attending to a patient in the antenatal ward, the patient collapses and
becomes unresponsive. You open their airway but they are not breathing. What should you do
next?
a) Commence artificial ventilation
b) Commence cardiac compressions
c) Get help
d) Give a precordial thump
e) Left lateral tilt
The answer is get help. Basic and Advanced Life Support guidelines highlight the need to get
help if a patient has collapsed and is unresponsive. You should then place the patient in left
lateral tilt and commence cardiac compressions at a rate of 30:2. Artificial ventilation is not
mandatory in the ALS guidelines.

5. Polyglactin sutures are used extensively in surgical procedures, particularly to ligate vessels.
What are the key features of polyglactin sutures?
a) Braided, absorbable and synthetic
b) Braided, non-absorbable and synthetic
c) Non-braided, absorbable and natural
d) Non-braided, absorbable and synthetic
e) Non-braided, non-absorbable and natural
The answer is braided, absorbable and synthetic. Polyglactin sutures are used to ligate pedicles
and close the uterus during a caesarean section. To achieve this, the sutures are braided to
prevent the knots from slipping.

6. You are performing a diagnostic laparoscopy on a patient. What is the appropriate distension
pressure upon completion of trocar insertion?
A. 2‐5 mmHg
B. 12‐15 mmHg
C. 20‐25 mmHg
D. 35‐50 mmHg
E. 55‐65 mmHg
Correct Answer: 12‐15 mmHg
Explanation: Pressures:
Intra-abdominal pressure 20-25 mmHg for gas insufflation prior to primary trocar.
Distension pressure 12-15 mmHg once trocar insertion complete

7.When consenting someone for laparoscopy you discuss the risk of vascular injury. The
incidence of vascular injury during laparoscopy according to the Green‐top guidelines is?
A) 0.2/1000
B) 0.4/1000
C) 2/1000
D) 4/1000
E) 4/100.
Correct Answer: 0.2/1000
Explanation: Major vessel injury is the most important potential complication when undertaking
laparoscopy. It's incidence is 0.2/1000.
Bowel Injury is more common at 0.4/1000

8.A 26 year old women presents for her 12 week scan. She has been pregnant once before but
had a 1st trimester miscarriage. She reports no problems with this pregnancy and has had no
vaginal bleeding or spotting. The scan shows no fetal cardiac activity and a small gestational
sac. What is the likely diagnosis?
A. Recurrent Miscarriage
B. Complete Miscarriage
C. Missed Miscarriage
D. Inevitable Miscarriage
E. Threatened Miscarriage
Correct Answer: Missed Miscarriage
Explanation: As there has been no bleeding or expulsion of the products of conception this is a
missed miscarriage

9. A patient has been seeing you due to itching during pregnancy and you have diagnosed
cholestatic jaundice. What is the most appropriate advice regarding testing LFTs postnatally?
A. No postnatal testing required
B. Test 24 hours postnatally
C. Test 48 hours postnatally
D. Test 7 days postnatally
E. Test 10 days postnatally
Correct Answer: Test 10 days postnatally
Explanation: LFTs should be deferred for at least 10 days according to greentop guideline 43

10. A 32 year old women who is 25 weeks pregnant presents with vaginal bleeding and
cramping lower abdominal pain. On examination the cervix is closed. Fetal cardiac activity is
noted on ultrasound. What is the likely diagnosis?
A. Antepartum Haemorrhage
B. Threatened Miscarriage
C. Inevitable Miscarriage
D. Septic Miscarriage
E. None of the above
Correct Answer: Antepartum Haemorrhage
Explanation: Antepartum haemorrhage (APH) is defined as bleeding from or in to the genital
tract, occurring from 24+0 weeks of pregnancy and prior to the birth of the baby.
1. You wish to perform karyotype analysis on a patient that you suspect has Turner’s
syndrome. What is the most suitable form of consent which must be obtained in this case?
A. Consent from carer
B. Consent from court of law
C. Consent from next of kin if possible
D. Verbal consent required
E. Written consent required

2. A cardiotocograph shows type 1 variable decelerations. What is the cause of this feature?
a) Fetal head compression
b) Fetal hypoxia
c) Fetal movements
d) Placental insufficiency
e) Umbilical cord compression

3. Which of the following is the drug of choice for the treatment of Chlamydia trachomatis
infection during pregnancy?
A Amoxicillin
B Cephazolin
C Clindamycin
D Metronidazole
E Tetracycline
4.An 18‐year‐old Asian girl was found to be pregnant after missing her last menstrual period
despite her appropriate use of the oral contraceptive pill for the last two years.
She was found also to have been taking additional medication prescribed by a specialist two
months ago.
Which of the following accounts for the pill failure?
A Cimetidine
B Erythromycin
C Isoniazid
D Ketoconazole
E Rifamipicin

5.Which of the following following substances, when activated by phosphorylation, increases


phosphorylation of myosin, and subsequent uterine contractility?
A Actin
B Calcium ions
C Calmodulin
D Myosin light chain kinase (MLCK)
E Myosin light chain phosphatase (MLCP)

6.At what stage of pregnancy is there an increase in the serum alkaline phosphatase
concentration?
A First trimester
B No change in pregnancy
C Second trimester
D Third trimester
E Throughout pregnancy

7. A 26 year old patient attends the A&E department. She reports becoming acutely short of
breath with chest pain 1 hour prior to attending. She is not on any regular medication. Of note
she had an uncomplicated birth by normal vaginal delivery at term 3 weeks ago. Her
observations are as follows:
Blood pressure: 100/60 Heart Rate: 100
Respiratory Rate: 20 Oxygen sats: 94% on air
What is the likely diagnosis?
A. Amniotic fluid embolism
B. Myocardial Infarction
C. Muscular chest pain
D. Pneumothorax
E. Pulmonary embolism

8. You are performing a diagnostic laparoscopy on a patient. What is the appropriate distension
pressure upon completion of trocar insertion?
A. 1 hour
B. 2 hours
C. 3 hours
D. 4 hours
E. 6 hours

9. An 18 year old patient comes to see you in clinic. Her BMI is 25.0 and her BP is 122/80. She
is a non‐smoker and there is no personal or family history of VTE or migraine. She would like to
start the pill for her acne. She has used topical Zineryt in the past but still has moderate acne.
What is the most appropriate option?
A. Cerazette (Desogestrel)
B. Marvelon (Ethinylestradiol/Desogestrel)
C. Dianette (Co‐cyprindiol)
D. Norimin (Ethinylestradiol/Norethisterone)
E. Yasmin (ethinylestradiol / drosperinone)

10. Which of the following conditions would prevent prescription of a POP to a women or
warrant referral to a specialist contraceptive provider?
A. Current VTE on anticoagulants
B. AIDS using HAART
C. Raynauds
D. Liver tumour
E. Endometriosis
1.a surgical team presented their data demonstrating an increased rate of post-surgical wound
infection following gastro-intestinal surgery compared with published standards from the Royal
College of Surgeons. What is the most appropriate next step to be taken up by the team who is
undertaking audit in this case?
A. Data analysis
B. Data collection
C. Identify standards
D. Implement change
E. Needs assessment
Answer D

2.at a recent directorate meeting, an obstetrician has been nominated to undertake the next
clinical audit. What is the most appropriate next step for the team undertaking audit in this
case?
A.Data analysis
B. Data collection
C. Needs assessment
D. Identify standards
E.Implement change
Answer C
3. A 16-year-old woman presents with secondary amenorrhoea. She is healthy, with no past
medical history. Her BMI is 17. What is the most appropriate initial investigation?
a) Bone mineral density scan
b) Dehydroepiandrosterone sulphate (DHEAS) measurement
c) Follicle-stimulating hormone measurement
d) Karyotype
e) Thyroid function test
The answer is follicle-stimulating hormone measurement.
The likely diagnosis is amenorrhoea caused by weight loss, so follicle-stimulating hormone
measurements would be appropriate. Since she is clinically euthyroid, thyroid function tests
would be of limited value.

4. Whilst you are attending to a patient in the antenatal ward, the patient collapses and
becomes unresponsive. You open their airway but they are not breathing. What should you do
next?
a) Commence artificial ventilation
b) Commence cardiac compressions
c) Get help
d) Give a precordial thump
e) Left lateral tilt
The answer is get help. Basic and Advanced Life Support guidelines highlight the need to get
help if a patient has collapsed and is unresponsive. You should then place the patient in left
lateral tilt and commence cardiac compressions at a rate of 30:2. Artificial ventilation is not
mandatory in the ALS guidelines.

5. Polyglactin sutures are used extensively in surgical procedures, particularly to ligate vessels.
What are the key features of polyglactin sutures?
a) Braided, absorbable and synthetic
b) Braided, non-absorbable and synthetic
c) Non-braided, absorbable and natural
d) Non-braided, absorbable and synthetic
e) Non-braided, non-absorbable and natural
The answer is braided, absorbable and synthetic. Polyglactin sutures are used to ligate pedicles
and close the uterus during a caesarean section. To achieve this, the sutures are braided to
prevent the knots from slipping.

6. You are performing a diagnostic laparoscopy on a patient. What is the appropriate distension
pressure upon completion of trocar insertion?
A. 2‐5 mmHg
B. 12‐15 mmHg
C. 20‐25 mmHg
D. 35‐50 mmHg
E. 55‐65 mmHg
Correct Answer: 12‐15 mmHg
Explanation: Pressures:
Intra-abdominal pressure 20-25 mmHg for gas insufflation prior to primary trocar.
Distension pressure 12-15 mmHg once trocar insertion complete

7.When consenting someone for laparoscopy you discuss the risk of vascular injury. The
incidence of vascular injury during laparoscopy according to the Green‐top guidelines is?
A) 0.2/1000
B) 0.4/1000
C) 2/1000
D) 4/1000
E) 4/100.
Correct Answer: 0.2/1000
Explanation: Major vessel injury is the most important potential complication when undertaking
laparoscopy. It's incidence is 0.2/1000.
Bowel Injury is more common at 0.4/1000

8.A 26 year old women presents for her 12 week scan. She has been pregnant once before but
had a 1st trimester miscarriage. She reports no problems with this pregnancy and has had no
vaginal bleeding or spotting. The scan shows no fetal cardiac activity and a small gestational
sac. What is the likely diagnosis?
A. Recurrent Miscarriage
B. Complete Miscarriage
C. Missed Miscarriage
D. Inevitable Miscarriage
E. Threatened Miscarriage
Correct Answer: Missed Miscarriage
Explanation: As there has been no bleeding or expulsion of the products of conception this is a
missed miscarriage

9. A patient has been seeing you due to itching during pregnancy and you have diagnosed
cholestatic jaundice. What is the most appropriate advice regarding testing LFTs postnatally?
A. No postnatal testing required
B. Test 24 hours postnatally
C. Test 48 hours postnatally
D. Test 7 days postnatally
E. Test 10 days postnatally
Correct Answer: Test 10 days postnatally
Explanation: LFTs should be deferred for at least 10 days according to greentop guideline 43

10. A 32 year old women who is 25 weeks pregnant presents with vaginal bleeding and
cramping lower abdominal pain. On examination the cervix is closed. Fetal cardiac activity is
noted on ultrasound. What is the likely diagnosis?
A. Antepartum Haemorrhage
B. Threatened Miscarriage
C. Inevitable Miscarriage
D. Septic Miscarriage
E. None of the above
Correct Answer: Antepartum Haemorrhage
Explanation: Antepartum haemorrhage (APH) is defined as bleeding from or in to the genital
tract, occurring from 24+0 weeks of pregnancy and prior to the birth of the baby.

You might also like